SlideShare a Scribd company logo
1 of 58
Download to read offline
DIKTAT KULIAH (2 sks)
MX 127 Teori Bilangan
(Revisi Terakhir: Juli 2009 )

Oleh:
Didit Budi Nugroho, S.Si., M.Si.

Program Studi Matematika
Fakultas Sains dan Matematika
Universitas Kristen Satya Wacana
KATA PENGANTAR
Diktat ini merupakan catatan kuliah Teori Bilangan (MX 127) tingkat sarjana tahun
pertama yang diberikan di Universitas Kristen Satya Wacana dalam semester 1 tahun
2008-2009. Karena itu naskah ini disajikan dalam cara yang sangat dasar (elementer).
Elementer berarti hampir tidak ada Analisis yang digunakan, dan hampir tidak ada
Aljabar Abstrak.
Naskah ini dirancang untuk mencakup beberapa ide dasar teori bilangan dalam satu
semester. Selain itu, di sini juga disertakan masalah-masalah teori bilangan yang digunakan dalam berbagai pelatihan dan kompetisi matematika internasional untuk memotivasi dan memberikan tantangan kepada mahasiswa.
Penulis berharap bahwa naskah ini akan memberikan manfaat yang lebih dalam pengajaran Teori Bilangan. Untuk itu masih diperlukan masukan dan saran dari pembaca
demi perbaikan dan pengembangan naskah ini secara terus menerus.
Salatiga, Juli 2009
Didit B. Nugroho

i
DAFTAR ISI
KATA PENGANTAR

i

DAFTAR ISI

ii

DAFTAR SINGKATAN

iii

1 Aksioma Dasar untuk Z

1

2 Bukti dengan Induksi

3

3 Keterbagian
3.1 Sifat-sifat Keterbagian Elementer . . . . . . . . . . . . . . . . . . . . . .
3.2 Algoritma Pembagian . . . . . . . . . . . . . . . . . . . . . . . . . . . .
3.3 Beberapa Identitas Aljabar . . . . . . . . . . . . . . . . . . . . . . . . .

6
6
12
14

4 Kongruensi Zn
4.1 Kongruensi . . . . . .
4.2 Persamaan Kongruensi
4.3 Uji Keterbagian . . . .
4.4 Sisa lengkap . . . . . .

.
.
.
.

18
18
22
25
25

5 Faktorisasi Tunggal
5.1 FPB dan KPK . . . . . . . . . . . . . . . . . . . . . . . . . . . . . . . .
5.2 Bilangan Prima dan Faktorisasi . . . . . . . . . . . . . . . . . . . . . . .
5.3 Teorema Fermat dan Teorema Euler . . . . . . . . . . . . . . . . . . . .

27
27
30
34

6 Algoritma Euclid
6.1 Sistem Kongruensi Linear . . . . . . . . . . . . . . . . . . . . . . . . . .

36
41

7 Fungsi-fungsi Bilangan-Teoritik
7.1 Fungsi Floor . . . . . . . . . .
7.2 Fungsi Legendre . . . . . . . .
7.3 Bilangan Fermat . . . . . . . .
7.4 Bilangan Mersenne . . . . . . .
7.5 Bilangan Sempurna . . . . . . .

45
45
50
51
52
53

.
.
.
.

.
.
.
.

.
.
.
.

.
.
.
.

.
.
.
.

.
.
.
.

.
.
.
.
.

.
.
.
.

.
.
.
.
.

DAFTAR PUSTAKA

.
.
.
.

.
.
.
.
.

.
.
.
.

.
.
.
.
.

.
.
.
.

.
.
.
.
.

.
.
.
.

.
.
.
.
.

.
.
.
.

.
.
.
.
.

.
.
.
.

.
.
.
.
.

.
.
.
.

.
.
.
.
.

.
.
.
.

.
.
.
.
.

.
.
.
.

.
.
.
.
.

.
.
.
.

.
.
.
.
.

.
.
.
.

.
.
.
.
.

.
.
.
.

.
.
.
.
.

.
.
.
.

.
.
.
.
.

.
.
.
.

.
.
.
.
.

.
.
.
.

.
.
.
.
.

.
.
.
.

.
.
.
.
.

.
.
.
.

.
.
.
.
.

.
.
.
.

.
.
.
.
.

.
.
.
.

.
.
.
.
.

.
.
.
.

.
.
.
.
.

.
.
.
.
.

54

ii
DAFTAR SINGKATAN
USAMO
IMO
HMMT
AHSME
UMMC
SMC
AIME
Putnam
ARML
APMC

:
:
:
:
:
:
:
:
:
:

United States of America Mathematical Olympiad
International Mathematical Olympiad
Harvard–
MIT Math Tournament
American High School Mathematics Examination
University of Michigan Mathematics Competition
Stanford Mathematics Competition
American Invitational Mathematics Examination
The William Lowell Putnam Mathematical Competition
American Regional Mathematics League
Austrian–
Polish Mathematics Competition

iii
Bab 1

Aksioma Dasar untuk Z
Perhatian dalam teori bilangan yaitu pada sifat-sifat bilangan bulat
: : : ; 4; 3; 2; 1; 0; 1; 2; 3; 4; : : :
Karena itu pertama kali diperkenalkan beberapa notasi dan mengingat kembali beberapa sifat dasar dari bilangan bulat yang akan diperlukan pada bahasan-bahasan
selanjutnya:
N = f1; 2; 3; :::g

(himpunan semua bilangan asli atau bulat positif)

Z = f:::; 3; 2; 1; 0; 1; 2; 3; :::g
(himpunan semua bilangan bulat)
nn
o
Q =
: n; m 2 Z dan m 6= 0
(himpunan semua bilangan rasional)
m
R = himpunan semua bilangan riil
Dicatat bahwa N

Z

Q

R.

Beberapa aksioma dasar untuk Z:
1. Jika a; b 2 Z, maka a + b, a b, ab 2 Z. (Z dikatakan tertutup terhadap operasi
penjumlahan, pengurangan, dan perkalian)
2. Jika a 2 Z, maka tidak ada x 2 Z sedemikian sehingga a < x < a + 1.
3. Jika a; b 2 Z dan ab = 1, maka a = b = 1 atau a = b =

1.

4. Hukum eksponen: Untuk n; m 2 N dan a; b 2 R berlaku:
(a) (an )m = anm
(b) (ab)n = an bn
(c) an am = an+m :
Aturan-aturan di atas berlaku untuk semua n; m 2 Z jika a; b 6= 0.
5. Sifat ketaksamaan: Untuk a; b; c 2 R berlaku:
(a) Transitif : Jika a < b dan b < c, maka a < c.
(b) Jika a < b maka a + c < b + c.
(c) Jika a < b dan 0 < c maka ac < bc.
1
2

Bab 1. Aksioma Dasar untuk Z
(d) Jika a < b dan c < 0 maka bc < ac.
(e) Trikotomi : Diberikan a dan b, hanya berlaku salah satu dari:
a = b, a < b, b < a.

6. Sifat terurut baik (well-ordering ) untuk N: setiap himpunan bagian tak
kosong dari N memuat suatu elemen terkecil atau minimal. Suatu elemen terkecil
dari suatu himpunan bagian S
N adalah suatu elemen s0 2 S dimana s0 s
untuk setiap s 2 S.
7. Prinsip Induksi Matematis: Diambil P (n) sebagai suatu pernyataan menyangkut variabel bilangan asli n. Diambil n0 adalah suatu bilangan asli. P (n)
adalah benar untuk semua bilangan asli n
n0 jika kedua pernyataan berikut
ini berlaku:
PIM(a) P (n) benar untuk n = n0 .
PIM(b) Jika P (n) benar untuk n0

n

k, maka P (n) benar untuk n = k + 1.
Bab 2

Bukti dengan Induksi
Pada bab ini diberikan beberapa pernyataan yang dapat dibuktikan dengan menggunakan Prinsip Induksi Matematis, atau secara sederhana disebut induksi. Berikut ini
diberikan suatu pernyataan beserta bukti induksi.
Proposisi 2.1 Jika n

5 maka 2n > 5n.

Bukti. Di sini digunakan Prinsip Induksi Matematis.
PIM(a) Diambil P (n) adalah pernyataan 2n > 5n. Untuk n0 diambil 5. Secara
sederhana dapat dituliskan:
P (n) : 2n > 5n dan n0 = 5:
Sekarang jika n = 4 maka P (n) menjadi pernyataan 24 > 5 4 yang adalah salah.
Tetapi jika n = 5 , P (n) adalah pernyataan 25 > 5 5 atau 32 > 25 yang adalah
benar. Jadi P (n) benar untuk n = 5.
PIM(b) Diasumsikan bahwa P (k) benar untuk suatu bilangan bulat positif k
Artinya, diasumsikan bahwa
2k > 5k untuk suatu k 2 N dan k

5:

5.

(2.1)

Asumsi (2.1) dinamakan hipotesis induksi dan akan digunakan untuk membuktikan bahwa P (n) benar ketika n = k +1. Atau dengan kata lain akan dibuktikan
bahwa
2k+1 > 5 (k + 1)
(2.2)
dan dilakukan seperti berikut ini. Berdasarkan (2.1), ruas kiri dari (2.2) dapat
dituliskan sebagai
2k+1 = 2 2k > 2 5k = 10k;
dan karena 5k > 5 untuk setiap k 5, maka 10k = 5k + 5k > 5k + 5 = 5 (k + 1),
sehingga
2k+1 > 10k > 5 (k + 1) :
yang berarti bahwa P (n) benar ketika n = k + 1.
Disimpulkan bahwa P (n) berlaku untuk n

3

5:
4

Bab 2. Bukti dengan Induksi

Contoh 2.2 (USAMO 1978) Suatu bilangan bulat dikatakan bagus (good) jika dapat
dituliskan sebagai
n = a1 + a2 +
+ ak ;
dimana a1 ; a2 ;
memenuhi

; ak adalah bilangan-bilangan bulat positif (tidak perlu berbeda) yang
1
1
+
+
a1 a2

+

1
= 1:
ak

Diberikan informasi bahwa bilangan-bilangan bulat 33 sampai 73 adalah bagus, buktikan
bahwa setiap bilangan bulat 33 adalah bagus.
Bukti. Diambil n = 33, maka bilangan-bilangan bulat 33 sampai 73 dapat dituliskan
sebagai barisan
n; n + 1; n + 2; :::; 2n + 7
yang adalah bagus berdasarkan yang diketahui. Akan dibuktikan bahwa 2n + 8 dan
2n + 9 adalah bagus. Karena n adalah bagus, maka dapat dituliskan
2n + 8 = 2 (a1 + a2 +
= 2a1 + 2a2 +
dan

+ ak ) + 4 + 4
+ 2ak + 4 + 4

1
1
+
+
2a1 2a2

+

1 1
1 1 1
1
+ + = + + = 1:
2ak
4 4
2 4 4

Juga,
2n + 9 = 2a1 + 2a2 +
dan

1
1
+
+
2a1 2a2

+

+ 2ak + 3 + 6

1
1 1
1 1 1
+ + = + + = 1:
2ak
3 6
2 3 6

Oleh karena itu,
2n + 8 dan 2n + 9 adalah bagus.

Latihan 2.3 Buktikan bahwa 2n > 6n untuk n

5.

n (n + 1)
untuk n 1.
2
Latihan 2.5 Buktikan bahwa jika 0 < a < b maka 0 < an < bn untuk setiap n 2 N.
Latihan 2.4 Buktikan bahwa 1 + 2 +

+n=

Latihan 2.6 Buktikan bahwa n! < nn untuk n

2.

Latihan 2.7 Buktikan bahwa jika a; r 2 R dan r 6= 1, maka untuk n
a + ar + ar2 +

+ arn =

a rn+1 1
:
r 1

Ini dapat dituliskan seperti
a rn+1

1 = (r

1) a + ar + ar2 +

+ arn :

Dan kasus khusus pentingnya adalah
rn+1

1 = (r

1) 1 + r + r2 +

+ rn :

1 berlaku
5

Bab 2. Bukti dengan Induksi
Latihan 2.8 Buktikan bahwa 1 + 2 + 22 +
Latihan 2.9 Buktikan bahwa 111{z 1 =
|
}
n kali

+ 2n = 2n+1

10n 1
untuk n
9

Latihan 2.10 Buktikan bahwa 12 +22 +32 +

+n2 =

1 untuk n

1.

1.

n (n + 1) (2n + 1)
untuk n
6

1.
Bab 3

Keterbagian
3.1

Sifat-sifat Keterbagian Elementer

Pertama kali diperkenalkan pernyataan d j n yang dapat dibaca seperti berikut ini:
1. d membagi n.
2. d adalah pembagi dari n.
3. d adalah suatu faktor dari n.
4. n adalah kelipatan dari n.
Jadi, lima pernyataan di bawah ini adalah ekivalen, artinya semua cara yang berbeda
mengatakan hal yang sama.
1. 2 j 6:
2. 2 membagi 6.
3. 2 adalah pembagi dari 6.
4. 2 adalah suatu faktor dari 6.
5. 6 adalah kelipatan dari 2.
De…nisi 3.1 d j n berarti terdapat suatu bilangan bulat k sedemikian sehingga n = dk,
a
sedangkan d - n berarti bahwa d j n adalah salah. Dicatat bahwa a j b 6= .
b
Suatu cara lain untuk menyatakan de…nisi dari d j n adalah seperti berikut ini.
De…nisi 3.2 d j n () n = dk untuk suatu k. (Dicatat bahwa notasi () diinterpretasikan dengan arti jika dan hanya jika.)
Teorema 3.3 (Sifat-sifat keterbagian) Jika n, m, dan d adalah bilangan-bilangan
bulat maka pernyataan-pernyataan berikut ini adalah benar:
(1) n j n (sifat re‡eksif );
(2) d j n dan n j m =) d j m (sifat transitif );
6
7

Bab 3. Keterbagian

(3) d j n dan d j m =) d j an + bm untuk setiap bilangan bulat a dan b (sifat linier);
(4) d j n =) ad j an untuk a 6= 0 (sifat perkalian);
(5) ad j an dan a 6= 0 =) d j n (sifat penghapusan);
(6) 1 j n (1 membagi sembarang bilangan);
(7) n j 1 =) n =

1 (hanya 1 dan

1 yang merupakan pembagi dari 1);

(8) d j 0 (sembarang nilai membagi nol);
(9) 0 j n =) n = 0 (nol hanya membagi nol);
(10) d, n adalah positif dan d j n =) d

n (sifat perbandingan).

(11) d j n dan d j (n + m) =) d j m.
Contoh 3.4 Buktikan sifat 1 sampai 10 dalam Teorema 3.3.
Bukti. Untuk (1), dicatat bahwa n = 1 n. Untuk (2) sampai (5), (10) dan (11), syarat
d j n diberikan, artinya n = kd untuk suatu bilangan bulat k. Untuk (2), dipunyai
n j m, artinya m = k1 n, maka m = (k1 k) d atau d j m. Untuk (3), diasumsikan bahwa
m = k2 d, maka an + bm = (ka + k2 b) d. Untuk (4) dan (5), karena a 6= 0, d 6= 0 jika
dan hanya jika ad 6= 0. Dicatat bahwa n = kd jika dan hanya jika na = kda. Untuk (6),
dicatat bahwa n = n 1. Untuk (7), dicatat bahwa 1 = 1 1 atau 1 = ( 1) ( 1). Untuk
(8), dicatat bahwa 0 = d 0. Untuk (9), dipunyai 0 j n, artinya n = 0 k, maka n = 0.
Untuk (10), dicatat bahwa d; n > 0, maka jkj 1 dan juga n = jkj d d. Untuk (11),
dipunyai d j (n + m), artinya n + m = k1 d, maka kd + m = k1 d atau m = (k1 k) d
atau d j m.
De…nisi 3.5 Jika c = as + bt untuk suatu bilangan bulat s dan t, dikatakan bahwa c
merupakan suatu kombinasi linier dari a dan b.
Jadi, pernyataan (3) dalam Teorema 3.3 mengatakan bahwa jika d membagi a dan b,
maka d membagi semua kombinasi linear dari a dan b. Khususnya, d membagi a + b
dan a b.
Contoh 3.6 Diambil x dan y adalah bilangan bulat. Buktikan bahwa 2x + 3y dapat
dibagi oleh 17 jika dan hanya jika 9x + 5y dapat dibagi oleh 17.
Bukti. 17 j (2x + 3y) =) 17 j 13 (2x + 3y) atau 17 j (26x + 39y) =) 17 j
[(9x + 5y) + (17x + 34y)] =) 17 j (9x + 5y), dan sebaliknya, 17 j (9x + 5y) =)
17 j 4 (9x + 5y) atau 17 j (36x + 20y) =) 17 j [(2x + 3y) + (34x + 17y)] =)
17 j (2x + 3y).
Contoh 3.7 Tentukan semua bilangan bulat positif d sedemikian sehingga d membagi
n2 + 1 dan (n + 1)2 + 1 untuk suatu bilangan bulat n.
h
i
Penyelesaian. Diambil d j n2 + 1 dan d j (n + 1)2 + 1 atau d j n2 + 2n + 2 .

Jadi d j n2 + 2n + 2
n2 + 1 atau d j (2n + 1) =) d j 4n2 + 4n + 1 , sehingga
2 + 2n + 2
2 + 4n + 1
dj 4 n
4n
atau d j (4n + 7) : Jadi d j [(4n + 7) 2 (2n + 1)]
atau d j 5. Disimpulkan bahwa d adalah 1 atau 5. (Dapat ditunjukkan bahwa nilai
dicapai dengan mengambil n = 2:)
8

Bab 3. Keterbagian
Contoh 3.8 Buktikan bahwa 33n+3
untuk semua bilangan asli n.

26n

27 merupakan suatu kelipatan dari 169

Bukti. Di sini digunakan Prinsip Induksi Matematis.
PIM(a) Diambil P (n) adalah pernyataan 33n+3 26n 27 = 169 n, n 2 N. Untuk
n = 1, kita menyatakan bahwa 36 53 = 676 = 169 4 yang berarti dapat dibagi
oleh 169. Jadi P (n) benar untuk n = 1.
PIM(b) Diasumsikan bahwa pernyataan benar untuk n = k
33k

26k

1, k > 1, yaitu

1 = 169N

untuk suatu bilangan bulat N . Karena itu
33k+3

26k

27 = 27 33k

26k

27 = 27 33k

26k

1 + 676k

yang direduksi menjadi
27 169N + 169 4k,
yang dapat dibagi oleh 169, yang berarti bahwa P (n) benar untuk n = k.

Contoh 3.9 (IMO 1984) Diandaikan bahwa a1 , a2 , :::, a2n adalah bilangan-bilangan
bulat berbeda sedemikian sehingga persamaan
(x

a1 ) (x

a2 )

(x

( 1)n (n!)2 = 0

a2n )

mempunyai suatu penyelesaian bilangan bulat r. Tunjukkan bahwa
r=

a1 + a2 + ::: + a2n
:
2n

Bukti. Jelas r 6= ai untuk semua i, dan r
sehingga
j(r

a1 ) (r

a2 )

(r

a2n )j

ai adalah 2n bilangan bulat berbeda,

j(1) (2)

(n) ( 1) ( 2)

( n)j = (n!)2 ;

dengan kesamaan terjadi jika dan hanya jika
fr

a1 ; r

a2 ;

;r

a2n g = f1; 2;

; n; 1; 2;

; ng :

Oleh karena itu
(r

a1 ) + (r

a2 ) +

+ (r

a2n ) = 2nr

(a1 + a2 + ::: + a2n )

= 1+2+

+ n + ( 1) + ( 2) +

= 0
yang mengakibatkan
r=

a1 + a2 + ::: + a2n
:
2n

+ ( n)
9

Bab 3. Keterbagian

Himpunan bilangan bulat dapat dipartisi menjadi dua himpunan bagian, yaitu himpunan bilangan bulat ganjil dan himpunan bilangan genap:
f 1; 3; 5; :::g

dan

f0; 2; 4; :::g

berturut-turut.
Berikut ini diberikan beberapa ide dasar:
(1) suatu bilangan ganjil mempunyai bentuk 2k + 1, untuk suatu bilangan bulat k;
(2) suatu bilangan genap mempunyai bentuk 2k, untuk suatu bilangan bulat k;
(3) jumlahan dari dua bilangan ganjil adalah suatu bilangan genap;
(4) jumlahan dari dua bilangan genap adalah suatu bilangan genap;
(5) jumlahan dari suatu bilangan ganjil dan genap adalah suatu bilangan ganjil;
(6) hasil kali dari dua bilangan ganjil adalah suatu bilangan ganjil;
(7) hasil kali dari bilangan-bilangan bulat adalah genap jika dan hanya jika paling
sedikit dari faktor-faktornya adalah genap.
Contoh 3.10 Diambil n adalah suatu bilangan bulat yang lebih besar dari 1. Buktikan
bahwa
(a) 2n adalah jumlahan dari dua bilangan bulat ganjil berturutan.
(b) 3n adalah jumlahan dari tiga bilangan bulat berturutan.
Bukti. Untuk (a), dari hubungan 2n = 22+n 2 = 4 2n 2 dimisalkan k = 2n 2 , sehingga
karena 4k = (2k 1) + (2k + 1) maka diperoleh 2n = 4k = 2n 1 1 + 2n 1 + 1 .
Untuk (b), dari hubungan 3n = 31+n 1 = 3 3n 1 dimisalkan s = 3n 1 , sehingga karena
3s = (s 1) + s + (s + 1) maka diperoleh 3n = 3s = 3n 1 1 + 3n 1 + 3n 1 + 1 .
Contoh 3.11 Diambil k adalah suatu bilangan genap. Apakah mungkin untuk menuliskan 1 sebagai jumlahan dari kebalikan k bilangan ganjil?
Penyelesaian. Diasumsikan bahwa
1=

1
+
n1

+

1
nk

untuk bilangan-bilangan ganjil n1 , ..., nk ; maka diperoleh n1
nk = s1 +
+ sk ,
dimana semua si adalah ganjil. Ini tidaklah mungkin terjadi karena ruas kiri adalah
ganjil dan ruas kanan adalah genap.
Contoh 3.12 (HMMT 2004) Andi memilih lima bilangan dari himpunan f1, 2, 3,
4, 5, 6, 7g. Selanjutnya Andi memberitahu Vian berapa hasil kali dari bilangan-bilangan
terpilih tersebut, yang tidak akan menjadi informasi yang cukup bagi Vian untuk membayangkan apakah jumlahan dari bilangan-bilangan terpilih adalah genap atau ganjil.
Berapa hasil kali dari bilangan-bilangan terpilih tersebut?
10

Bab 3. Keterbagian

Penyelesaian. Mencari hasil kali dari bilangan-bilangan terpilih ekivalen dengan
mengetahui hasil kali dari dua bilangan yang tidak terpilih. Hasil kali dari bilanganbilangan tidak terpilih dan mungkin diperoleh dari lebih satu pasangan bilangan yaitu
hanya 12 ({3,4} dan {2,6}) dan 6 ({1,6} dan {2,3}). Tetapi dalam kemungkinan kedua, jumlahan dari dua bilangan (tidak terpilih) adalah ganjil (dan juga lima bilangan
terpilih mempunyai jumlahan ganjil), sehingga belum tentu benar. Oleh karena itu kemungkinan pertama pasti benar, dan hasil kali dari lima bilangan terpilih sama dengan
1 2 3
12

7

= 420:

Contoh 3.13 Buktikan bahwa
1+

p

2n

2

p

+ 1

2n

2

adalah suatu bilangan bulat genap dan bahwa
1+

p

2

2n

p

1

2n

2

p
=b 2

untuk suatu bilangan bulat positif b, untuk semua bilangan bulat n

1.

Bukti. Diproses dengan induksi pada n 2 N.

p 2n
p 2n
2
PIM(a) Diambil P (n) adalah pernyataan: 1 + 2 + 1
adalah genap dan
p 2n
p
p 2n
1
2
= b 2 untuk suatu b 2 N. Untuk n = 1, dipunyai
1+ 2
1+

p

2

2

+ 1

p

2

2

=6

yang adalah bilangan genap, dan
1+

p

2

2

1

p

2

2

p
= 4 2:

Oleh karena itu P (1) adalah benar.
PIM(b) Diasumsikan bahwa P (n) benar untuk n = k 1, k > 1, yaitu diasumsikan
bahwa
p 2(k 1)
p 2(k 1)
1+ 2
+ 1
2
= 2N
untuk suatu bilangan bulat N dan bahwa
1+

p

2

2(k 1)

untuk suatu bilangan bulat positif a.

1

p

2

2(k 1)

p
=a 2
11

Bab 3. Keterbagian
Sekarang diperhatikan
1+

p

2k

2

+ 1

p

2

2k

=

1+
+ 1

p

2

2
p

2

p
3+2 2

=

+ 3
= 3

p

2

2

p

p
p

1
2(k 1)

2k 2

2

1

1+

p
2 2

1+

1+

2

2
p

2k 2

2(k 1)

2

2(k 1)

+ 1

p

2

2(k 1)

p
p 2(k 1)
1
2
1+ 2
p
p
= 6N + 2 2 a 2 = 2 (3N + 2a)
p
2 2

+

2(k 1)

yang merupakan suatu bilangan bulat genap dan secara serupa
1+

p

2

2k

1

p

2

2k

p
p
p
= 3a 2 + 2 2 (2N ) = (3a + 4N ) 2:

Jadi P (k) adalah benar.

Contoh 3.14 (USAMO 2003) Buktikan bahwa untuk setiap bilangan bulat positif n
terdapat suatu bilangan n-digit yang dapat dibagi oleh 5n dimana semua digit-nya ganjil.
Bukti. Diproses dengan induksi pada n 2 N.
PIM(a) Pernyataan adalah benar untuk n = 1 karena terdapat bilangan satu digit
yang dapat dibagi oleh 5, yaitu 5.
PIM(b) Diasumsikan bahwa N = a1 a2 :::an dapat dibagi oleh 5n dan hanya mempunyai digit-digit ganjil. Diperhatikan bilangan-bilangan
N1 = 1a1 a2 :::an = 1 10n + 5n M = 5n (1 2n + M ) ;
N2 = 3a1 a2 :::an = 3 10n + 5n M = 5n (3 2n + M ) ;
N3 = 5a1 a2 :::an = 5 10n + 5n M = 5n (5 2n + M ) ;
N4 = 7a1 a2 :::an = 7 10n + 5n M = 5n (7 2n + M ) ;
N5 = 9a1 a2 :::an = 9 10n + 5n M = 5n (9 2n + M ) :
Diperhatikan dua kemungkinan. Kemungkinan pertama, bilangan-bilangan 1
2n +M , 3 2n +M , 5 2n +M , 7 2n +M , 9 2n +M memberikan sisa-sisa yang berbeda
ketika dibagi oleh 5. Kemungkinan kedua, beda dari dua diantaranya merupakan
kelipatan dari 5, yang adalah tidak mungkin karena 2n bukan kelipatan dari 5
dan bukan beda dari sembarang bilangan-bilangan 1, 3, 5, 7, 9. Karena itu
yang benar adalah kemungkinan pertama, dan ini berarti bahwa satu di antara
bilangan-bilangan N1 , N2 , N3 , N4 , N5 dapat dibagi oleh 5n 5.
12

Bab 3. Keterbagian
Latihan 3.15 Buktikan bahwa jika d j a dan d j b maka d j a
Latihan 3.16 Buktikan bahwa jika d j a dan d j a

b.

b maka d j b.

Latihan 3.17 Buktikan bahwa jika d j n dan n 6= 0 maka jdj

jnj.

Latihan 3.18 Buktikan bahwa jika d j n dan n j d maka jdj = jnj.
Latihan 3.19 Buktikan bahwa jika d j n dan n 6= 0 maka

n
j n.
d

Latihan 3.20 Buktikan bahwa jika a 2 Z maka pembagi positif dari a dan a + 1 hanya
1:
Latihan 3.21 Diambil a dan b adalah bilangan bulat positif sedemikian sehingga a j b2 ,
b2 j a3 , a3 j b4 , b4 j a5 , ... . Buktikan bahwa a = b.

3.2

Algoritma Pembagian

Tujuan dari bagian ini adalah membuktikan hasil penting berikut ini.
Teorema 3.22 (Algoritma Pembagian) Jika a dan b adalah bilangan bulat dan b >
0 maka terdapat secara tunggal bilangan bulat q dan r yang memenuhi dua kondisi:
a = bq + r

dan

0

r < b.

(3.1)

Dalam situasi ini q dinamakan hasil bagi (quotient) dan r dinamakan sisa (remainder )
ketika a dibagi oleh b. Dicatat bahwa terdapat dua bagian untuk hasil ini. Satu bagian
adalah EKSISTENSI dari bilangan bulat q dan r yang memenuhi (3.1) dan bagian
kedua adalah KETUNGGALAN dari bilangan bulat q dan r yang memenuhi (3.1).
Bukti. Pertama kali diperkenalkan fungsi ‡oor :
bxc = bilangan bulat terbesar yang lebih kecil atau sama dengan x
dimana x adalah sembarang bilangan riil. Dipunyai sifat bahwa x
Bahasan lebih lanjut mengenai fungsi ‡oor diberikan dalam Bab 7.
Sekarang diambil b > 0 dan sembarang a mende…nisikan
jak
q =
b
r = a bq:

1 < bxc

Secara jelas dipunyai a = bq+r. Tetapi kita perlu untuk membuktikan bahwa 0
Berdasarkan sifat fungsi ‡oor dipunyai
jak a
a
1<
:
b
b
b
Sekarang dikalikan semua suku dari ketaksamaan dengan
jak
b a> b
a:
b

x.

r < b.

b yang akan menghasilkan
13

Bab 3. Keterbagian
Jika ditambahkan a ke semua ruas dari ketaksamaan dan
diperoleh
b > a bq 0:

jak
b

diganti dengan q maka

Karena r = a bq, maka persamaan terakhir memberikan hasil 0 r < b.
Kita tetap harus membuktikan bahwa q dan r ditentukan secara tunggal. Untuk itu
diasumsikan bahwa
a = bq1 + r1 dan 0 r1 < b;
dan
a = bq2 + r2

dan

0

r2 < b:

Kita harus menunjukkan bahwa r1 = r2 dan q1 = q2 . Jika r1 6= r2 , tanpa kehilangan
keumuman, dapat diasumsikan bahwa r2 > r1 . Pengurangan kedua persamaan di atas
akan menghasilkan
0=a

a = (bq1 + r1 )

(bq2 + r2 ) = b (q1

q 2 ) + r1

r2 :

Ini mengakibatkan
r2

r1 = b (q1

q2 ) :

(3.2)

Ini berarti bahwa b j (r2 r1 ). Berdasarkan Teorema 3.3 nomor (10), ini mengakibatkan bahwa b r2 r1 . Tetapi karena
0

r1 < r2 < b

maka dipunyai r2 r1 < b. Ini kontradiksi dengan b
r2 r1 . Jadi kita harus
menyimpulkan bahwa r1 = r2 . Sekarang dari (3.2) kita mempunyai 0 = b (q1 q2 ).
Karena b > 0, akibatnya q1 q2 = 0, artinya q1 = q2 . Ini melengkapi bukti ketunggalan
dari q dan r.
Contoh 3.23 (AHSME 1976) Diambil r adalah sisa ketika 1059, 1417 dan 2312
dibagi oleh b > 1. Tentukan nilai dari b r.
Penyelesaian. Berdasarkan Algoritma Pembagian, 1059 = q1 b+r, 1417 = q2 b+r, dan
2312 = q3 b + r untuk bilangan-bilangan bulat q1 , q2 , q3 . Dari sini, 358 = 1417 1059 =
b (q2 q1 ), 1253 = 2312 1059 = b (q3 q1 ), dan 895 = 2312 1417 = b (q3 q2 ).
Karena itu b j 358 atau b j 2 179, b j 1253 atau b j 7 179, dan b j 895 atau b j 5 179.
Karena b > 1, disimpulkan bahwa b = 179. Jadi (sebagai contoh) 1059 = 5 179 + 164,
yang berarti bahwa r = 164. Disimpulkan bahwa b r = 179 164 = 15.
Contoh 3.24 Tunjukkan bahwa n2 + 23 dapat dibagi oleh 24 untuk n tak hingga banyaknya.
Bukti. n2 + 23 = n2 1 + 24 = (n 1) (n + 1) + 24. Jika diambil n = 24k
k = 0; 1; 2; :::, maka pernyataan dapat dibagi oleh 24.

1,

De…nisi 3.25 Suatu bilangan prima (prime) p adalah bilangan bulat positif lebih besar
1 yang pembagi positifnya hanya 1 dan p. Jika bilangan bulat n > 1 bukan prima, maka
bilangan tersebut dinamakan bilangan composite.
Dicatat bahwa bilangan 1 bukan bilangan prima ataupun composite.
14

Bab 3. Keterbagian
Contoh 3.26 Tunjukkan bahwa jika p > 3 adalah prima, maka 24 j p2

1 .

Bukti. Berdasarkan Algoritma Pembagian, sembarang bilangan bulat dapat dinyatakan sebagai salah satu dari: 6k, 6k 1, 6k 2, atau 6k + 3, dengan k 2 Z: Jika p > 3
adalah prima, maka p mempunyai bentuk p = 6k 1 (karena pilihan lainnya dapat
dibagi 2 atau 3). Dicatat bahwa (6k 1)2 1 = 36k 2 12k = 12k (3k 1). Karena k
atau 3k 1 adalah genap, maka 12k (3k 1) dapat dibagi oleh 24.
Contoh 3.27 Buktikan bahwa kuadrat dari sembarang bilangan mempunyai bentuk 4k
atau 4k + 1.
Bukti. Berdasarkan Algoritma Pembagian, sembarang bilangan bulat dapat dinyatakan sebagai salah satu dari: 2a atau 2a + 1. Dikuadratkan,
(2a)2 = 4a2 ; (2a + 1)2 = 4 a2 + a + 1;
sehingga pernyataan adalah benar.
Contoh 3.28 Buktikan bahwa tidak ada bilangan bulat dalam barisan
11; 111; 1111; 11111; :::

(3.3)

yang merupakan kuadrat dari suatu bilangan bulat.
Bukti. Sudah diperoleh bahwa kuadrat dari sembarang bilangan bulat mempunyai
bentuk 4k atau 4k + 1. Semua bilangan dalam barisan (3.3) mempunyai bentuk 4k 1,
yang berarti tidak bisa menjadi kuadrat dari sembarang bilangan bulat.
Contoh 3.29 Tunjukkan bahwa dari sembarang tiga bilangan bulat, selalu dapat dipilih
dua diantaranya, misalnya a dan b, sehingga a3 b ab3 dapat dibagi 10.
Bukti. Jelas bahwa a3 b ab3 = ab (a b) (a + b) selalu genap. Jika satu dari tiga
bilangan bulat mempunyai bentuk 5k, maka selesai (misalnya diambil a = 5k). Jika
tidak, dipilih tiga bilangan yang terletak dalam klas-klas sisa 5k 1 atau 5k 2.
Dua dari tiga bilangan bulat pasti terletak di salah satu dari dua kelompok tersebut.
Akibatnya jumlah atau selisih dari dua bilangan tersebut berbentuk 5k dan diperoleh
hasil yang diinginkan.
Contoh 3.30 Buktikan bahwa jika 3 j a2 + b2 , maka 3 j a dan 3 j b.
Bukti. Dibuktikan dengan kontraposisi seperti berikut ini. Diandaikan bahwa 3 - a
atau 3 - b, dan akan dibuktikan bahwa 3 - a2 + b2 . Dari hipotesis dapat dinyatakan
a = 3k 1 atau b = 3m 1. Dari sini diperoleh a2 = 3 3k 2 2k + 1 atau a2 = 3x + 1,
dan serupa dengan itu b2 = 3y + 1. Jadi a2 + b2 = 3x + 1 + 3y + 1 = 3s + 2, yang berarti
3 - a2 + b2 .

3.3

Beberapa Identitas Aljabar

Pada bagian ini diberikan beberapa contoh dimana penyelesaiannya tergantung pada
penggunaan beberapa identitas aljabar elementer.
15

Bab 3. Keterbagian
Contoh 3.31 Tentukan semua bilangan prima berbentuk n3
n > 1.

1, untuk bilangan bulat

Penyelesaian. n3 1 = (n 1) n2 + n + 1 . Jika pernyataan ini merupakan bilangan
prima, karena n2 +n+1 > 1, pasti dipunyai n 1 = 1, yang berarti n = 2. Jadi bilangan
prima yang dimaksud hanyalah 7.
Contoh 3.32 Buktikan bahwa n4 + 4, n 2 N, adalah prima hanya untuk n = 1.
Bukti. Diamati bahwa
n4 + 4 = n4 + 4n2 + 4
2

4n2

(2n)2

=

n2 + 2

=

n2 + 2n + 2

=

(n + 1)2 + 1

n2

2n + 2
1)2 + 1 :

(n

Setiap faktor lebih besar 1 untuk n > 1, akibatnya n4 + 4 bukanlah prima.
1 yang memenuhi n4 + 4n adalah

Contoh 3.33 Tentukan semua bilangan bulat n
prima.

Penyelesaian. Untuk n = 1, jelas bahwa pernyataan merupakan bilangan prima.
Lebih lanjut haruslah diambil n adalah ganjil. Untuk n 3, semua bilangan di bawah
ini adalah bulat:
n4 + 22n = n4 + 2n2 2n + 22n
2

2n2 2n

n 2 + 2n

=

n2 + 2n + n2 2 (n+1)

n2 2 (n+1)
1

Ini mudah dilihat bahwa jika n
tersebut bukan prima.

2

1

=

n 2 + 2n

1

n2 2 (n+1) :

3, maka setiap faktor lebih besar 1, sehingga bilangan

Contoh 3.34 Buktikan bahwa untuk semua n 2 N, n2 membagi (n + 1)2

1.

Bukti. Jika n = 1, maka pernyataan benar. Sekarang diandaikan n > 1, maka
berdasarkan Teorema Binomial,
(n + 1)2

1=

n
X n
nk ;
k
k=1

dan setiap sukunya dapat dibagi oleh n2 :
Contoh 3.35 Buktikan bahwa
xn
Jadi x

y n = (x

y) xn

y selalu membagi xn

1

+ xn

yn.

2

y + xn

3 2

y +

+ xy n

2

+ yn

1

:
16

Bab 3. Keterbagian

Bukti. Diasumsikan bahwa x 6= y dan xy 6= 0. Dalam kasus ini, hasil di atas mengikuti
identitas
n 1
X
an 1
ak = a
6= 1;
a 1
k=0

x
pada pengambilan a = dan dikalikan dengan y n .
y
Sebagai contoh, tanpa penghitungan dapat dilihat bahwa 87672345
dibagi 666:

81012345 dapat

Contoh 3.36 (E½ tv½ s 1899) Tunjukkan bahwa
o o
2903n

803n

464n + 261n

dapat dibagi oleh 1897 unuk semua bilangan asli n.
Bukti. Berdasarkan hasil sebelumnya, 2903n 803n dapat dibagi oleh 2903 803 =
2100 = 7 300, dan 261n 464n dapat dibagi oleh 261 464 = 203 = 7 ( 29). Jadi
pernyataan 2903n 803n 464n + 261n dapat dibagi oleh 7. Dan juga, 2903n 464n
dapat dibagi oleh 2903 464 = 9 271 dan 261n 803n dapat dibagi oleh 261 803 =
542 = ( 2) 271. Jadi pernyataan juga dapat dibagi oleh 271. Karena 7 dan 271 tidak
mempunyai faktor prima yang sama (kecuali 1), maka disimpulkan bahwa pernyataan
dapat dibagi oleh 7 271 = 1897.
Contoh 3.37 (UMMC 1987) Diberikan bahwa 1002004008016032 mempunyai suatu
faktor prima p > 250000, cari bilangan tersebut.
Penyelesaian. Jika a = 103 dan b = 2, maka
1002004008016032 = a5 + a4 b + a3 b2 + a2 b3 + ab4 + b5 =

a6
a

b6
:
b

Pernyataan terakhir dinyatakan sebagai
a6
a

b6
b

= (a + b) a2 + ab + b2

a2

ab + b2

= 1002 1002004 998004
= 4 4 1002 250501 k;
dimana k < 250000. Oleh karena itu p = 250501.
Contoh 3.38 (Grünert 1856) Jika x; y; z; n 2 N dan n
hubungan
xn + y n = z n :

z, maka tidak berlaku

Bukti. Jelas bahwa jika berlaku hubungan xn + y n = z n untuk x; y; z 2 N, maka
x < z dan y < z. Berdasarkan kesimetrian, bisa diandaikan bahwa x < y. Selanjutnya
pernyataan dibuktikan dengan kontraposisi. Jadi diandaikan bahwa xn + y n = z n dan
n z, maka
zn

y n = (z

y) z n

1

+ yz n

2

+

+ yn

yang bertentangan dengan pernyataan xn + y n = z n .

1

1 nxn

1

> xn
17

Bab 3. Keterbagian
Contoh 3.39 Buktikan bahwa untuk n ganjil,
xn + y n = (x + y) xn

1

xn

2

y + xn

3 2

y

+

+ xy n

2

+ yn

1

:

Jadi jika n adalah ganjil, maka x + y membagi xn + y n .
Bukti. Bukti diperoleh dengan pensubstitusian y untuk y dalam Contoh 3.35 dan
diperhatikan bahwa ( y)n = y n untuk n ganjil.
Contoh 3.40 Tunjukkan bahwa 1001 membagi
11993 + 21993 + 31993 +

+ 10001993 :

Penyelesaian. Berdasarkan contoh sebelumnya karena setiap 11993 + 10001993 , 21993 +
9991993 , ..., 5001993 + 50011993 dapat dibagi oleh 1001.
Contoh 3.41 (SMC 250) Tunjukkan bahwa untuk sembarang bilangan asli n, terdapat bilangan asli lain x sedemikian sehingga setiap suku dari barisan
x

x + 1; xx + 1; xx + 1; :::
dapat dibagi oleh n.
Bukti. Cukup diambil x = 2n

1.
Bab 4

Kongruensi Zn
4.1

Kongruensi

De…nisi 4.1 Diambil n 2 N. Untuk x; y 2 Z, x dikatakan kongruen dengan y modulo
n jika n j (y x) dan dituliskan x = y (mod n) atau . Selanjutnya y dinamakan sisa
n
dari x ketika dibagi oleh n.
Lebih lanjut, modulo n adalah relasi ekivalensi pada Z, artinya untuk x; y; z 2 Z
berlaku:
1. (re‡eksif ) x = x (mod n),
2. (simetris) x = y (mod n) =) y = x (mod n);
3. (transitif ) x = y (mod n) dan y = z (mod n) =) x = z (mod n).
Klas kongruensi dari suatu bilangan bulat x modulo n, dinotasikan dengan xn (seringkali juga digunakan notasi x atau [x]n ), adalah himpunan dari semua bilangan bulat
yang kongruen dengan x modulo n. Dengan kata lain, menggunakan de…nisi keterbagian,
xn = fy 2 Z : n j (y
= fy 2 Z : y

x)g

x = kn; k 2 Zg

= fx + kn : k 2 Zg .

Himpunan dari klas-klas kongruensi dinotasikan dengan Zn . Elemen-elemen berbeda
dari Zn biasanya didaftar seperti
0n ; 1n ; 2n ; :::; (n

1)n :

Contoh 4.2 Diambil n = 4, maka dipunyai klas-klas kongruensi dari Zn :
08 = f0 + 4k : k 2 Zg = f0; 4; 8; :::g ;

14 = f1 + 4k : k 2 Zg = f:::; 7; 3; 1; 5; 9; :::g ;

24 = f2 + 4k : k 2 Zg = f:::; 6; 2; 2; 6; 10; :::g ;

34 = f3 + 4k : k 2 Zg = f:::; 5; 1; 3; 7; 11; :::g :

Klas-klas ekivalensi dapat dijumlahkan dan dikalikan menggunakan sifat berikut ini.
18
19

Bab 4. Kongruensi Zn
Lemma 4.3 Diambil n 2 N. Pada Zn berlaku rumus
xn

yn = (x

y)n ;

xn yn = (xy)n :

0
Bukti. Jika x0 = xn dan yn = yn maka
n

x0 + y 0 = x + y + x0
0 0

xy

=

0

x+ x

x + y0

x
0

= xy + y x

y

0

y + x0

y+ y
x +x y

y = x + y (mod n);
0

y0

x

y

= xy (mod n):

Selanjutnya, dengan mengaplikasikan rumus perkalian di atas akan diperoleh sifat
bahwa jika a = b (mod n) maka ak = bk .
Contoh 4.4 Tentukan sisa ketika 61987 dibagi oleh 37.
Penyelesaian. Dapat dituliskan 61987 = 6 61986 = 6 62
62 = 1 (mod 37). Jadi 61987 = 6 = 31 (mod 37).

993

= 6 ( 1)993 karena

Contoh 4.5 Cari digit terakhir dalam ekspansi desimal dari 4100 .
Penyelesaian. Digit terakhir adalah sisa ketika dibagi oleh 10. Jadi harus dihitung
klas kongruensi dari 4100 (mod 10). Dipunyai 42 = 6(mod 10), dan selanjutnya 62 =
6(mod 10). Jadi 4100 = (42 )50 = 650 = 6(mod 10).
7

Contoh 4.6 Cari digit satuan dari 77 :
7

Penyelesaian. Harus dicari 77 (mod 10). Diamati bahwa 72 = 1 (mod 10), 73 =
2
72 7 = 7 = 3 (mod 10), dan 74 = 72 = 1 (mod 10). Selain itu, 72 = 1 (mod 4)
3
dan 77 = 72
7 = 3 (mod 4), yang berarti bahwa terdapat suatu bilangan bulat t
sedemikian sehingga 77 = 3 + 4t. Sekarang diperoleh
7

77 = 74t+3 = 74

t

73 = 1t 3 = 3 (mod 10):

Jadi digit terakhir adalah 3.
Contoh 4.7 (Putnam 1986) Berapakah digit satuan dari
1020000
?
10100 + 3
Penyelesaian. Dimisalkan a
1020000
10100 + 3

=

3 = 10100 , maka
(a

200

3)200
1 X 200 200
=
a
a
a
k

k

( 3)k

k=0

=

199
X
k=0

200 199
a
k

k

( 3)k :

(4.1)
20

Bab 4. Kongruensi Zn

Karena

200
X
k=0

( 1)k

200
k

3

199

= 0, maka untuk a = 3 (mod 10) persamaan (??) menjadi
199
X
k=0

( 1)k

200
k

=

3199 = 3(mod 10):

Contoh 4.8 Apakah 4100 dapat dibagi 3?
Penyelesaian. Karena 4100 = 1100 = 1(mod 3), berarti 3 j 4100 .
Contoh 4.9 Buktikan bahwa 7 membagi 32n+1 + 2n+2 untuk setiap n 2 N.
Bukti. Diamati bahwa 32n+1 = 3 9n = 3 2n (mod 7) dan 2n+2 = 4 2n (mod 7).
Karena itu
32n+1 + 2n+2 = 7 2n = 0 (mod 7),
untuk setiap n 2 N.
Contoh 4.10 Buktikan hasil Euler: 641 j 232 + 1 .
Bukti. Diamati bahwa 641 = 27 5 + 1 = 24 + 54 . Karena itu 27 5 = 1 (mod 641)
dan 54 = 24 (mod 641). Sekarang, 27 5 = 1 (mod 641) menghasilkan 54 228 =
4
5 27 = 1 (mod 641). Kongruensi terakhir dan 54 = 24 (mod 641) menghasilkan
32 = 24 228 = 54 228 = 1 (mod 641), yang berarti bahwa 641 j 232 + 1 .
2
Contoh 4.11 Tentukan bilangan-bilangan kuadrat sempurna di modulo 13:
Penyelesaian. Karena r2 = (13 r)2 , maka hanya dipunyai r = 0; 1; :::; 6. Diamati
bahwa 02 = 0, 12 = 1, 22 = 4, 32 = 9, 42 = 3, 52 = 12, dan 62 = 10 (mod 13). Jadi
kuadrat sempurna di modulo 13 yaitu 0; 1; 4; 9; 3; 12; 10.
Contoh 4.12 Buktikan bahwa tidak ada bilangan bulat yang memenuhi x2

5y 2 = 2:

Bukti. Jika x2 = 2 + 5y 2 , maka x2 = 2 (mod 5). Tetapi karena 2 bukanlah kuadrat
sempurna modulo 5, maka dapat disimpulkan tidak ada bilangan bulat yang memenuhi
x2 5y 2 = 2:
Contoh 4.13 Buktikan bahwa 7 j 22225555 + 55552222 .
Bukti. Diamati bahwa 2222 = 3 (mod 7), 5555 = 4 (mod 7), 35 = 5 (mod 7), dan
42 = 2 = 5 (mod 7). Diperoleh
22225555 + 55552222 = 35555 + 42222
= 35(1111) + 42(1111)
= 51111

51111 = 0 (mod 7):

Contoh 4.14 Cari bilangan-bilangan bulat n sedemikian sehingga 2n + 27 dapat dibagi
oleh 7.
21

Bab 4. Kongruensi Zn

Penyelesaian. Diamati bahwa 21 = 2, 22 = 4, 23 = 1, 24 = 2, 25 = 4, 26 = 1 (mod 7)
dan juga 23k = 1 (mod 3) untuk semua k 2 N. Karena itu 23k + 27 = 1 + 27 = 0 (mod 7)
untuk semua k 2 N. Jadi n = 3k; k 2 N.
Contoh 4.15 Tentukan semua bilangan bulat n sedemikian sehingga 13 j 4(n2 + 1).
Penyelesaian. Ini adalah ekivalen dengan menyelesaikan kongruensi 4(n2 + 1) =
0(mod 13). Karena faktor persekutuan terkecil dari 4 dan 3 adalah 1, maka kita dapat
menghapus 4 untuk mendapatkan n2 = 1(mod 13). Penghitungan kuadrat-kuadrat
di modulo 13 memberikan ( 1)2 = 1, ( 2)2 = 4, ( 3)2 = 9, ( 4)2 = 3(mod 13),
( 5)2 = 1(mod 13), dan ( 6)2 = 3(mod 13). Jadi, telah dilakukan penghitungan
untuk perwakilan dari setiap klas kongruensi, sehingga jawaban untuk pertanyaan asli
adalah x 5(mod 13).
Contoh 4.16 Di modulo 7, apakah ada x; y 2 N sedemikian sehingga x3 = 2y + 15?
Penyelesaian. Diamati bahwa setiap pangkat dari 2 kongruen dengan 1; 2; atau
4 (mod 7). Jadi 2y + 15 = 2; 3; atau 5 (mod 7). Di sisi lain, karena pangkat tiga
sempurna di modulo 7 yaitu 0; 1, dan 6, maka tidak mungkin terjadi 2y + 15 = x3 .
Disimpulkan tidak ada x; y 2 N sedemikian sehingga x3 = 2y + 15.
Contoh 4.17 Buktikan bahwa 2k
ketika dibagi oleh 7.

5, k = 0; 1; 2; ::: tidak pernah mempunyai sisa 1

Bukti. Diamati bahwa 21 = 24 = ::: = 2, 22 = 25 = ::: = 4, 20 = 23 = 26 = ::: =
1 (mod 7). Jadi 2k 5 = 3; 4; atau 6 yang tidak lain adalah sisa atas pembagian oleh
7.
Contoh 4.18 (AIME 1994) Barisan naik
3; 15; 24; 48; :::

(4.2)

terdiri dari kelipatan positif dari 3 dan kurang satu dari suatu kuadrat sempurna. Berapakah sisa dari suku ke-1994 dari barisan tersebut ketika dibagi oleh 1000?
Penyelesaian. Diinginkan 3 j n2 1 atau 3 j (n 1) (n + 1). Karena 3 adalah
bilangan prima, ini mengharuskan n = 3k + 1 atau n = 3k 1, k = 1; 2; 3; :::. Barisan
3k + 1, k = 1; 2; ::: menghasilkan suku-suku n2 1 = (3k + 1)2 1 yang merupakan
suku-suku di posisi genap dari barisan (4.2). Barisan 3k 1, k = 1; 2; ::: menghasilkan
suku-suku n2 1 = (3k 1)2 1 yang merupakan suku-suku di posisi ganjil dari
barisan (4.2). Selanjutnya harus dicari suku ke-997 dari barisan 3k + 1, k = 1; 2; :::,
yaitu (3 997 + 1)2 1 = (3 ( 3) + 1)2 1 = 82 1 = 63 (mod 1000). Jadi, sisa yang
dicari adalah 63.
Contoh 4.19 (USAMO 1979) Tentukan semua penyelesaian tak negatif
(n1 ; n2 ; :::; n14 )
di modulo 16, jika ada, dari persamaan
n4 + n4 +
1
2

n4 = 1599.
14

(4.3)
22

Bab 4. Kongruensi Zn

Penyelesaian. Semua pangkat 4 sempurna di modulo 16 adalah 0; 1 (mod 16). Ini
berarti bahwa
n4 + n4 +
n2
1
2
14
paling besar adalah 14 (mod 16), padahal 1599 = 15 (mod 16). Jadi tidak ada penyelesaian untuk (4.3).
Contoh 4.20 Diambil
1
2!

n!! = n!

1
+
3!

( 1)n
+
n!

:

Buktikan bahwa untuk semua n 2 N, n > 3,
n!! = n! (mod (n

1)) :

Bukti. Dipunyai
n!

1
2)! 1
2!
!
n 1
n
( 1)
( 1)
+
+
(n 1)!
n!

n!! = n (n

= (n

1) m + ( 1)n

= (n

n

( 1)n
n 1

1) (m + ( 1)n )

dengan m 2 Z, dan dicatat bahwa (n

4.2

1
+
3!

1) (n

1

n

1

+

2)! dapat dibagi oleh k; k

n

2.

Persamaan Kongruensi

De…nisi 4.21 Bilangan bulat x0 yang memenuhi persamaan (kongruensi) dinamakan
penyelesaian untuk persamaan tersebut.
Pertama kali dipelajari persamaan linear terhadap penjumlahan. Persamaan linear
(terhadap penjumlahan) dalam kongruensi:
a + x = b (mod n)
selalu mempunyai penyelesaian. Kunci dari penyelesaian persamaan tersebut adalah
bilangan c sedemikian sehingga c + a = n.
Contoh 4.22 Cari semua x yang memenuhi persamaan 7 + x = 4 (mod 5).
Penyelesaian. Persamaan diubah menjadi 2 + x = 4 (mod 5) dan
3 + 2 + x = 3 + 4 (mod 5)
5 + x = 7 (mod 5)
x = 2 (mod 5) :
23

Bab 4. Kongruensi Zn
Berikutnya dipelajari persamaan linear terhadap perkalian dalam kongruensi:
ax = b (mod n) :

Kunci dari penyelesaian persamaan tersebut adalah bilangan c sedemikian sehingga
ac = 1 (mod n). Tetapi hal ini tidak selalu terjadi. Sebagi contoh, diambil n = 4
dan a; c 2 f0; 1; 2; 3g. Jika a = 2 (mod 4), maka tidak ada c 2 f0; 1; 2; 3g sehingga
2c = 1 (mod 2).
Contoh 4.23 Periksa apakah persamaan-persamaan berikut ini mempunyai penyelesaian:
a) 2x = 1 (mod 4)
b) 3x = 1 (mod 4)
c) 12x = 8 (mod 15)
Penyelesaian. Diperiksa satu persatu seperti berikut.
a) Jika persamaan tersebut mempunyai penyelesaian, maka penyelesaiannya adalah
2x

1 = 4t,

dengan t adalah suatu bilangan bulat.

Karena ruas kiri adalah bilangan ganjil dan ruas kanan adalah bilangan genap,
maka kesamaan tersebut tidak pernah terjadi. Jadi, persamaan kongruensi tidak
mempunyai penyelesaian.
b) Karena (3; 4) = 1, maka terdapat bilangan bulat p, q sehingga
3p + 4q = 1 atau 3p = 1

4q:

Bilangan p = 3 dan q = 2. Jadi, persamaan di atas mempunyai penyelesaian
x = p + 4r = 3 + 4r dengan r adalah suatu bilangan bulat.
c) Jika persamaan tersebut mempunyai penyelesaian, maka terdapat x yang memenuhi
12x

8 = 15t:

Ruas kanan dapat dibagi 3, maka ruas kiri harus dapat dibagi 3. Suku 12x
habis dibagi 3, tetapi 8 tidak habis dibagi 3. Jadi persamaan tidak mempunyai
penyelesaian.

Contoh 4.24 Selesaikan setiap kongruensi di bawah ini.
a) 5x = 7 (mod 12).
b) 3x = 6 (mod 101) :
c) 2x = 8 (mod 10) :
Penyelesaian. Diselesaikan satu persatu seperti berikut.
24

Bab 4. Kongruensi Zn
a) Dicatat bahwa 5 5 = 25 = 1 (mod 12). Karena itu
5 5x = 5 7 (mod 12)
x = 35 (mod 12) = 11:
b) Dicatat 34 3 = 102 = 1 (mod 101). Karena itu
34 3x = 34 6 (mod 101)
x = 204 (mod 101) = 2:

c) Di sini (2; 10) = 2, sehingga metode seperti di atas tidak bisa diaplikasikan. Berdasarkan
de…nisi kongruensi dan keterbagian, diminta 2x 8 = k 10 = 0 (mod 10) untuk
suatu k 2 Z. Persamaan dapat dituliskan menjadi
2 (x

4) = 0 (mod 10) = 10

dan memberikan persamaan
x

4=0

atau x

4 = 5:

Karena itu diperoleh penyelesaian untuk persamaan kongruensi yaitu x = 4 atau
x = 9.

Latihan 4.25 Cari penyelesaian untuk persamaan berikut ini.
a) 5x = 4 (mod 11) :
b) 3x = 7 (mod 17) :
c) 9x = 4 (mod 49) :
d) 100x = 7 mod 112 :
Latihan 4.26 Jika ada, cari penyelesaian untuk persamaan berikut ini.
a) 4x = 5 (mod 6) :
b) 6x = 2 (mod 8) :
c) 14x = 12 (mod 21) :
d) 8x = 4 (mod 12) :
Latihan 4.27 Untuk a = 1; 2; :::; 6, cari semua penyelesaian untuk persamaan ax =
1 (mod 7).
Latihan 4.28 Cari semua bilangan bulat a dimana 1

a

5 sehingga ax = 1 (mod 6).

Latihan 4.29 Diberikan bilangan-bilangan bulat a dan b. Jika 0 < a < 7 dan 0 < b <
7, tentukan a dan b sedemikian sehingga ab = 0 (mod 15).
25

Bab 4. Kongruensi Zn

4.3

Uji Keterbagian

Berikut ini diberikan suatu contoh aturan keterbagian yang sangat terkenal.
Teorema 4.30 (Casting-out 9’ Suatu bilangan asli n dapat dibagi oleh 9 jika dan
s)
hanya jika jumlahan dari digit-digitnya dapat dibagi oleh 9.
Bukti. Diambil n = ak 10k + ak 1 10k 1 +
+ a1 10 + a0 sebagai ekspansi basisj = 1 (mod 9). Karena itu diperoleh
10 dari n. Untuk 10 = 1 (mod 9), dipunyai 10
n = ak + ak 1 +
+ a1 + a0 .
Contoh 4.31 (AHSME 1992) Bilangan bulat dua digit dari 19 sampai 92 dituliskan
secara berturutan untuk membentuk bilangan bulat
192021222324:::89909192:

(4.4)

Berapakah pangkat terbesar dari 3 yang membagi bilangan tersebut?
Penyelesaian. Dengan penggunaan aturan casting-out 9, bilangan (4.4) dapat dibagi
oleh 9 jika dan hanya jika
19 + 20 +

+ 91 + 92 = 372 3

Oleh karena itu, bilangan (4.4) dapat dibagi oleh 3 tetapi tidak oleh 9.
Contoh 4.32 (IMO 1975) Ketika 44444444 dituliskan dalam notasi desimal, jumlahan dari digit-digitnya adalah A. Diambil B sebagai jumlahan dari digit-digit pada
bilangan A. Tentukan jumlahan dari digit-digit pada bilangan B. (A dan B dituliskan
dalam notasi desimal)
Penyelesaian. Dipunyai 4444 = 7 (mod 9), karena itu 44443 = 73 = 1 (mod 9). Jadi
44444444 = 44443(1481) 4444 = 1 7 = 7 (mod 9). Diambil C sebagai jumlahan dari
digit-digit pada bilangan B.
Berdasarkan aturan casting-out 9, 7 = 44444444 = A = B = C (mod 9). Sekarang,
4444 log (4444) < 4444 log 104 = 17776. Ini berarti bahwa 44444444 mempunyai paling
banyak 17776 digit, sehingga jumlahan dari digit-digit pada 44444444 adalah paling
besar 9 17776 = 159984, yang berarti A
159984. Di antara semua bilangan asli
159984, bilangan yang mempunyai jumlahan digit terbesar adalah 99999, sehingga
diperoleh B 45. Dari semua bilangan asli 45, bilangan yang mempunyai jumlahan
digit terbesar adalah 39. Jadi jumlahan dari digit-digit B adalah paling besar 12.
Tetapi karena C = 7 (mod 9), maka diperoleh C = 7. Jadi, jumlahan dari digit-digit
pada bilangan B adalah 7.

4.4

Sisa lengkap

De…nisi 4.33 Suatu himpunan x1 ; x2 ; :::; xn dinamakan sistem sisa lengkap (complete
residue system) modulo n jika untuk setiap bilangan bulat y terdapat secara tepat satu
indeks j sedemikian sehingga y = xj (mod n).
26

Bab 4. Kongruensi Zn

Dalam hal ini jelas bahwa untuk sembarang himpunan berhingga A dari bilanganbilangan bulat, himpunan A akan membentuk himpunan sisa lengkap modulo n jika
dan hanya jika himpunan A mempunyai n anggota dan setiap anggota dari himpunan
tidak saling kongruen modulo n. Sebagai contoh, himpunan A = f0; 1; 2; 3; 4; 5g membentuk suatu himpunan sisa lengkap modulo 6, karena setiap bilangan bulat x kongruen
dengan satu dan hanya satu anggota dari A. Himpunan B = f 3; 2; 1; 1; 2; 3g tidak
membentuk himpunan sisa lengkap modulo 6 karena 3 = 3 (mod 6).
Sekarang diperhatikan himpunan Zn = f0; 1; 2; :::; n 1g. Sebagai contoh, diambil
n = 3 sehingga dipunyai Z3 = f0; 1; 2g. Elemen 0 menyatakan semua semua bilangan
bulat yang dapat dibagi oleh 3, sedangkan 1 dan 2 berturut-turut menyatakan semua
bilangan bulat yang mempunyai sisa 1 dan 2 ketika dibagi oleh 3. Dide…nisikan jumlahan pada Z3 seperti berikut ini. Diberikan a; b 2 Z3 , maka terdapat c 2 Z3 sedemikian
sehingga a +3 b = c (mod 3). Tabel 4.1 memuat semua penjumlahan yang mungkin.
Tabel 4.1: Tabel penjumlahan untuk Z3 .

+3
0
1
2

0
0
1
2

1
1
2
0

2
2
0
1

Diamati bahwa Z3 bersama-sama dengan operasi +3 seperti yang diberikan dalam Tabel
4.1 memenuhi sifat-sifat:
1. Elemen 0 2 Zn merupakan suatu elemen identitas untuk Z3 , yaitu 0 memenuhi
0 +3 a = a +3 0 = a untuk semua a 2 Z3 .
2. Setiap elemen a 2 Z3 mempunyai suatu invers penjumlahan b, yaitu suatu elemen
sedemikan sehingga a +3 b = b +3 a = 0. Invers penjumlahan dari a dinotasikan
dengan a. Dicatat bahwa di Z3 dipunyai 0 = 0, 1 = 2, dan 2 = 1.
3. Operasi penjumlahan di Z3 adalah asosiatif, yaitu untuk setiap a; b; c 2 Z3 berlaku
a +3 (b +3 c) = (a +3 b) +3 c.
Selanjutnya dikatakan bahwa (Z3 ; +3 ) membentuk suatu grup (group) dan dinamakan
grup dari sisa dibawah penjumlahan modulo 3.
Secara serupa, dide…nisikan (Zn ; +n ) sebagai grup dari sisa dibawah penjumlahan modulo n.
Latihan 4.34 Konstruksikan tabel penjumlahan untuk Z6 dan Z8 .
Latihan 4.35 Berapa banyak pasangan berurutan (a; b) 6= 0 yang berbeda di Z12 sedemikian sehingga a +12 b = 0?
Bab 5

Faktorisasi Tunggal
5.1

FPB dan KPK

Diberikan a; b 2 Z dan keduanya tidak nol. Bilangan bulat positif terbesar yang membagi a dan b dinamakan faktor persekutuan terbesar (greatest common divisor ) dari a
dan b, dan dinotasikan dengan (a; b). Dicatat bahwa jika d j a dan d j b maka d j (a; b).
Sebagai contoh, (68; 8) = 2, (1998; 1999) = 1.
Jika (a; b) = 1, maka a dan b dikatakan prima relatif (relatively prime) atau koprima
(coprime). Jadi, jika a; b adalah prima relatif, maka keduanya tidak mempunyai faktor
bersama yang lebih besar dari 1.
Jika a; b 2 Z, keduanya tidak nol, bilangan bulat positif terkecil yang merupakan kelipatan dari a dan b dinamakan kelipatan persekutuan terkecil (least common multiple)
dari a dan b, dan dinotasikan dengan [a; b]. Dicatat bahwa jika a j c dan b j c maka
[a; b] j c.
Berikut ini diberikan teorema-teorema yang berkaitan dengan faktor persekutuan terbesar.
Teorema 5.1 (Teorema Bachet-Bezout) Faktor persekutuan terbesar, disingkat FPB,
dari sembarang dua bilangan bulat a dan b dapat dituliskan sebagai kombinasi linier dari
a dan b, yaitu terdapat bilangan bulat x; y dimana
(a; b) = ax + by:
Bukti. Dimisalkan F = fax + by > 0 : x; y 2 Zg. Jelas bahwa satu di antara a,
b berada di F, untuk a dan b yang tak nol. Berdasarkan Prinsip Terurut Baik,
F mempunyai elemen terkecil, misalnya d. Oleh karena terdapat x0 , y0 sedemikian
sehingga d = ax0 + by0 . akan dibuktikan bahwa d = (a; b). Atau dengan kata lain akan
dibuktikan bahwa d j a, d j b dan jika t j a, t j b maka t j d.
Pertama kali akan dibuktikan d j a. Berdasarkan Algoritma Pembagian, dapat dicari
bilangan bulat q, r, dengan 0 r < d sedemikian sehingga a = dq + r. Karena itu
r=a

dq = a

q (ax0 + by0 ) = a (1

qx0 )

by0 :

Jika r > 0, maka r 2 F lebih kecil daripada elemen terkecil d di F, yang kontradiksi
dengan kenyataan bahwa d adalah elemen terkecil di F. Jadi r = 0. Akibatnya dq = a,
yang berarti d j a. Dengan cara serupa dapat dibuktikan bahwa d j b.
27
28

Bab 5. Faktorisasi Tunggal

Berikutnya diandaikan bahwa t j a dan t j b, maka a = tm dan b = tn untuk bilangan
bulat m, n. Karena itu d = ax0 + by0 = t (mx0 + ny0 ), yang berarti t j d.
Di sini jelas bahwa sembarang kombinasi linier dari a dan b dapat dibagi oleh (a; b).
Akibat 5.2 Bilangan bulat positif a dan b adalah prima relatif jika dan hanya jika
terdapat bilangan bulat x dan y sedemikian sehingga ax + by = 1.
Lemma 5.3 (Lemma Euclid) Jika a j bc dan (a; b) = 1, maka a j c.
Bukti. Untuk (a; b) = 1, berdasarkan Teorema Bachet-Bezout, terdapat bilangan
bulat x; y dimana ax + by = 1. Karena a j bc, terdapat suatu bilangan bulat s dimana
as = bc. Selanjutnya c = c 1 = cax + cby = cax + asy, yang berarti a j c.
Teorema 5.4 Jika (a; b) = d, maka
a b
;
d d

= 1:

Bukti. Berdasarkan Teorema Bachet-Bezout, terdapat bilangan bulat x; y dimana
a
b
a b
ax + by = d. Karena itu diperoleh
x+
y = 1 dimana , adalah bilangand
d
d d
a b
;
= 1:
bilangan bulat. Disimpulkan bahwa
d d
Teorema 5.5 Jika c adalah suatu bilangan bulat positif, maka
(ca; cb) = c (a; b) :
Bukti. Diambil d1 = (ca; cb) dan d2 = (a; b). Akan dibuktikan bahwa d1 j cd2 dan cd2 j
d1 . Untuk d2 j a dan d2 j b, maka cd2 j ca dan cd2 j cb. Jadi cd2 merupakan pembagi
persekutuan dari ca dan cb, karena itu d1 j cd2 . Berdasarkan Teorema Bachet-Bezout,
dapat ditemukan bilangan-bilangan bulat x; y dimana d1 = acx + bcy = c (ax + by).
Tetapi karena ax + by merupakan kombinasi linier dari a dan b, maka ini dapat dibagi
oleh d2 . Karena itu terdapat suatu bilangan bulat s sedemikian sehingga sd2 = ax+by.
Ini berarti bahwa d1 = csd2 , artinya cd2 j d1 .
Serupa dengan di atas, berlaku (ca; cb) = jcj (a; b) untuk sembarang bilangan bulat tak
nol c.
Lemma 5.6 Untuk bilangan-bilangan bulat tak nol a, b, c berlaku
(a; bc) = (a; (a; b) c) :
Bukti. Karena (a; (a; b) c) membagi (a; b) c dan (a; b) c membagi bc (menurut Teorema
5.5(a; b) c) maka (a; (a; b) c) membagi bc. Jadi (a; (a; b) c) membagi a dan bc, atau
dituliskan (a; (a; b) c) j (a; bc). Di sisi lain, (a; bc) membagi a dan bc, karena itu (a; bc)
membagi ac dan bc. Oleh karena itu, (a; bc) membagi (ac; bc) = (a; b) c. Jadi (a; bc)
membagi a dan (a; b) c, atau dituliskan (a; bc) j (a; (a; b) c). Disimpulkan (a; bc) =
(a; (a; b) c).
Teorema 5.7 a2 ; b2 = (a; b)2 .
29

Bab 5. Faktorisasi Tunggal

Bukti. Diandaikan bahwa (m; n) = 1. Diaplikasikan lemma sebelumnya dua kali untuk
memperoleh
m2 ; n2 = m2 ; m2 ; n n = m2 ; (n; (m; n) m) n :
Untuk (m; n) = 1, ruas kanan dari pernyataan di atas sama dengan m2 ; n . Diaplikasikan kembali lemma di atas, diperoleh
m2 ; n = (n; (m; n) m) = 1:
Jadi (m; n) = 1 mengakibatkan m2 ; n2 = m2 ; n = 1. Berdasarkan Teorema 5.4,
a
b
;
(a; b) (a; b)
karena itu

a2
b2
;
(a; b)2 (a; b)2

= 1,

= 1.

Berdasarkan Teorema 5.5, pernyataan terakhir dikalikan dengan (a; b)2 untuk memperoleh
a2 ; b2 = (a; b)2 .

Contoh 5.8 Diambil (a; b) = 1. Buktikan bahwa a + b; a2

ab + b2 = 1 atau 3.

Bukti. Dimisalkan d = a + b; a2 ab + b2 . Berdasarkan Teorema Bachet-Bezout,
sembarang kombinasi linier dari a + b dan a2 ab + b2 dapat dibagi oleh d. Karena itu
d membagi
(a + b) (a + b) + ( 1) a2 ab + b2 = 3ab:
Karena itu d membagi a + b dan 3ab, akibatnya d membagi 3b (a + b) + ( 1) 3ab = 3b2
atau dituliskan d j 3b2 . Serupa dengan itu, diperoleh d j 3a2 . Jadi
d j 3a2 ; 3b2 = 3 a2 ; b2 = 3 (a; b)2 = 3:
Disimpulkan bahwa d = 1 atau 3.
Contoh 5.9 (IMO 1959) Buktikan bahwa pecahan

21n + 4
adalah irreducible (tidak
14n + 3

dapat disederhanakan) untuk setiap bilangan asli n.
Bukti. Untuk semua bilangan asli n dipunyai 3 (14n + 3) 2 (21n + 4) = 1. Jadi,
berdasarkan Akibat 5.2, diperoleh bahwa pembilang dan penyebut adalah prima relatif,
atau dengan kata lain tidak mempunyai faktor persekutuan yang lebih besar dari 1.
Contoh 5.10 (AIME 1985) Bilangan-bilangan dalam barisan
101; 104; 109; 116; :::
mempunyai bentuk an = 100+n2 , n = 1; 2; :::. Untuk setiap n, diambil dn = (an ; an+1 ).
Cari maksfdn gn 1 .
30

Bab 5. Faktorisasi Tunggal
Penyelesaian. Diamati bahwa
dn =

100 + n2 ; 100 + (n + 1)2

=
=
Jadi dn j 2 100 + n2

100 + n2 ; 100 + n2 + 2n + 1
100 + n2 ; 2n + 1 :

n (2n + 1) atau dn j (200
dn j (2 (200

n). Oleh karena itu

n) + (2n + 1))

atau dn j 401 untuk semua n. Jadi maksfdn gn

1

= 401.

Contoh 5.11 Buktikan bahwa jika m dan n adalah bilangan-bilangan asli dan m adalah
ganjil, maka (2m 1; 2n + 1) = 1.
Bukti. Dimisalkan d = (2m 1; 2n + 1). Karena 2m 1 dan 2n +1 adalah ganjil, maka
d haruslah suatu bilangan ganjil. Selain itu, dapat dituliskan 2m 1 = kd dan 2n +1 = ld
untuk bilangan-bilangan asli k dan l. Oleh karena itu, 2mn = (kd + 1)n = td+1, dimana
n 1
X n
t=
k n j dn j 1 . Melalui cara yang sama diperoleh 2mn = (ld 1)m = ud 1,
j
j=0

dengan menggunakan kenyataan bahwa m adalah ganjil. Untuk td + 1 = ud
dapat dituliskan (u t) d = 2, haruslah d j 2. Akibatnya d = 1.
Contoh 5.12 Berapa banyak bilangan bulat positif
1260?

1 atau

1260 yang prima relatif terhadap

Penyelesaian. Karena 1260 = 22 32 5 7, sekarang masalahnya adalah mencari
bilangan-bilangan yang lebih kecil dari 1260 dan tidak dapat dibagi oleh 2, 3, 5, atau 7.
Diambil A menyatakan himpunan dari bilangan-bilangan bulat 1260 dan merupakan
kelipatan dari 2, B adalah himpunan kelipatan dari 3, dan seterusnya. Berdasarkan
Prinsip Inklusi-Eksklusi,
jA [ B [ C [ Dj = jAj + jBj + jCj + jDj
jA  Bj

jB  Cj

jA  Cj

jA  Dj

jB  Dj

jC  Dj

+ jA  B  Cj + jA  B  Dj + jA  C  Dj + jB  C  Dj
jA  B  C  Dj

= 630 + 420 + 252 + 180
+42 + 30 + 18 + 12

210

126

90

84

60

36

6

= 972:
Jadi, banyaknya bilangan bulat positif
1260 972 = 288.

5.2

1260 yang prima relatif terhadap 1260 adalah

Bilangan Prima dan Faktorisasi

Diingat kembali de…nisi suatu bilangan prima, yaitu suatu bilangan bulat positif lebih
besar dari 1 yang hanya mempunyai pembagi positif 1 dan dirinya sendiri. Jelas bahwa
Bab 5. Faktorisasi Tunggal

31

hanya 2 yang merupakan bilangan prima genap, dan juga hanya 2 dan 3 yang merupakan bilangan-bilangan prima yang berturutan. Suatu bilangan, selain 1, yang tidak
prima dinamakan bilangan composite. Jelas bahwa jika n > 1 adalah composite maka
n dapat dituliskan sebagai n = ab, dimana 1 < a b < n dan a; b 2 N.
Contoh 5.13 Tentukan semua bilangan bulat positif n untuk yang mana 3n 4, 4n 5,
dan 5n 3 adalah bilangan-bilangan prima.
Penyelesaian. Jumlah dari ketiga bilangan tersebut adalah 12n 12, yang jelas
merupakan suatu bilangan genap, maka paling sedikit satu diantaranya adalah bilangan
genap. Dipunyai bahwa bilangan prima genap hanyalah 2. Diamati bahwa 4n 5 tidak
mungkin menjadi bilangan genap karena 4n selalu genap untuk setiap n, sehingga jika
dikurangi suatu bilangan ganjil maka hasilnya ganjil. Tetapi 3n 4 dan 5n 3 adalah
mungkin untuk menjadi bilangan genap. Karena itu diselesaikan persamaan 3n 4 = 2
dan 5n 3 = 2 yang secara berturutan menghasilkan n = 2 dan n = 1. Secara mudah
bisa diperiksa bahwa n = 2 akan membuat ketiga bilangan tersebut adalah prima.
Contoh 5.14 (AHSME 1976) Jika p dan q adalah prima, dan x2
mempunyai dua akar bulat positif berbeda, tentukan p dan q.

px + q = 0

Penyelesaian. Diambil x1 dan x2 , dengan x1 < x2 , sebagai dua akar bulat positif yang
berbeda. Karena itu bisa dituliskan x2 px+q = (x x1 ) (x x2 ), yang mengakibatkan
p = x1 + x2 dan q = x1 x2 . Karena q adalah prima, maka x1 = 1. Jadi, q = x2 dan
p = x2 + 1, yang berarti p dan q adalah dua bilangan prima yang berurutan, yaitu
q = 2 dan p = 3.
Teorema 5.15 Jika n > 1, maka n dapat dibagi oleh paling sedikit satu bilangan prima.
Bukti. Karena n > 1, maka dipunyai paling sedikit satu pembagi > 1. Berdasarkan
Prinsip Terurut Baik, n pasti mempunyai paling sedikit satu pembagi positif yang lebih
besar dari 1, misalnya q. Diklaim bahwa q adalah prima. Jika q bukan prima maka
dapat dituliskan q = ab, 1 < a b < q. Ini berarti bahwa a adalah suatu pembagi dari
n yang lebih besar dari 1 dan lebih kecil dari q. Timbul kontradiksi dengan kenyataan
bahwa q adalah minimal.
Teorema 5.16 (Euclid) Terdapat tak hingga banyak bilangan prima.
Bukti. Diandaikan terdapat berhingga banyak bilangan prima, misalnya p1 , p2 , ...,
pn . Diambil
N = p1 p2
pn + 1
Bilangan bulat N adalah lebih besar dari 1, sehingga berdasarkan teorema sebelumnya diperoleh bahwa N pasti mempunyai suatu pembagi prima p. Bilangan prima p
haruslah salah satu dari bilangan-bilangan p1 , p2 , ..., pn . Tetapi, diamati bahwa p pasti
berbeda dari sembarang p1 , p2 , ..., pn karena N mempunyai sisa 1 ketika dibagi oleh
sembarang pi . Jadi timbul kontradiksi.
Teorema 5.17 Jika bilangan bulat positif n adalah composite, maka n pasti mempunp
yai suatu faktor prima p dengan p
n.
32

Bab 5. Faktorisasi Tunggal

p
Bukti. Diandaikan bahwa n = ab, dimana 1 < a
b < n. Jika a; b > n, maka
p p
n = ab > n n = n, yang adalah kontradiksi. Jadi n mempunyai suatu faktor 6= 1
p
p
dan
n. Karena itu, faktor prima dari n adalah
n.
Contoh 5.18 Berapa banyak bilangan prima

100?

p
Penyelesaian. Diamati bahwa 100 = 10. Berdasarkan teorema sebelumnya, semua
bilangan composite dalam range 10 n 100 mempunyai suatu faktor prima diantara
2, 3, 5, atau 7. Dimisalkan Am adalah himpunan bilangan-bilangan bulat positif yang
merupakan kelipatan dari m dan
100. Diperoleh jA2 j = 50, jA3 j = 33, jA5 j = 20,
jA7 j = 14, jA6 j = 16, jA10 j = 10, jA14 j = 7, jA15 j = 6, jA21 j = 4, jA35 j = 2, jA30 j = 3,
jA42 j = 2, jA70 j = 1, jA105 j = 0, jA210 j = 0. Jadi, banyaknya bilangan prima
100
adalah
= 100

(banyak bilangan composite

100)

1

= 100 + 4

(kelipatan dari 2; 3; 5; atau 7

= 100 + 4

(50 + 33 + 20 + 14) + (16 + 10 + 7 + 6 + 4 + 2)

(3 + 2 + 1 + 0)

0

100)

1

1

= 25
dengan mengingat bahwa 1 bukanlah prima atau composite.
Sekarang diperhatikan bilangan bulat 1332. Jelas bahwa bilangan tersebut dapat dibagi
oleh 2, sehingga diperoleh 1332 = 2 666. Selanjutnya, 666 dapat dibagi oleh 6, sehingga
1332 = 2 2 3 111. Terakhir, 111 dapat dibagi oleh 3, sehingga diperoleh 1332 =
2 2 3 3 37. Karena 2, 3, 37 adalah bilangan-bilangan prima maka proses faktorisasi
dari 1332 berhenti. Selanjutnya faktorisasi dari 1332 dapat dituliskan seperti 22 32 37.
Faktorisasi demikian dinamakan faktorisasi kanonis (canonical factorisation).
Teorema 5.19 (Teorema Fundamental Aritmatika) Setiap bilangan asli n
mempunyai suatu faktorisasi tunggal dalam bentuk
n = pa 1 pa 2
1 2

1

pa s
s

dimana pi adalah prima berbeda dan ai adalah bilangan bulat positif.
Bukti. Diasumsikan bahwa
n = pa 1 p a 2
1 2

b b
pas = q11 q22
s

b
qt t

merupakan dua faktorisasi kanonis dari n. Berdasarkan Lemma Euclid, disimpulkan
bahwa setiap p pasti membagi suatu q dan setiap q membagi suatu p. Akibatnya s = t.
Selanjutnya, dari p1 < p2 <
< ps dan q1 < q2 <
< qt disimpulkan bahwa pi = qi ,
1 i s.
Jika ai > bi untuk suatu i, atas pembagian oleh pai , diperoleh
i
p a 1 pa 2
1 2

pa i
i

bi

pas = pb1 pb2
s
1 2

b

b

i+1
pi i 11 pi+1

pbs ;
s

yang adalah tidak mungkin, karena ruas kiri dapat dibagi oleh pi dan ruas kanan tidak
dapat dibagi oleh pi . Serupa dengan itu, diperoleh hasil yang sama untuk ai < bi . Jadi
haruslah ai = bi untuk semua i.
33

Bab 5. Faktorisasi Tunggal
Contoh 5.20 Diambil p adalah suatu bilangan prima. Buktikan bahwa
bilangan rasional.

p

p bukan suatu

a
p
p
Bukti. Diasumsikan bahwa p adalah rasional, artinya p =
dimana a dan b
b
bilangan-bilangan asli yang prima relatif sebab faktor-faktor persekutuannya dapat
dihapus. Karena itu bisa dituliskan pb2 = a2 . Jadi p j a2 dan juga p j a. Dituliskan a =
a1 p untuk suatu bilangan bulat a1 , maka dipunyai pb2 = a2 p2 , sehingga b2 = a2 p yang
1
1
berarti p j b. Jadi, p adalah faktor persekutuan dari a dan b, sehingga ini kontradiksi
p
dengan asumsi. Jadi, p bukan suatu bilangan rasional.
Contoh 5.21 Buktikan bahwa terdapat tepat satu bilangan asli n dimana 28 + 211 + 2n
adalah pangkat dua sempurna.
Bukti. Jika k 2 = 28 +211 +2n = 2304+2n = 482 +2n , maka k 2 482 = (k 48) (k + 48) =
2n . Berdasarkan faktorisasi tunggal diperoleh k 48 = 2s , k + 48 = 2t , s + t = n. Dari
sini diperoleh 2t 2s = 96 = 3 25 atau 2s 2t s 1 = 3 25 . Berdasarkan faktorisasi
tunggal, dari kesamaan terakhir diperoleh s = 5, t s = 2. Jadi, s = 5 dan t = 7,
sehingga n = s + t = 12.
Untuk suatu bilangan prima p, pk dikatakan membagi penuh (fully divide) n dan dituliskan pk k n jika k adalah bilangan bulat positif terbesar sedemikian sehingga pk j n.
Contoh 5.22 (ARML 2003) Tentukan pembagi terbesar dari 1001001001 yang tidak
melebihi 1000.
Penyelesaian. Dipunyai
1001001001 = 1001 106 + 1001 = 1001
= 7 11 13

106 + 1

106 + 1 :

3

Berdasarkan rumus x6 + 1 = x2 + 1 = x2 + 1 x4 x2 + 1 , dituliskan 106 + 1 =
101 9901. Karena itu dapat dituliskan 1001001001 = 7 11 13 101 9901. Dari sini
tidaklah sulit untuk memeriksa bahwa tidak ada kombinasi dari 7, 11, 13, dan 101 yang
dapat menghasilkan suatu hasil kali yang lebih besar dari 9901 tetapi kurang dari 1000.
Jadi jawabannya adalah 9901.
n

Contoh 5.23 Diambil n adalah suatu bilangan bulat positif. Buktikan bahwa 32 + 1
dapat dibagi oleh 2, tetapi tidak dapat dibagi oleh 4.
n

n

Bukti. Jelas bahwa 32 adalah ganjil dan 32 + 1 adalah genap. Dicatat bahwa
n 1
n
n 1
2n 1
32 = 32
= 92
= (8 + 1)2 . Dipunyai rumus binomial
(x + y)m = xm +

m m
x
1

1

y+

m m
x
2

2 2

y +

+

m
xy m
m 1

1

+ ym:

Diambil x = 8, y = 1, dan m = 2n 1 dalam persamaan di atas, maka pada ruas kanan
dapat dilihat bahwa setiap bagian jumlahan merupakan kelipatan dari 8 kecuali yang
n
terakhir (yaitu y m = 1). Karena itu sisa dari 32 ketika dibagi oleh 4 adalah sama
n
dengan 1, dan sisa dari 32 + 1 ketika dibagi oleh 4 adalah sama dengan 2.
Contoh 5.24 Tentukan n sedemikian sehingga 2n k 31024

1 .
34

Bab 5. Faktorisasi Tunggal
Penyelesaian. Dicatat bahwa 210 = 1024 dan x2
32

10

1 =
=
=

32

9

2

y 2 = (x + y) (x

y). Karena itu

12

9

32 + 1

32
9

9

= 32 + 1

9

1 = 32 + 1
8

32 + 1

8

32 + 1

7

32 + 1

32
1

8

32 + 1

1
0

32 + 1 (3

1) :

k

Berdasarkan contoh sebelumnya, 2 j 32 + 1 untuk bilangan bulat positif k. Karena
itu, dari persamaan terakhir dapat dilihat bahwa setiap faktor dapat dibagi oleh 2.
10
Karena banyaknya faktor adalah 11 dan juga 32
1 dapat dibagi oleh 20 , maka
n = 11 + 1 = 12.

5.3

Teorema Fermat dan Teorema Euler

Untuk sembarang bilangan bulat positif m dinotasikan ' (m) sebagai banyaknya bilangan bulat positif yang kurang dari m dan prima relatif terhadap m. Fungsi '
dinamakan fungsi totient Euler. Ini jelas bahwa ' (1) = 1 dan ' (p) = p 1 untuk
sembarang bilangan prima p. Selain itu, jika n adalah suatu bilangan bulat positif
sedemikian sehingga ' (n) = n 1, maka n adalah prima. Selain itu ' pk = pk pk 1
untuk semua bilangan bulat positif k, karena terdapat pk 1 bilangan bulat x yang
memenuhi 0
x < pk yang dapat dibagi oleh p, dan bilangan-bilangan bulat yang
prima relatif terhadap pk tidak dapat dibagi oleh p.
Proposisi 5.25 Diambil m adalah suatu bilangan bulat positif dan a adalah bilangan
bulat yang prima relatif terhadap m. Diandaikan bahwa S adalah sistem sisa lengkap
modulo m. Himpunan
T = aS = fas : s 2 Sg
juga merupakan sistem lengkap modulo m.
Proposisi di atas menyediakan dua teorema yang sangat terkenal dalam teori bilangan.
Teorema 5.26 (Teorema Euler) Diambil a dan m adalah bilangan-bilangan bulat
positif prima relatif, maka a'(m) = 1 (mod m) :
Bukti. Diperhatikan himpunan S = a1 ; a2 ; :::; a'(m) yang terdiri dari semua bilangan bulat positif yang kurang dari m dan prima relatif terhadap m. Karena (a; m) = 1,
dari proposisi sebelumnya diperoleh bahwa
aS = aa1 ; aa2 ; :::; aa'(m)
merupakan sistem sisa lengkap modulo m yang lain, maka
(aa1 ) (aa2 ) ::: aa'(m) = a1 a2 :::a'(m)

(mod m):

Dengan penggunaan (ak ; m) = 1, k = 1; 2; :::; ' (m) diperoleh hasil yang diinginkan.
Teorema 5.27 (Teorema Fermat) Diambil p adalah suatu bilangan prima, maka
ap = a (mod p) untuk semua bilangan bulat a. Selain itu jika a adalah prima relatif
terhadap p, maka ap 1 = 1(mod p)
35

Bab 5. Faktorisasi Tunggal
Contoh 5.28 Diambil p adalah bilangan prima. Buktikan bahwa p membagi abp
untuk semua bilangan bulat a dan b.

bap

Bukti. Dicatat bahwa abp bap = ab bp 1 ap 1 .
Jika p j ab, maka p j (abp bap ); jika p - ab, maka (p; a) = (p; b) = 1, sehingga
bp 1 = ap 1 = 1(mod p), berdasarkan Teorema kecil Fermat. Karena p j bp 1 ap 1 ,
akibatnya p j (abp bap ). Oleh karena itu p j (abp bap ) untuk semua p.
Contoh 5.29 Diambil suatu bilangan prima p

7. Buktikan bahwa bilangan prima

11:::1
| {z }
p 1

dapat dibagi p.
Bukti. Dipunyai

11:::1 =
| {z }

10p

1

1

9

p 1

;

dan kesimpulan diperoleh dari Teorema Fermat. (Dicatat juga bahwa (10; p) = 1).
Contoh 5.30 Diambil suatu bilangan prima p

5. Buktikan bahwa p8 = 1(mod 240).

Bukti. Dicatat bahwa 240 = 24 3 5. Berdasarkan Teorema Fermat, p2 = 1(mod 3)
dan p4 = 1(mod 5). Karena suatu bilangan bulat positif adalah prima relatif terhadap
24 jika hanya jika bilangan bulat positif adalah ganjil, maka ' 24 = 23 . Berdasarkan
Teorema Euler, dipunyai 28 = 1(mod 16). Oleh karena itu p8 = 1(mod m) untuk m = 3,
5, dan 6, yang berakibat p8 = 1(mod 240).
Contoh 5.31 (IMO 2005) Diperhatikan barisan a1 ; a2 ; ::: yang dide…nisikan oleh
an = 2n + 3n + 6n

1

untuk semua bilangan bulat positif n. Tentukan semua bilangan bulat positif yang prima
relatif terhadap setiap suku dari barisan.
Penyelesaian. Cukup ditunjukkan bahwa setiap bilangan prima p membagi an untuk
suatu bilangan bulat positif n. Dicatat bahwa p = 2 dan p = 3 membagi a2 = 48.
Diandaikan p
5. Berdasarkan Teorema Fermat, dipunyai 2p 1 = 3p 1 = 6p 1 =
1 (mod p), maka
3 2p

1

+ 2 3p

1

+ 6p

1

= 3 + 2 + 1 = 6(mod 6);

atau 6 2p 2 + 3p 2 + 6p 2 1 = 0(mod p), artinya 6ap 2 dapat dibagi oleh p. Karena
p adalah prima relatif terhadap 6, ap 2 dapat dibagi oleh p. Karena itu jawabannya
adalah 1.
Bab 6

Algoritma Euclid
Sekarang akan diperiksa suatu prosedur yang menghindari pemfaktoran dua bilangan
bulat positif untuk memperoleh faktor persekutuan terbesar. Ini dinamakan Algoritma
Euclid dan digambarkan seperti berikut ini. Diambil a, b adalah bilangan-bilangan bulat tak nol. Setelah Algoritma Pembagian diaplikasikan secara berulang-ulang, diperoleh barisan kesamaan, dengan r0 = a dan r1 = b,
r0 = q 1 r1 + r2 ;
r1 = q 2 r2 + r3 ;
r2 = q 3 r3 + r4 ;
.
.
.
rn
rn
rn

3
2
1

= q n 2 rn
= q n 1 rn
= q n rn :

0 < r2 < jr1 j ;
0 < r3 < r2 ;
0 < r4 < r3 ;
.
.
.
+ rn 1 ;
+ rn ;
1

0 < rn 1 < rn 2 ;
0 < rn < rn 1 ;

2

Dicatat bahwa
0

rn < rn

1

<

< r3 < r2 < b;

karena itu pada akhirnya dicapai rn+1 yang sama dengan nol. Selain itu, diperhatikan
bahwa barisan kesamaan memperbolehkan setiap rk , k = 2; :::; n dinyatakan dalam
suku-suku dari rk 2 dan rk 1 . Sebagai contoh, dipunyai
rn

1

= rn

2

qn

1 rn 1 :

Digunakan pernyataan tersebut secara berulang, maka dapat dituliskan
rn = ur0 + vr1 = ua + vb:
Jadi rn dapat dinyatakan sebagai suatu kombinasi linier dari a dan b. Berdasarkan
Teorema Bachet-Bezout, disimpulkan bahwa rn adalah FPB dari a dan b. Jadi, suku
sisa tak nol terakhir rn yang dihasilkan oleh algoritma Euclid adalah (a; b).
Selanjutnya, FPB dari dua bilangan bulat boleh dinyatakan sebagai suatu kombinasi
linier dari dua bilangan bulat tersebut dengan menggunakan metode substitusi balik.
Contoh 6.1 Tentukan (84; 60), dan selanjutnya nyatakan sebagai suatu kombinasi linier dari kedua bilangan bulat tersebut.

36
37

Bab 6. Algoritma Euclid
Penyelesaian. Diambil a = 84 dan b = 60, maka
84 = 1 60 + 24;
60 = 2 24 + 12;
24 = 1 12;

24 = 84 + ( 1) 60;
12 = 60 + ( 2) 24;
12 = (84; 60) :

Selanjutnya dikerjakan secara mundur untuk mendapatkan
12 = 60 + ( 2) 24
= 60 + ( 2) (84 + 60 ( 1))
= ( 2) 84 + 3 60.
Jadi,
(84; 60) = 12 = ( 2) 84 + 3 60.

Contoh 6.2 Tentukan (190; 72), dan selanjutnya nyatakan sebagai suatu kombinasi
linier dari kedua bilangan bulat tersebut.
Penyelesaian. Diambil a = 190 dan b =

72, maka

190 = ( 2) ( 72) + 46;
72 = ( 2) 46 + 20;
46 = 2 20 + 6;
20 = 3 6 + 2;
6 = 3 2;

46 = 190 + 2 ( 72) ;
20 = 72 + 2 46;
6 = 46 + ( 2) 20;
2 = 20 + ( 3) 6;
2 = (190; 72) :

Selanjutnya dikerjakan secara mundur untuk mendapatkan
2 = 20 + ( 3) 6
= 20 + ( 3) (46 + ( 2) 20)
= ( 3) 46 + 7 20
= ( 3) 4 + 7 ( 72 + 2 46)
= 7 ( 72) + 11 46
= 7 ( 72) + 11 (190 + 2 ( 72))
= 11 190 + 29 ( 72) :
Jadi, (190; 72) = 2 = 11 190 + 29 ( 72).
Ini juga dapat dilakukan dengan menggunakan kenyataan bahwa (190; 72) = (190; 72)
dan dikerjakan seperti berikut ini.
Contoh 6.3 Tentukan (190; 72), dan selanjutnya nyatakan sebagai suatu kombinasi
linier dari kedua bilangan bulat tersebut.
38

Bab 6. Algoritma Euclid
Penyelesaian. Diambil a = 190 dan b = 72, maka
190 = 2 72 + 46;
72 = 1 46 + 26;
46 = 1 26 + 20;
26 = 1 20 + 6
20 = 3 6 + 2;
6 = 3 2;

46 = 190 + ( 2) 72;
26 = 72 + ( 1) 46;
20 = 46 + ( 1) 26;
6 = 26 + ( 1) 20
2 = 20 + ( 3) 6;
2 = (190; 72) :

Selanjutnya dikerjakan secara mundur untuk mendapatkan
2 = 20 + ( 3) 6
= 20 + ( 3) (26 + ( 1) 20)
= ( 3) 26 + 4 20
= ( 3) 26 + 4 (46 + ( 1) 26)
= 4 46 + ( 7) 26
= 4 46 + ( 7) (72 + ( 1) 46)
= ( 7) 72 + 11 46
= ( 7) 72 + 11 (190 + ( 2) 72)
= 11 190 + 29 ( 72) :
Jadi, (190; 72) = 2 = 11 190 + 29 ( 72).
Dicatat bahwa jika (a; b) = ua + vb, maka nilai-nilai u; v tidak tunggal. Sebagai contoh,
83 190 + 219 ( 72) = 2:
Secara umum, bilangan-bilangan u; v dapat dimodi…kasi menjadi u + tb dan v
karena
(u + tb) a + (v ta) b = (ua + vb) + (tba tab) = ua + vb:

ta

Jadi, pendekatan-pendekatan berbeda untuk menentukan kombinasi linier dari (a; b)
dapat menghasilkan jawaban-jawaban berbeda.
Suatu persamaan yang meminta penyelesaian-penyelesaian bilangan bulat dinamakan
persamaan diophantine. Berdasarkan Teorema Bachet-Bezout, diperhatikan bahwa
persamaan diophantine linier
ax + by = c
(6.1)
mempunyai suatu penyelesaian bilangan bulat jika dan hanya jika (a; b) j c. Algoritma
Euclid merupakan suatu cara yang e…sien untuk mencari suatu penyelesaian bagi persamaan (6.1). Sebagai contoh, dari masalah sebelumnya, penyelesaian bilangan bulat
untuk persamaan diophantine
190x + 72y = 2
adalah x = 11, y =

29.

Contoh 6.4 Tunjukkan bahwa persamaan diophantine 206x + 446y = 40 mempunyai
penyelesaian-penyelesaian bulat. Cari penyelesaian (x; y) sedemikian sehingga x + y
mengambil nilai positif terkecil.
39

Bab 6. Algoritma Euclid
Penyelesaian. Diaplikasikan Algoritma Euclid:
446 = 2:206 + 34 =) 206 = 6:34 + 2 =) 34 = 2:17:

Karena (206; 446) = 2 dan 2 j 40, maka terdapat penyelesaian-penyelesaian bilangan
bulat.
Selanjutnya disubstitusi balik untuk memperoleh
2 = 206

6 34 = 206

6 (446

2 206) = 13:206

6:446:

Sekarang, karena 40 = 20 2, maka dapat dituliskan
40 = 20 (13 206

6 446) = 260 206

Jadi, penyelesaiannya adalah x = 260 dan y =

120 446:

120. Penyelesaian umumnya adalah

446
t = 260 223t;
2
206
t = 120 + 103t;
120 +
2

x = 260
y =

untuk suatu t 2 Z. Karena itu, x + y = 140 120t, dan t 2 Z, sehingga x + y bernilai
positif terkecil untuk t = 1, bernilai negatif untuk t 2, dan x + y > 140 untuk t 0.
Jadi, penyelesaian yang diminta yaitu x = 37 dan y = 17.
Contoh 6.5 Cari semua penyelesaian bulat x dimana 0
x < 9 dari kongruensi
linear 6x = 15 (mod 9), atau, jika tidak ada penyelesaian, berikan alasan kenapa tidak
ada penyelesaian.
Penyelesaian. Kita harus menyelesaikan persamaan diophantine 6x + 9y = 15 atau
ekivalen dengan 2x + 3y = 5. Karena 2 ( 1) + 3 (1) = 1, maka 2 ( 5) + 3 (5) = 5.
Oleh karena itu, suatu penyelesaiannya adalah x = 5. Karena (6; 9) = 3, semua
penyelesaiannya mempunyai bentuk
x=

5

9
t=
3

5

3t

untuk suatu t 2 Z. Terdapat tiga penyelesaian mod 9, yaitu
3 ( 3) = 4, dan 5 3 ( 4) = 7.

5

3 ( 2) = 1,

5

Contoh 6.6 Vian ingin membeli beberapa prangko klas kedua dengan harga $20 per
prangko, dan beberapa prangko klas pertama dengan harga $26 per prangko. Uang yang
saya miliki 264. Berapa banyak prangko yang dapat dibeli oleh Vian?
Penyelesaian. Dimisalkan x adalah banyaknya prangko klas kedua, dan y adalah
banyaknya prangko klas pertama. Selanjutnya x; y 2 Z, dengan 20x + 26y = 264, dan
x; y 0.
Sekarang dicari suatu penyelesaian bilangan bulat dari persamaan (menggunakan Algoritma Euclid)
26 = 1 20 + 6 =) 6 = 3 2 + 0:
Selanjutnya disubstitusi balik untuk memperoleh
2 = 20

3 6 = 20

3 (26

20) = 4 20

3 26:
40

Bab 6. Algoritma Euclid
Jadi, 2 = 4 20

3 6. Sekarang 264 = 132 2, sehingga
264 = 132 (4 20

3 26) = 528 20

396 26:

Dari sini, penyelesaian umumnya adalah
26
t dan
2

x = 528 +

y=

396

20
t
2

untuk suatu t 2 Z. Dengan kata lain, x = 528 + 13t dan y = 396 10t untuk suatu
t 2 Z.
Diminta juga bahwa x dan y adalah tak negatif. x 0 ekivalen dengan 528 + 13t 0,
528
8
yaitu t
(40 + 3 ). Untuk t 2 Z, harus dipunyai t
40. y
0 ekivalen
13 =
6
396
(39 + 10 ). Untuk t 2 Z, harus dipunyai
dengan 396 10t 0, yaitu t
10 =
t
40. Penyelesaian persekutuannya yaitu t = 40. Ini memberikan hasil:
x = 528 + ( 40:13) = 8;

dan

y=

396

( 40) 10 = 4:

Dengan kata lain, Vian membeli 8 prangko klas kedua dan 4 prangko klas pertama.

Contoh 6.7 Tentukan suatu penyelesaian bulat untuk persamaan
91x + 126y + 294z = 21:
Penyelesaian. Dimulai dengan mencari (126; 294). Diaplikasikan Algoritma Euclid:
294 = 2 126 + 42 =) 126 = 3:42 + 0:
Jadi, (126; 294) = 42. Selanjutnya disubstitusi balik untuk memperoleh
42 = 1:294

2:126:

(6.2)

Ini berarti bahwa (91; 126; 294) = (91; (126; 294)) = (91; 42). Diaplikasikan Algoritma
Euclid:
91 = 2:42 + 7 =) 42 = 6:7 + 0;
dan disubstitusi balik untuk memperoleh
7 = 1:91

2:42:

(6.3)

Sekarang dicari suatu penyelesaian bulat dari persamaan
91X + 42W = 21:
Karena (91; 42) = 7 j 21, maka terdapat penyelesaian-penyelesaian bulat. Karena
21 = 3:7, maka dengan penggunaan (6.3) dan (6.2) dipunyai
21 = 3 7
= 3 (1 91

2 42) = 3 91

= 3 91

6 (1 294

2 126)

= 3 91

6 294 + 12 126:

6 42
41

Bab 6. Algoritma Euclid
Jadi, dipunyai penyelesaian bulat
x = 3; y = 12; z =

6:

Contoh 6.8 (HMMT 2002) Hitung
x = 2002 + 2; 20022 + 2; 20023 + 2; ::: :
Penyelesaian. Dicatat bahwa
20022 + 2 = 2002 (2000 + 2) + 2 = 2000 (2002 + 2) + 6:
Jadi, berdasarkan Algoritma Euclid dipunyai
2002 + 2; 20022 + 2 = (2004; 6) = 6:
Karena itu x j 2002 + 2; 20022 + 2 = (2004; 6) atau x j 6. Di sisi lain, setiap bilangan
dalam barisan 2002+2; 20022 +2; 20023 +2; ::: dapat dibagi oleh 2. Lebih lanjut, karena
2002 = 2001 + 1 = 667 3 + 1, untuk semua bilangan bulat positif k, 2002k = 3ak + 1
untuk suatu bilangan bulat ak . Jadi, 2002k + 2 dapat dibagi oleh 3. Karena 2 dan
3 adalah prima relatif, setiap bilangan dalam barisan tersebut dapat dibagi 6. Oleh
karena itu x = 6.
Latihan 6.9 Tentukan
1. (34567; 987)
2. (560; 600)
3. (4554; 36)
4. (8098643070; 8173826342)
Latihan 6.10 Selesaikan persamaan diophantine berikut ini, jika diketahui bahwa penyelesaiannya ada.
1. 24x + 25y = 18
2. 3456x + 246y = 44
3. 1998x + 2000y = 33

6.1

Sistem Kongruensi Linear

Suatu sistem kongruensi linear dalam variabel x mempunyai bentuk
8
> a1 x = b1 (mod n1 )
>
>
< a2 x = b2 (mod n2 )
:
.
.
>
.
>
>
:
ar x = br (mod nr )
42

Bab 6. Algoritma Euclid

Berikut ini diberikan suatu contoh yang mengilustrasikan bahwa hubungan antara modulo kongruensi-kongruensi merupakan syarat terpenting dalam menentukan apakah suatu sistem kongruensi linear mempunyai penyelesaian atau tidak.
Contoh 6.11 Apakah sistem kongruensi
x = 8 (mod 12) ; x = 6 (mod 9)
mempunyai penyelesaian? Berikan penjelasan.
Penyelesaian. Karena (12; 9) = 3 dan kongruensi pertama mengakibatkan x = 8 =
2 (mod 3), sedangkan kongruensi kedua mengakibatkan x = 6 = 0 (mod 3), maka sistem
tidak mempunyai penyelesaian.
Sifat penyelesaian dari suatu sistem kongruensi linear ditemukan pertama kali oleh
matematikawan Cina kuno dan ditulis pertama kali dalam Shushu Jiuzhang (Nine
Chapters on the Mathematical Arts) oleh matematikawan abad 13 Qin Jiushao.
Teorema 6.12 (Teorema Sisa Cina) Jika n1 ; n2 2 Z+ adalah koprima dan b1 ; b2 2
Z, maka sistem kongruensi
x = b1 (mod n1 ) ; x = b2 (mod n2 )
mempunyai suatu penyelesaian tunggal di modulo n1 n2 .
Contoh 6.13 Selesaikan sistem
x = 2 (mod 5) ; x = 1 (mod 3) :
Penyelesaian. Dituliskan x = 2 (mod 5) menjadi x = 2 + 5m untuk suatu m 2 Z dan
dituliskan x = 1 (mod 3) menjadi x = 1 + 3n untuk suatu n 2 Z. Disamakan kedua
persamaan untuk memperoleh 2 + 5m = 1 + 3n atau 3n 5m = 1. Persamaan terakhir
dapat diselesaikan menggunakan algoritma Euclid yang menyatakan kombinasi linear
dari (3; 5) = 1:
3 2 + ( 5) 1 = 1:
Jadi penyelesaiannya adalah m = 1 dan n = 2, sedangkan penyelesaian umumnya yaitu
m = 1 + 3t dan
karena 3 (2 + 5t)

n = 2 + 5t;

untuk suatu t 2 Z;

5 (1 + 3t) = 1. Jadi, x yang memenuhi kedua kongruensi yaitu
x = 2 + 5 (1 + 3t) = 7 + 15t;

t 2 Z;

atau dengan kata lain x = 7 (mod 15).
Contoh 6.14 Selesaikan sistem kongruensi
3x = 1 (mod 4) ; 5x = 2 (mod 7) :
Penyelesaian. Dimulai dengan pengamatan bahwa 32 = 9 = 1 (mod 4) dan 3 5 =
15 = 1 (mod 7), karena itu sistem kongruensi asli ekivalen dengan sistem kongruensi
x = 3 (mod 4) ; x = 6 (mod 7) :
43

Bab 6. Algoritma Euclid

Selanjutnya sistem diselesaikan seperti pada contoh sebelumnya (diserahkan kepada
pembaca sebagai latihan) untuk memperoleh penyelesaian bilangan bulat umum yaitu
x = 27 (mod 28).
Contoh 6.15 Tentukan semua penyelesaian bulat dari sistem kongruensi
7x = 1 (mod 8) ; x = 2 (mod 3) ; x = 1 (mod 5) :
Penyelesaian. Diproses dalam dua tahap. Tahap pertama diselesaikan sistem kongruensi
7x = 1 (mod 8) ; x = 2 (mod 3)
di modulo 24. Diamati bahwa 72 = 1 (mod 8), sehingga sistem kongruensi menjadi
x = 7 (mod 8) ; x = 2 (mod 3) :
Diaplikasikan Algoritma Euclid untuk memperoleh
( 1)

8+3

3 = 1;

dan karena itu diperoleh penyelesaian tunggal
x = ( 1)

8

2+3

3

7 = 23 (mod 24) :

Tahap kedua diselesaikan sistem kongruensi
x = 23 (mod 24) ; x = 1 (mod 5) :
Dipunyai bahwa x = 23 (mod 24) =
x=

1 (mod 24), sehingga sistem kongruensi menjadi

1 (mod 24) ; x = 1 (mod 5) :

Diaplikasikan Algoritma Euclid untuk memperoleh
( 1)

24 + 5

5 = 1;

dan karena itu diperoleh penyelesaian tunggal
x = ( 1)

24

1+5

5

( 1) = 71 (mod 120) :

Jadi, penyelesaian bulat umum dari sistem yaitu x = 71 + 120n, n 2 N.
Latihan 6.16 Selesaikan sistem persamaan berikut ini.
a) x = 1 (mod 2) ; x = 2 (mod 3) :
b) 3x = 1 (mod 5) ; 2x = 3 (mod 7) :
c) x = 5 (mod 15) ; 4x = 7 (mod 11) :
d) 2x = 3 (mod 5) ; 7x = 9 (mod 13) :
Latihan 6.17 Selesaikan sistem persamaan berikut ini.
a) x = 1 (mod 2) ; x = 1 (mod 3) ; x = 1 (mod 5) :
Bab 6. Algoritma Euclid
b) x = 1 (mod 2) ; x = 2 (mod 3) ; x = 4 (mod 5) :
c) 2x = 1 (mod 3) ; 3x = 4 (mod 5) ; 3x = 7 (mod 8) :
d) 5x = 2 (mod 3) ; 4x = 3 (mod 10) ; 5x = 9 (mod 25) :
Latihan 6.18 Cari penyelesaian untuk sistem persamaan
x = 1 (mod 3) ; x = 3 (mod 5) ; x = 5 (mod 7) ; x = 7 (mod 9) :

44
Bab 7

Fungsi-fungsi Bilangan-Teoritik
7.1

Fungsi Floor

Untuk suatu bilangan riil x, terdapat secara tunggal bilangan bulat n sedemikian sehingga n x < n + 1. Dengan kata lain, n adalah bilangan bulat terbesar yang tidak
melebihi x, atau n dinamakan ‡oor dari x, dan dinotasikan dengan bxc. Selisih x bxc
dinamakan bagian pecahan dari x dan dinotasikan dengan fxg. Bilangan bulat terkecil
yang lebih besar atau sama dengan x dinamakan ceiling dari x dan dinotasikan dengan dxe. Jika x adalah suatu bilangan bulat, maka bxc = dxe dan fxg = 0; jika x
bukan suatu bilangan bulat, maka dxe = bxc + 1. Berikut ini diberikan contoh-contoh
sederhana:
1. b3; 1c = 3 dan d3; 1e = 4
2. b3c = 3 dan d3e = 3
3. b 3; 1c =

4 dan d 3; 1e =

3

Lemma 7.1 Untuk setiap x 2 R berlaku
x

1 < bxc

x:

Bukti. Diambil n = bxc, maka dipunyai bahwa n x < n + 1. Hal ini memberikan
bxc x, seperti dalam de…nisi. Ini juga memberikan x < n + 1 yang mengakibatkan
bahwa x 1 < n, artinya x 1 < bxc.
Selanjutnya, fungsi ‡oor memiliki sifat-sifat seperti dalam teorema berikut ini.
Teorema 7.2 Jika

;

2 R, a 2 Z, n 2 N, maka

(1) b + ac = b c + a
j k
b c
=
(2)
n
n
(3) b c + b c

b + c

b c+b c+1

Bukti.
(1) Diambil m = b + ac, maka m
+a < m+1. Karena itu m a
Ini berarti bahwa m a = b c atau m = b c + a.
45

< m a+1.
46

Bab 7. Fungsi-fungsi Bilangan-Teoritik
(2) Dituliskan

sebagai

j k

=

+ , 0

< 1. Karena n

n
n
n
bilangan bulat, berdasarkan (1) disimpulkan bahwa
j j k
k
j k
b c= n
+n =n
+ bn c :
n
n
Dipunyai 0 bn c
maka diperoleh

Ini berarti bahwa

j k
n

adalah suatu

bn c
< 1. Jika diambil
n

n < n, sehingga 0
b c j k
=
+ ; 0
n
n
j k
b c
=
.
n
n

=

bn c
,
n

< 1:

(3) Dari ketaksamaan
1<b c
dan
1<b c
diperoleh +
2<
b c+b c
+ . Karena b c + b c adalah suatu bilangan bulat yang kurang
dari atau sama dengan + , maka b c+b c pasti kurang dari atau sama dengan
bagian bulat dari + , yaitu b + c. Selain itu, + kurang dari bilangan bulat
b c + b c + 2, sehingga b + c pasti kurang dari b c + b c + 2, dan akibatnya
b + c < b c + b c + 2 menghasilkan b + c b c + b c + 1.
Contoh 7.3 (APMC 1999) Diambil suatu barisan bilangan riil a1 , a2 , ... yang
memenuhi
ai+j ai + aj
untuk semua i; j = 1; 2; :::. Buktikan bahwa
a1 +

a2 a3
+
+
2
3

+

an
n

an

untuk semua bilangan bulat positif n.
Bukti. Digunakan induksi kuat seperti berikut ini. Basis induksi untuk n = 1 dan
2 adalah trivial. Sekarang diasumsikan bahwa pernyataan benar untuk n
k untuk
suatu bilangan bulat positif k 2. Artinya,
a1
a2
a1 +
2
a1 +

a2
+
2

+

a1 ;
a2 ;
.
.
.

ak
k

ak :

Dijumlahkan semua ketaksamaan untuk memperoleh
ka1 + (k

1)

a2
+
2

+

ak
k

a1 + a2 +

+ ak :

Selanjutnya kedua ruas dari ketaksamaan terakhir ditambahkan dengan a1 +a2 +
yang menghasilkan
(k + 1) a1 +

a2 a3
+
+
2
3

+

an
n

(a1 + ak ) + (a2 + ak
kak+1 :

1)

+

+ak ,

+ (ak + a1 )
47

Bab 7. Fungsi-fungsi Bilangan-Teoritik
Jika kedua ruas dari ketaksamaan terakhir dibagi dengan (k + 1), maka
a2 a3
+
+
2
3

a1 +

+

an
n

kak+1
k+1

atau

a2 a3
an
ak+1
+
+
+
+
2
3
n
k+1
Ini berarti pernyataan benar untuk n = k + 1.
a1 +

ak+1 :

Contoh 7.4 (USAMO 1981) Untuk suatu bilangan positif x, buktikan bahwa
bxc +

b2xc b3xc
+
+
2
3

bnxc
n

+

bnxc :

Bukti. Berdasarkan Teorema 7.2 (3), ini merupakan kasus khusus dari Contoh 7.3
dengan mengambil ai = bixc.
Contoh 7.5 (Putnam 1948) Jika n adalah suatu bilangan bulat positif, tunjukkan
bahwa
p
p
p
n+ n+1 =
4n + 2 :
Bukti. Diperhatikan bahwa
4n + 1 <

p

n+

p

n+1

2

= 2n + 2

p

n2 + n + 1 < 4n + 3

p
p
p
p
karena n2 + n > n2 = n dan juga n2 + n < n2 + 2n + 1 = n + 1. Karena itu
diperoleh
p
p
p
p
4n + 1 < n + n + 1 < 4n + 3:
Bilangan 4n+2 dan 4n+3 bukan merupakan bilangan kuadrat karena bilangan-bilangan
kuadrat dalam mod(4) kongruen dengan 0 atau 1, sehingga
p
p
4n + 2 =
4n + 3 :
Oleh karena itu

p

n+

p

n+1 =

p

4n + 2 :

Contoh 7.6 (Australia 1999) Selesaikan sistem persamaan:
x + byc + fzg = 200;

fxg + y + bzc = 190; 1;

bxc + fyg + z = 178; 8:
Penyelesaian. Karena x = bxc + fxg untuk semua bilangan riil x, maka jumlahan
dari tiga persamaan dalam sistem adalah
2x + 2y + 2z = 568; 9 atau x + y + z = 284; 45:
48

Bab 7. Fungsi-fungsi Bilangan-Teoritik

Selain itu, jika persamaan terakhir dikurangi dengan setiap persamaan dalam sistem,
maka diperoleh
fyg + bzc = 84; 45;

bxc + fzg = 94; 35;

fxg + byc = 105; 65:
Oleh karena itu bzc = bfyg + bzcc = b84; 45c = 84. Jadi, bzc = 84 dan fyg = 0; 45.
Dengan cara serupa diperoleh bxc = 94 dan fzg = 0; 35, dan juga byc = 105 dan
fxg = 0; 65. Dari sini dihasilkan x = 94; 65, y = 105; 45, dan z = 84; 35.
1
Contoh 7.7 (ARML 2003) Cari bilangan bulat positif n sedemikian sehingga pan
p
ling dekat dengan
123456789 .
Penyelesaian. Dicatat bahwa
11111; 112 = 123456765; 4321 < 123456789
< 123456789; 87654321 = 11111; 11112 :
Karena itu
dan

jp

k
123456789 = 11111

np
o
1
1
< 0; 11 <
123456789 < 0; 1111 < :
10
9

Contoh 7.8 (AIME 1997) Diandaikan bahwa a adalah positif, a
2 < a2 < 3. Cari nilai dari a12 144a 1 .

1

= a2 , dan

Penyelesaian. Pertama kali dicatat bahwa hipotesis yang diberikan mengakibatkan
a 1 = a 1 fkarena 1 < a dan 0 < a 1 < 1) dan a2 = a2 2. Karena itu a harus
memenuhi persamaan a 1 = a2 2 atau a3 2a 1 = 0. Persamaan terakhir dapat
dituliskan seperti
(a + 1) a2 a 1 = 0
p
1+ 5
yang akar positifnya hanya a =
. Digunakan hubungan a2 = a+1 dan a3 = 2a+1
2
untuk menghitung
a6 = 8a + 5, a12 = 144a + 89, dan a13 = 233a + 144
yang mengakibatkan bahwa
a12

144a

1

=

a13

144
a

= 233.

Contoh 7.9 Cari semua penyelesaian riil untuk persamaan
4x2

40 bxc + 51 = 0:
49

Bab 7. Fungsi-fungsi Bilangan-Teoritik
Penyelesaian. Dicatat bahwa
(2x

17) = 4x2

3) (2x

40x + 51

2

4x

40 bxc + 51 = 0;

yang memberikan 3 x 17 , dan berakibat 1
2
2
diberikan dapat dituliskan seperti
p
40 bxc
x=
2

bxc
51

8. Di sisi lain, persamaan yang

;

sehingga dari sini dipunyai
bxc =

$p

40 bxc
2

51

%

:

Dari pilihan bxc 2 f1; 2; :::; 8g, hanya 2, 6, 7, atau 8 yang memenuhi persamaan terakhir.
p
p
p
p
29
Jadi, penyelesaian untuk x adalah 2 , 189 , 229 , dan 269 .
2
2
2
Proposisi 7.10 (Identitas Hermite) Jika x adalah suatu bilangan riil dan n adalah
suatu bilangan bulat positif, maka
bxc + x +

2
1
+ x+
+
n
n

+ x+

n

1
n

= bnxc :

Bukti. Jika x adalah suatu bilangan bulat, maka hasil jelas benar. Diandaikan bahwa
x bukan suatu bilangan bulat, artinya 0 < fxg < 1. Karena itu terdapat 1 i n 1
sedemikian sehingga
fxg +

i

1
n

< 1 dan

fxg +

i
n

1;

(7.1)

artinya
n

i
n

fxg <

n

i+1
:
n

(7.2)

Berdasarkan (7.1) dipunyai
bxc = x +
dan
x+

i
=
n

1
=
n

= x+

= x+

n

1
n

i

1
n

= bxc + 1;

dan juga
bxc + x +

1
2
+ x+
+
n
n

+ x+

n

1

n
= i bxc + (n

i) (bxc + 1) = n bxc + n

Di sisi lain, berdasarkan (7.2) diperoleh
n bxc + n

i

n bxc + n fxg = nx < n bxc + n

i + 1;

i: (7.3)
50

Bab 7. Fungsi-fungsi Bilangan-Teoritik
yang mengakibatkan
bnxc = n bxc + n

i:

(7.4)

Dari (7.3) dan (7.4) disimpulkan bahwa
bxc + x +

2
1
+ x+
+
n
n

+ x+

n

1

= n bxc + n

n

i

= bnxc :

Contoh 7.11 (AIME 1991) Diandaikan bahwa r adalah suatu bilangan riil dimana
r+

20
19
+ r+
+
100
100

+ r+

91
= 546:
100

Cari b100rc.
Penyelesaian. Jumlahan yang diberikan mempunyai 91 19+1 = 73 suku, dan setiap
suku sama dengan brc atau brc + 1. Diamati bahwa 73 7 < 546 < 73 8, sehingga
diperoleh brc = 7. Karena 546 = 73 7 + 35, maka 38 suku pertama bernilai 7 dan 35
suku terakhir bernilai 8; artinya
r+
Akibatnya 7; 43

56
= 7 dan
100

r+

57
= 8:
100

r < 7; 44, dan karena itu b100rc = 743.

Contoh 7.12 (IMO 1968) Diambil x adalah suatu bilangan riil. Buktikan bahwa
1
X x + 2k
2k+1
k=0

= bxc :

Bukti. Dalam identitas Hermite diambil n = 2 yang memberikan
bxc + x +
atau
x+

1
= b2xc ;
2

1
= b2xc
2

bxc :

Diaplikasikan identitas terakhir secara berulang untuk memperoleh
!
1
1
1
X x + 2k
X
Xj x k j x k
x
1
=
+
=
= bxc :
2k+1
2k+1 2
2k
2k+1
k=0

7.2

k=0

k=0

Fungsi Legendre

Diambil p adalah suatu bilangan prima. Untuk sembarang bilangan bulat positif n,
dide…nisikan ep (n) = k sedemikian sehingga pk k n!. Fungsi aritmatika ep (n) dinamakan fungsi Legendre yang berasosiasi dengan bilangan prima p.
51

Bab 7. Fungsi-fungsi Bilangan-Teoritik

jak
Dicatat bahwa jika a > 0 dan n adalah suatu bilangan bulat positif, maka
adalah
n
banyaknya bilangan bulat positif yang tidak melebihi a dan merupakan kelipatan dari
n.
Teorema 7.13 (Rumus Legendre) Jika n adalah suatu bilangan bulat positif dan p
adalah suatu bilangan prima, maka
ep (n) = k =

1
X
j=1

n
pj

=

n
n
n
+ 2 + 3 +
p
p
p

:

Bukti. Diandaikan m 2 N dan 1 m n. Jika pr j m dan pr+1 - n, ingin dihitung
kontribusi dari r. Dengan kata lain, ingin dihitung kontribusi 1 untuk setiap j 2 N
sedemikian sehingga pj j m. Karena itu
k=

n
1
XX

1=

m=1 j=1
pj jm

1
n
XX

1=

j=1 m=1
pj jm

1
X
j=1

n
pj

dalam pandangan dari catatan di atas.
Contoh 7.14 Diambil s dan t adalah bilangan bulat positif sedemikian sehingga
7s k 400!

dan

3t k ((3!)!)!:

Hitung s + t.
Penyelesaian. Dicatat bahwa ((3!)!)! = (6!)! = 720!. Diaplikasikan rumus Legendre
untuk memperoleh
s = e7 (400) =

400
400
400
+
+
= 57 + 8 + 1 = 66
2
7
7
73

dan
720
720
720
720
720
+
+
+
+
3
32
33
34
35
= 240 + 80 + 26 + 8 + 2 = 356:

t = e3 (720) =

Jadi s + t = 66 + 356 = 422.

7.3

Bilangan Fermat

Untuk mencari semua bilangan prima berbentuk 2m + 1, Fermat melihat bahwa m
harus merupakan pangkat dari 2. Tentu saja jika m = k h dengan k adalah suatu
bilangan bulat ganjil yang lebih besar dari 1, maka
2m + 1 = 2h

k

+ 1 = 2h + 1

2h(k

1)

2h(k

2)

dan juga 2m + 1 tidak akan menjadi suatu bilangan prima.
Bilangan-bilangan bulat
n
fn = 22 + 1; n 0

+

2h + 1 ;
52

Bab 7. Fungsi-fungsi Bilangan-Teoritik
dinamakan bilangan-bilangan Fermat. Dipunyai
f0 = 3; f1 = 5; f2 = 17; f3 = 257; f4 = 65537; dan f5 = 4294967297:

Setelah diperiksa bahwa lima bilangan di atas adalah prima, Fermat menduga bahwa fn
adalah prima untuk semua n. Tetapi Euler membuktikan bahwa 641 j f5 . Argumennya
adalah seperti berikut:
f5 = 232 + 1 = 228 54 + 24
= 641 2

28

5 27

2

639 640 + 1

4

+ 1 = 228 641

6404

1

:

Ini tetap tidak diketahui apakah terdapat tak berhingga banyak bilangan-bilangan Fermat prima.
Contoh 7.15 Buktikan bahwa untuk bilangan bulat positif m dan n dengan m > n, fn
membagi fm 2.
Bukti. Diaplikasikan rumus a2
jukkan bahwa
fm

b2 = (a
2 = fm

b) (a + b) secara berulang untuk menun1 fm 2

f1 f0 :

Contoh 7.16 Untuk bilangan bulat positif berbeda m dan n, buktikan bahwa fm dan
fn adalah prima relatif.
Bukti. Berdasarkan contoh sebelumnya, dipunyai bahwa (fm ; fn ) = (f2 ; 2) = 1.
Contoh 7.17 Buktikan bahwa untuk semua bilangan bulat positif n, fn membagi 2fn
2.
Bukti. Dipunyai
2fn

2 = 2 22

2n

22

1 =2

n

22

n

n

1 :

(7.5)

n

Jelas bahwa 22 n adalah genap. Dicatat bahwa untuk suatu bilangan bulat positif
2n n
2m, x2m 1 dapat dibagi oleh x + 1. Karena itu x + 1 membagi x2
1. Dengan
n

n

pengambilan x = 22 disimpulkan bahwa fn = 22 + 1 = x + 1 membagi 22
sehingga dari (7.5) diperoleh bahwa fn membagi 2fn 2.

7.4

n

22

n

n

1,

Bilangan Mersenne

Bilangan-bilangan bulat Mn = 2n 1, n 1, dinamakan bilangan Mersenne. Ini jelas
bahwa jika n adalah composite, maka Mn juga composite. Karena itu Mk adalah prima
hanya jika k adalah prima. Selain itu, jika n = ab, dimana a dan b adalah bilanganbilangan bulat lebih besar dari 1, maka Ma dan Mb membagi Mn . Meskipun begitu
terdapat bilangan prima n untuk yang mana Mn adalah composite. Sebagai contoh,
47 j M23 , 167 j M83 , 263 j M13 , dan seterusnya.
Berikut ini diberikan suatu hasil tanpa bukti pada bilangan Mersenne.
Teorema 7.18 Jika p adalah suatu bilangan prima ganjil dan q adalah suatu pembagi
prima dari Mp , maka q = 2kp + 1 untuk suatu bilangan bulat positif k.
53

Bab 7. Fungsi-fungsi Bilangan-Teoritik

7.5

Bilangan Sempurna

Untuk setiap bilangan bulat positif n dide…nisikan fungsi
X
m;
(n) =
mjn

dimana m adalah bilangan bulat positif. Secara jelas, nilai (n) menyatakan jumlahan
dari semua pembagi positif dari bilangan bulat positif n. Suatu bilangan bulat n
2 dimana (n) = 2n dikenal sebagai bilangan sempurna (perfect number ). Sebagai
contoh, bilangan 6, 28, 496 adalah sempurna. Mudah dilihat bahwa 6 = 1+ 2+ 3 dan
28 = 1+ 2+ 4+ 7+ 14.
Berikut ini diberikan dua hasil (tanpa bukti) pada bilangan sempurna.
Teorema 7.19 Suatu bilangan bulat positif genap n adalah sempurna jika dan hanya
jika n = 2k 1 Mk = 2k 1 2k 1 untuk suatu bilangan bulat positif k dimana Mk
adalah prima. Lebih lanjut, tidak ada bilangan sempurna genap lainnya.
Teorema 7.20 Jika n adalah suatu bilangan sempurna ganjil, maka faktorisasi prima
dari n mempunyai bentuk
2b 2b
2b
n = pa q 1 1 q 2 2
qt t ;
dimana a dan p kongruen terhadap 1 modulo 4 dan t

2.
DAFTAR PUSTAKA
[1] Andreescu, T., D. Andrica, Z. Feng (2006). 104 Number Theory Problems: From
the Training of the USA IMO Team. Birkhäuser Boston.
[2] Baker, A. (2003). Algebra & Number Theory. Naskah. University of Glasgow.
[3] Budhi, W.S. (2003). Langkah Awal Menuju ke Olimpiade Matematika. Ricardo.
[4] Chen, W.W.L. (2003). Elementary Number Theory. Naskah. University of London.
[5] Clark, W.E. (2002). Elementary Number Theory. Naskah. University of South
Florida.
[6] Santos, D.A. (2007). Number Theory for Mathematical Contests. GNU Free Documentation License.
[7] Sato, N. (2009). Number Theory. CiteSeerX.
[8] Wilkins, D.R. (2005). Part I: Topics in Number Theory. Naskah.

54

More Related Content

What's hot

Sub grup normal dan grup fakto
Sub grup normal dan grup faktoSub grup normal dan grup fakto
Sub grup normal dan grup faktoYadi Pura
 
Teorema multinomial dan prinsip sarang merpati
Teorema multinomial dan prinsip sarang merpatiTeorema multinomial dan prinsip sarang merpati
Teorema multinomial dan prinsip sarang merpatiArdika MathEdu
 
Contoh soal dan pembahasan subgrup
Contoh soal dan pembahasan subgrupContoh soal dan pembahasan subgrup
Contoh soal dan pembahasan subgrupKabhi Na Kehna
 
Makalah geseran (translasi)
Makalah geseran (translasi)Makalah geseran (translasi)
Makalah geseran (translasi)Nia Matus
 
Matematika Diskrit Relasi Rekursif
Matematika Diskrit Relasi RekursifMatematika Diskrit Relasi Rekursif
Matematika Diskrit Relasi RekursifAyuk Wulandari
 
Rangkuman materi Isometri
Rangkuman materi IsometriRangkuman materi Isometri
Rangkuman materi IsometriNia Matus
 
BAB 2 Pencerminan (Refleksi)
BAB 2 Pencerminan (Refleksi)BAB 2 Pencerminan (Refleksi)
BAB 2 Pencerminan (Refleksi)Nia Matus
 
Homomorfisma grup
Homomorfisma grupHomomorfisma grup
Homomorfisma grupYadi Pura
 
Makalah setengah putaran
Makalah setengah putaranMakalah setengah putaran
Makalah setengah putaranNia Matus
 
Analisis Real (Barisan Bilangan Real) Latihan bagian 2.1
Analisis Real (Barisan Bilangan Real) Latihan bagian 2.1Analisis Real (Barisan Bilangan Real) Latihan bagian 2.1
Analisis Real (Barisan Bilangan Real) Latihan bagian 2.1Arvina Frida Karela
 
Fungsi phi dan teorema euler
Fungsi phi dan teorema eulerFungsi phi dan teorema euler
Fungsi phi dan teorema eulervionk
 
Pengantar analisis real_I
Pengantar analisis real_IPengantar analisis real_I
Pengantar analisis real_IFerry Angriawan
 
Modul 2 keterbagian bilangan bulat
Modul 2   keterbagian bilangan bulatModul 2   keterbagian bilangan bulat
Modul 2 keterbagian bilangan bulatAcika Karunila
 

What's hot (20)

Sub grup normal dan grup fakto
Sub grup normal dan grup faktoSub grup normal dan grup fakto
Sub grup normal dan grup fakto
 
Teori Group
Teori GroupTeori Group
Teori Group
 
Modul 3 kongruensi
Modul 3   kongruensiModul 3   kongruensi
Modul 3 kongruensi
 
Teorema multinomial dan prinsip sarang merpati
Teorema multinomial dan prinsip sarang merpatiTeorema multinomial dan prinsip sarang merpati
Teorema multinomial dan prinsip sarang merpati
 
Contoh soal dan pembahasan subgrup
Contoh soal dan pembahasan subgrupContoh soal dan pembahasan subgrup
Contoh soal dan pembahasan subgrup
 
Makalah geseran (translasi)
Makalah geseran (translasi)Makalah geseran (translasi)
Makalah geseran (translasi)
 
Matematika Diskrit Relasi Rekursif
Matematika Diskrit Relasi RekursifMatematika Diskrit Relasi Rekursif
Matematika Diskrit Relasi Rekursif
 
Rangkuman materi Isometri
Rangkuman materi IsometriRangkuman materi Isometri
Rangkuman materi Isometri
 
Geometri analitik ruang
Geometri analitik ruangGeometri analitik ruang
Geometri analitik ruang
 
Prinsip Inklusi Eksklusi
Prinsip Inklusi EksklusiPrinsip Inklusi Eksklusi
Prinsip Inklusi Eksklusi
 
BAB 2 Pencerminan (Refleksi)
BAB 2 Pencerminan (Refleksi)BAB 2 Pencerminan (Refleksi)
BAB 2 Pencerminan (Refleksi)
 
Homomorfisma grup
Homomorfisma grupHomomorfisma grup
Homomorfisma grup
 
Makalah setengah putaran
Makalah setengah putaranMakalah setengah putaran
Makalah setengah putaran
 
Analisis real-lengkap-a1c
Analisis real-lengkap-a1cAnalisis real-lengkap-a1c
Analisis real-lengkap-a1c
 
Ring
RingRing
Ring
 
Analisis Real (Barisan Bilangan Real) Latihan bagian 2.1
Analisis Real (Barisan Bilangan Real) Latihan bagian 2.1Analisis Real (Barisan Bilangan Real) Latihan bagian 2.1
Analisis Real (Barisan Bilangan Real) Latihan bagian 2.1
 
Fungsi phi dan teorema euler
Fungsi phi dan teorema eulerFungsi phi dan teorema euler
Fungsi phi dan teorema euler
 
Pengantar analisis real_I
Pengantar analisis real_IPengantar analisis real_I
Pengantar analisis real_I
 
Modul 2 keterbagian bilangan bulat
Modul 2   keterbagian bilangan bulatModul 2   keterbagian bilangan bulat
Modul 2 keterbagian bilangan bulat
 
Jawaban Soal Latihan
Jawaban Soal LatihanJawaban Soal Latihan
Jawaban Soal Latihan
 

Viewers also liked

Sistem bilangan bulat (ma kul teori bilangan)
Sistem bilangan bulat (ma kul teori bilangan)Sistem bilangan bulat (ma kul teori bilangan)
Sistem bilangan bulat (ma kul teori bilangan)Ig Fandy Jayanto
 
Teori bilangan bab ii
Teori bilangan bab iiTeori bilangan bab ii
Teori bilangan bab iiSeptian Amri
 
Konsep Bilangan Bulat
Konsep Bilangan BulatKonsep Bilangan Bulat
Konsep Bilangan BulatAbdul Rais P
 
Aplikasi modulo dalam menentukan hari lahir - teori bilangan - Roswati 2014B
Aplikasi modulo dalam menentukan hari lahir - teori bilangan - Roswati 2014BAplikasi modulo dalam menentukan hari lahir - teori bilangan - Roswati 2014B
Aplikasi modulo dalam menentukan hari lahir - teori bilangan - Roswati 2014Broswati_terbil
 
Mengapa negatif dikali negatif adalah positif?
Mengapa negatif dikali negatif adalah positif?Mengapa negatif dikali negatif adalah positif?
Mengapa negatif dikali negatif adalah positif?Hannasiti
 
ALJABAR LINEAR ELEMENTER
ALJABAR LINEAR ELEMENTERALJABAR LINEAR ELEMENTER
ALJABAR LINEAR ELEMENTERMella Imelda
 
Bahan ajar alin 2 rev 2014 pdf
Bahan ajar alin 2 rev 2014 pdfBahan ajar alin 2 rev 2014 pdf
Bahan ajar alin 2 rev 2014 pdfPawit Ngafani
 

Viewers also liked (15)

Sistem bilangan bulat (ma kul teori bilangan)
Sistem bilangan bulat (ma kul teori bilangan)Sistem bilangan bulat (ma kul teori bilangan)
Sistem bilangan bulat (ma kul teori bilangan)
 
Modul bilangan bulat
Modul bilangan bulatModul bilangan bulat
Modul bilangan bulat
 
Teori bilangan bab ii
Teori bilangan bab iiTeori bilangan bab ii
Teori bilangan bab ii
 
Konsep Bilangan Bulat
Konsep Bilangan BulatKonsep Bilangan Bulat
Konsep Bilangan Bulat
 
83047338 modul2
83047338 modul283047338 modul2
83047338 modul2
 
Teori bilangan
Teori bilanganTeori bilangan
Teori bilangan
 
Aplikasi modulo dalam menentukan hari lahir - teori bilangan - Roswati 2014B
Aplikasi modulo dalam menentukan hari lahir - teori bilangan - Roswati 2014BAplikasi modulo dalam menentukan hari lahir - teori bilangan - Roswati 2014B
Aplikasi modulo dalam menentukan hari lahir - teori bilangan - Roswati 2014B
 
Aplikasi modulo dalam
Aplikasi modulo dalamAplikasi modulo dalam
Aplikasi modulo dalam
 
Mengapa negatif dikali negatif adalah positif?
Mengapa negatif dikali negatif adalah positif?Mengapa negatif dikali negatif adalah positif?
Mengapa negatif dikali negatif adalah positif?
 
Keterbagian
KeterbagianKeterbagian
Keterbagian
 
Kongruensi linear simultan
Kongruensi linear simultanKongruensi linear simultan
Kongruensi linear simultan
 
Teori bilangan bab3_1
Teori bilangan bab3_1Teori bilangan bab3_1
Teori bilangan bab3_1
 
ALJABAR LINEAR ELEMENTER
ALJABAR LINEAR ELEMENTERALJABAR LINEAR ELEMENTER
ALJABAR LINEAR ELEMENTER
 
Bahan ajar alin 2 rev 2014 pdf
Bahan ajar alin 2 rev 2014 pdfBahan ajar alin 2 rev 2014 pdf
Bahan ajar alin 2 rev 2014 pdf
 
R5 h kel 4 teori bil 2
R5 h kel 4 teori bil 2R5 h kel 4 teori bil 2
R5 h kel 4 teori bil 2
 

Similar to TEORI BILANGAN

Matematika Diskrit - 11 kompleksitas algoritma - 04
Matematika Diskrit - 11 kompleksitas algoritma - 04Matematika Diskrit - 11 kompleksitas algoritma - 04
Matematika Diskrit - 11 kompleksitas algoritma - 04KuliahKita
 
Kisi-kisi UN Matematika kelas 9 SMP
Kisi-kisi UN Matematika kelas 9 SMPKisi-kisi UN Matematika kelas 9 SMP
Kisi-kisi UN Matematika kelas 9 SMPNisriinaaf
 
Induksi matematika teobil
Induksi matematika teobilInduksi matematika teobil
Induksi matematika teobilNailul Hasibuan
 
Kel 1 bilangan
Kel 1 bilanganKel 1 bilangan
Kel 1 bilanganMas Becak
 
Modul bentuk pangkat
Modul bentuk pangkatModul bentuk pangkat
Modul bentuk pangkatVino Hidayat
 
INDUKSI MATEMATIK
 INDUKSI MATEMATIK INDUKSI MATEMATIK
INDUKSI MATEMATIKT. Astari
 
Matdis-Induksi Matematika
Matdis-Induksi MatematikaMatdis-Induksi Matematika
Matdis-Induksi MatematikaCeria Agnantria
 
Pengantar_Analisis_Real_I.pdf
Pengantar_Analisis_Real_I.pdfPengantar_Analisis_Real_I.pdf
Pengantar_Analisis_Real_I.pdfHamzaHamid27
 
Kuliah 4 induksi matematika
Kuliah 4   induksi matematikaKuliah 4   induksi matematika
Kuliah 4 induksi matematikaEnosLolang
 
Kunci jawaban-un-matematika-paket-i
Kunci jawaban-un-matematika-paket-iKunci jawaban-un-matematika-paket-i
Kunci jawaban-un-matematika-paket-iDian Darmawanoptimis
 
Soal un fisika 2012 dan pembahasannya
Soal un fisika 2012 dan pembahasannyaSoal un fisika 2012 dan pembahasannya
Soal un fisika 2012 dan pembahasannyaRenny Aniwarna
 

Similar to TEORI BILANGAN (20)

Matematika Diskrit - 11 kompleksitas algoritma - 04
Matematika Diskrit - 11 kompleksitas algoritma - 04Matematika Diskrit - 11 kompleksitas algoritma - 04
Matematika Diskrit - 11 kompleksitas algoritma - 04
 
Kisi-kisi UN Matematika kelas 9 SMP
Kisi-kisi UN Matematika kelas 9 SMPKisi-kisi UN Matematika kelas 9 SMP
Kisi-kisi UN Matematika kelas 9 SMP
 
Induksi matematika teobil
Induksi matematika teobilInduksi matematika teobil
Induksi matematika teobil
 
Pendahulan teori bilangan
Pendahulan teori bilanganPendahulan teori bilangan
Pendahulan teori bilangan
 
Kel 1 bilangan
Kel 1 bilanganKel 1 bilangan
Kel 1 bilangan
 
Pertemuan ke 6 induksi matematika
Pertemuan ke 6   induksi matematikaPertemuan ke 6   induksi matematika
Pertemuan ke 6 induksi matematika
 
Diskret III Induksi
Diskret III InduksiDiskret III Induksi
Diskret III Induksi
 
Ppt induksi matematika
Ppt induksi matematikaPpt induksi matematika
Ppt induksi matematika
 
Modul bentuk pangkat
Modul bentuk pangkatModul bentuk pangkat
Modul bentuk pangkat
 
Modul bab 1
Modul bab 1Modul bab 1
Modul bab 1
 
INDUKSI MATEMATIK
 INDUKSI MATEMATIK INDUKSI MATEMATIK
INDUKSI MATEMATIK
 
Matdis-Induksi Matematika
Matdis-Induksi MatematikaMatdis-Induksi Matematika
Matdis-Induksi Matematika
 
Pengantar_Analisis_Real_I.pdf
Pengantar_Analisis_Real_I.pdfPengantar_Analisis_Real_I.pdf
Pengantar_Analisis_Real_I.pdf
 
induksi matematika
induksi matematikainduksi matematika
induksi matematika
 
Kuliah 4 induksi matematika
Kuliah 4   induksi matematikaKuliah 4   induksi matematika
Kuliah 4 induksi matematika
 
Analisis Riel 1
Analisis Riel 1Analisis Riel 1
Analisis Riel 1
 
Ppt heppi pryitno
Ppt heppi pryitnoPpt heppi pryitno
Ppt heppi pryitno
 
Pertemuan 2 bilangan 2
Pertemuan 2 bilangan 2Pertemuan 2 bilangan 2
Pertemuan 2 bilangan 2
 
Kunci jawaban-un-matematika-paket-i
Kunci jawaban-un-matematika-paket-iKunci jawaban-un-matematika-paket-i
Kunci jawaban-un-matematika-paket-i
 
Soal un fisika 2012 dan pembahasannya
Soal un fisika 2012 dan pembahasannyaSoal un fisika 2012 dan pembahasannya
Soal un fisika 2012 dan pembahasannya
 

More from Dia Cahyawati

Media pembelajaran (dilatasi)
Media pembelajaran (dilatasi)Media pembelajaran (dilatasi)
Media pembelajaran (dilatasi)Dia Cahyawati
 
Jurnal pengembangan materi dilatasi
Jurnal pengembangan materi dilatasiJurnal pengembangan materi dilatasi
Jurnal pengembangan materi dilatasiDia Cahyawati
 
Korelasi dan regresi sederhana
Korelasi dan regresi sederhanaKorelasi dan regresi sederhana
Korelasi dan regresi sederhanaDia Cahyawati
 
Peningkatan keprofesionalan guru
Peningkatan keprofesionalan guruPeningkatan keprofesionalan guru
Peningkatan keprofesionalan guruDia Cahyawati
 
Ilmu%20 sosial%20dan%20budaya%201
Ilmu%20 sosial%20dan%20budaya%201Ilmu%20 sosial%20dan%20budaya%201
Ilmu%20 sosial%20dan%20budaya%201Dia Cahyawati
 
09soal olimp-mat-tk-provinsi-2009
09soal olimp-mat-tk-provinsi-200909soal olimp-mat-tk-provinsi-2009
09soal olimp-mat-tk-provinsi-2009Dia Cahyawati
 
Soal un-matematika-smp-22-314-ali-lia-23
Soal un-matematika-smp-22-314-ali-lia-23Soal un-matematika-smp-22-314-ali-lia-23
Soal un-matematika-smp-22-314-ali-lia-23Dia Cahyawati
 
Soal un-matematika-smp-22-312-bimo-fajar-39-283
Soal un-matematika-smp-22-312-bimo-fajar-39-283Soal un-matematika-smp-22-312-bimo-fajar-39-283
Soal un-matematika-smp-22-312-bimo-fajar-39-283Dia Cahyawati
 
Soal un-matematika-smp-21-214-ali-lia-23
Soal un-matematika-smp-21-214-ali-lia-23Soal un-matematika-smp-21-214-ali-lia-23
Soal un-matematika-smp-21-214-ali-lia-23Dia Cahyawati
 
Soal un-matematika-smp-21-212-bimo-fajar-39
Soal un-matematika-smp-21-212-bimo-fajar-39Soal un-matematika-smp-21-212-bimo-fajar-39
Soal un-matematika-smp-21-212-bimo-fajar-39Dia Cahyawati
 
Smp -matematika_2007
Smp  -matematika_2007Smp  -matematika_2007
Smp -matematika_2007Dia Cahyawati
 
Smp -matematika_2006
Smp  -matematika_2006Smp  -matematika_2006
Smp -matematika_2006Dia Cahyawati
 
Smp -matematika_2005
Smp  -matematika_2005Smp  -matematika_2005
Smp -matematika_2005Dia Cahyawati
 
Smp -matematika_2004
Smp  -matematika_2004Smp  -matematika_2004
Smp -matematika_2004Dia Cahyawati
 
Soal un-matematika-smp-31-221-akmal-fajar-32
Soal un-matematika-smp-31-221-akmal-fajar-32Soal un-matematika-smp-31-221-akmal-fajar-32
Soal un-matematika-smp-31-221-akmal-fajar-32Dia Cahyawati
 
Pegangan belajar matematika 1
Pegangan belajar matematika 1Pegangan belajar matematika 1
Pegangan belajar matematika 1Dia Cahyawati
 

More from Dia Cahyawati (20)

Media pembelajaran (dilatasi)
Media pembelajaran (dilatasi)Media pembelajaran (dilatasi)
Media pembelajaran (dilatasi)
 
Jurnal pengembangan materi dilatasi
Jurnal pengembangan materi dilatasiJurnal pengembangan materi dilatasi
Jurnal pengembangan materi dilatasi
 
Makalah lengkap
Makalah lengkapMakalah lengkap
Makalah lengkap
 
Korelasi dan regresi sederhana
Korelasi dan regresi sederhanaKorelasi dan regresi sederhana
Korelasi dan regresi sederhana
 
Peningkatan keprofesionalan guru
Peningkatan keprofesionalan guruPeningkatan keprofesionalan guru
Peningkatan keprofesionalan guru
 
Ilmu%20 sosial%20dan%20budaya%201
Ilmu%20 sosial%20dan%20budaya%201Ilmu%20 sosial%20dan%20budaya%201
Ilmu%20 sosial%20dan%20budaya%201
 
Media pembelajaran
Media pembelajaranMedia pembelajaran
Media pembelajaran
 
Trigonometri pptrad
Trigonometri pptradTrigonometri pptrad
Trigonometri pptrad
 
09soal olimp-mat-tk-provinsi-2009
09soal olimp-mat-tk-provinsi-200909soal olimp-mat-tk-provinsi-2009
09soal olimp-mat-tk-provinsi-2009
 
Soal un-matematika-smp-22-314-ali-lia-23
Soal un-matematika-smp-22-314-ali-lia-23Soal un-matematika-smp-22-314-ali-lia-23
Soal un-matematika-smp-22-314-ali-lia-23
 
Soal un-matematika-smp-22-312-bimo-fajar-39-283
Soal un-matematika-smp-22-312-bimo-fajar-39-283Soal un-matematika-smp-22-312-bimo-fajar-39-283
Soal un-matematika-smp-22-312-bimo-fajar-39-283
 
Soal un-matematika-smp-21-214-ali-lia-23
Soal un-matematika-smp-21-214-ali-lia-23Soal un-matematika-smp-21-214-ali-lia-23
Soal un-matematika-smp-21-214-ali-lia-23
 
Soal un-matematika-smp-21-212-bimo-fajar-39
Soal un-matematika-smp-21-212-bimo-fajar-39Soal un-matematika-smp-21-212-bimo-fajar-39
Soal un-matematika-smp-21-212-bimo-fajar-39
 
Smp -matematika_2007
Smp  -matematika_2007Smp  -matematika_2007
Smp -matematika_2007
 
Smp -matematika_2006
Smp  -matematika_2006Smp  -matematika_2006
Smp -matematika_2006
 
Smp -matematika_2005
Smp  -matematika_2005Smp  -matematika_2005
Smp -matematika_2005
 
Smp -matematika_2004
Smp  -matematika_2004Smp  -matematika_2004
Smp -matematika_2004
 
Soal un-matematika-smp-31-221-akmal-fajar-32
Soal un-matematika-smp-31-221-akmal-fajar-32Soal un-matematika-smp-31-221-akmal-fajar-32
Soal un-matematika-smp-31-221-akmal-fajar-32
 
Matematika 3
Matematika 3Matematika 3
Matematika 3
 
Pegangan belajar matematika 1
Pegangan belajar matematika 1Pegangan belajar matematika 1
Pegangan belajar matematika 1
 

Recently uploaded

RENCANA + Link2 Materi TRAINING "Effective LEADERSHIP & SUPERVISORY SKILL",
RENCANA + Link2 Materi TRAINING "Effective LEADERSHIP & SUPERVISORY  SKILL",RENCANA + Link2 Materi TRAINING "Effective LEADERSHIP & SUPERVISORY  SKILL",
RENCANA + Link2 Materi TRAINING "Effective LEADERSHIP & SUPERVISORY SKILL",Kanaidi ken
 
Topik 1 - Pengenalan Penghayatan Etika dan Peradaban Acuan Malaysia.pptx
Topik 1 - Pengenalan Penghayatan Etika dan Peradaban Acuan Malaysia.pptxTopik 1 - Pengenalan Penghayatan Etika dan Peradaban Acuan Malaysia.pptx
Topik 1 - Pengenalan Penghayatan Etika dan Peradaban Acuan Malaysia.pptxsyafnasir
 
LA PI 2 PE NDIDIKAN GURU PENGGERAK A9 OK
LA PI 2 PE NDIDIKAN GURU PENGGERAK A9 OKLA PI 2 PE NDIDIKAN GURU PENGGERAK A9 OK
LA PI 2 PE NDIDIKAN GURU PENGGERAK A9 OKDeviIndriaMustikorin
 
1.2.a.6. Demonstrasi Konstektual - Modul 1.2 (Shinta Novianti - CGP A10).pdf
1.2.a.6. Demonstrasi Konstektual - Modul 1.2 (Shinta Novianti - CGP A10).pdf1.2.a.6. Demonstrasi Konstektual - Modul 1.2 (Shinta Novianti - CGP A10).pdf
1.2.a.6. Demonstrasi Konstektual - Modul 1.2 (Shinta Novianti - CGP A10).pdfShintaNovianti1
 
Pembahasan Soal UKOM gerontik persiapan ukomnas
Pembahasan Soal UKOM gerontik persiapan ukomnasPembahasan Soal UKOM gerontik persiapan ukomnas
Pembahasan Soal UKOM gerontik persiapan ukomnasAZakariaAmien1
 
Panduan Mengisi Dokumen Tindak Lanjut.pdf
Panduan Mengisi Dokumen Tindak Lanjut.pdfPanduan Mengisi Dokumen Tindak Lanjut.pdf
Panduan Mengisi Dokumen Tindak Lanjut.pdfandriasyulianto57
 
Teknik Menjawab Kertas P.Moral SPM 2024.pptx
Teknik Menjawab Kertas P.Moral SPM  2024.pptxTeknik Menjawab Kertas P.Moral SPM  2024.pptx
Teknik Menjawab Kertas P.Moral SPM 2024.pptxwongcp2
 
Materi power point Kepemimpinan leadership .ppt
Materi power point Kepemimpinan leadership .pptMateri power point Kepemimpinan leadership .ppt
Materi power point Kepemimpinan leadership .pptAcemediadotkoM1
 
Keberagaman-Peserta-Didik-dalam-Psikologi-Pendidikan.pptx
Keberagaman-Peserta-Didik-dalam-Psikologi-Pendidikan.pptxKeberagaman-Peserta-Didik-dalam-Psikologi-Pendidikan.pptx
Keberagaman-Peserta-Didik-dalam-Psikologi-Pendidikan.pptxLeniMawarti1
 
Edukasi Haji 2023 pembinaan jemaah hajii
Edukasi Haji 2023 pembinaan jemaah hajiiEdukasi Haji 2023 pembinaan jemaah hajii
Edukasi Haji 2023 pembinaan jemaah hajiiIntanHanifah4
 
Modul Ajar Bahasa Indonesia Kelas 8 Fase D
Modul Ajar Bahasa Indonesia Kelas 8 Fase DModul Ajar Bahasa Indonesia Kelas 8 Fase D
Modul Ajar Bahasa Indonesia Kelas 8 Fase DAbdiera
 
aksi nyata pendidikan inklusif.pelatihan mandiri pmm
aksi nyata pendidikan inklusif.pelatihan mandiri pmmaksi nyata pendidikan inklusif.pelatihan mandiri pmm
aksi nyata pendidikan inklusif.pelatihan mandiri pmmeunikekambe10
 
Workshop penulisan buku (Buku referensi, monograf, BUKU...
Workshop penulisan buku                       (Buku referensi, monograf, BUKU...Workshop penulisan buku                       (Buku referensi, monograf, BUKU...
Workshop penulisan buku (Buku referensi, monograf, BUKU...Riyan Hidayatullah
 
Dinamika perwujudan Pancasila sebagai Dasar Negara dan Pandangan Hidup Bangsa
Dinamika perwujudan Pancasila sebagai Dasar Negara dan Pandangan Hidup BangsaDinamika perwujudan Pancasila sebagai Dasar Negara dan Pandangan Hidup Bangsa
Dinamika perwujudan Pancasila sebagai Dasar Negara dan Pandangan Hidup BangsaEzraCalva
 
SBM_Kelompok-7_Alat dan Media Pembelajaran.pptx
SBM_Kelompok-7_Alat dan Media Pembelajaran.pptxSBM_Kelompok-7_Alat dan Media Pembelajaran.pptx
SBM_Kelompok-7_Alat dan Media Pembelajaran.pptxFardanassegaf
 
P_E_R_I_L_A_K_U__K_O_N_S_E_L_O_R__v.1.ppt
P_E_R_I_L_A_K_U__K_O_N_S_E_L_O_R__v.1.pptP_E_R_I_L_A_K_U__K_O_N_S_E_L_O_R__v.1.ppt
P_E_R_I_L_A_K_U__K_O_N_S_E_L_O_R__v.1.pptAfifFikri11
 
MA Kelas XII Bab 1 materi musik mkontemnporerFase F.pdf
MA Kelas XII  Bab 1 materi musik mkontemnporerFase F.pdfMA Kelas XII  Bab 1 materi musik mkontemnporerFase F.pdf
MA Kelas XII Bab 1 materi musik mkontemnporerFase F.pdfcicovendra
 
UNGGAH PEGANGAN LOKAKARYA DAN PENDAMPINGAN INDIVIDU DALAM KEGIATAN PEMBEKALAN...
UNGGAH PEGANGAN LOKAKARYA DAN PENDAMPINGAN INDIVIDU DALAM KEGIATAN PEMBEKALAN...UNGGAH PEGANGAN LOKAKARYA DAN PENDAMPINGAN INDIVIDU DALAM KEGIATAN PEMBEKALAN...
UNGGAH PEGANGAN LOKAKARYA DAN PENDAMPINGAN INDIVIDU DALAM KEGIATAN PEMBEKALAN...jumadsmanesi
 
MTK BAB 5 PENGOLAHAN DATA (Materi 2).pptx
MTK BAB 5 PENGOLAHAN DATA (Materi 2).pptxMTK BAB 5 PENGOLAHAN DATA (Materi 2).pptx
MTK BAB 5 PENGOLAHAN DATA (Materi 2).pptxssuser0239c1
 
PPT-Sistem-Pencernaan-Manusia-Kelas-8-K13.pptx
PPT-Sistem-Pencernaan-Manusia-Kelas-8-K13.pptxPPT-Sistem-Pencernaan-Manusia-Kelas-8-K13.pptx
PPT-Sistem-Pencernaan-Manusia-Kelas-8-K13.pptxdanangpamungkas11
 

Recently uploaded (20)

RENCANA + Link2 Materi TRAINING "Effective LEADERSHIP & SUPERVISORY SKILL",
RENCANA + Link2 Materi TRAINING "Effective LEADERSHIP & SUPERVISORY  SKILL",RENCANA + Link2 Materi TRAINING "Effective LEADERSHIP & SUPERVISORY  SKILL",
RENCANA + Link2 Materi TRAINING "Effective LEADERSHIP & SUPERVISORY SKILL",
 
Topik 1 - Pengenalan Penghayatan Etika dan Peradaban Acuan Malaysia.pptx
Topik 1 - Pengenalan Penghayatan Etika dan Peradaban Acuan Malaysia.pptxTopik 1 - Pengenalan Penghayatan Etika dan Peradaban Acuan Malaysia.pptx
Topik 1 - Pengenalan Penghayatan Etika dan Peradaban Acuan Malaysia.pptx
 
LA PI 2 PE NDIDIKAN GURU PENGGERAK A9 OK
LA PI 2 PE NDIDIKAN GURU PENGGERAK A9 OKLA PI 2 PE NDIDIKAN GURU PENGGERAK A9 OK
LA PI 2 PE NDIDIKAN GURU PENGGERAK A9 OK
 
1.2.a.6. Demonstrasi Konstektual - Modul 1.2 (Shinta Novianti - CGP A10).pdf
1.2.a.6. Demonstrasi Konstektual - Modul 1.2 (Shinta Novianti - CGP A10).pdf1.2.a.6. Demonstrasi Konstektual - Modul 1.2 (Shinta Novianti - CGP A10).pdf
1.2.a.6. Demonstrasi Konstektual - Modul 1.2 (Shinta Novianti - CGP A10).pdf
 
Pembahasan Soal UKOM gerontik persiapan ukomnas
Pembahasan Soal UKOM gerontik persiapan ukomnasPembahasan Soal UKOM gerontik persiapan ukomnas
Pembahasan Soal UKOM gerontik persiapan ukomnas
 
Panduan Mengisi Dokumen Tindak Lanjut.pdf
Panduan Mengisi Dokumen Tindak Lanjut.pdfPanduan Mengisi Dokumen Tindak Lanjut.pdf
Panduan Mengisi Dokumen Tindak Lanjut.pdf
 
Teknik Menjawab Kertas P.Moral SPM 2024.pptx
Teknik Menjawab Kertas P.Moral SPM  2024.pptxTeknik Menjawab Kertas P.Moral SPM  2024.pptx
Teknik Menjawab Kertas P.Moral SPM 2024.pptx
 
Materi power point Kepemimpinan leadership .ppt
Materi power point Kepemimpinan leadership .pptMateri power point Kepemimpinan leadership .ppt
Materi power point Kepemimpinan leadership .ppt
 
Keberagaman-Peserta-Didik-dalam-Psikologi-Pendidikan.pptx
Keberagaman-Peserta-Didik-dalam-Psikologi-Pendidikan.pptxKeberagaman-Peserta-Didik-dalam-Psikologi-Pendidikan.pptx
Keberagaman-Peserta-Didik-dalam-Psikologi-Pendidikan.pptx
 
Edukasi Haji 2023 pembinaan jemaah hajii
Edukasi Haji 2023 pembinaan jemaah hajiiEdukasi Haji 2023 pembinaan jemaah hajii
Edukasi Haji 2023 pembinaan jemaah hajii
 
Modul Ajar Bahasa Indonesia Kelas 8 Fase D
Modul Ajar Bahasa Indonesia Kelas 8 Fase DModul Ajar Bahasa Indonesia Kelas 8 Fase D
Modul Ajar Bahasa Indonesia Kelas 8 Fase D
 
aksi nyata pendidikan inklusif.pelatihan mandiri pmm
aksi nyata pendidikan inklusif.pelatihan mandiri pmmaksi nyata pendidikan inklusif.pelatihan mandiri pmm
aksi nyata pendidikan inklusif.pelatihan mandiri pmm
 
Workshop penulisan buku (Buku referensi, monograf, BUKU...
Workshop penulisan buku                       (Buku referensi, monograf, BUKU...Workshop penulisan buku                       (Buku referensi, monograf, BUKU...
Workshop penulisan buku (Buku referensi, monograf, BUKU...
 
Dinamika perwujudan Pancasila sebagai Dasar Negara dan Pandangan Hidup Bangsa
Dinamika perwujudan Pancasila sebagai Dasar Negara dan Pandangan Hidup BangsaDinamika perwujudan Pancasila sebagai Dasar Negara dan Pandangan Hidup Bangsa
Dinamika perwujudan Pancasila sebagai Dasar Negara dan Pandangan Hidup Bangsa
 
SBM_Kelompok-7_Alat dan Media Pembelajaran.pptx
SBM_Kelompok-7_Alat dan Media Pembelajaran.pptxSBM_Kelompok-7_Alat dan Media Pembelajaran.pptx
SBM_Kelompok-7_Alat dan Media Pembelajaran.pptx
 
P_E_R_I_L_A_K_U__K_O_N_S_E_L_O_R__v.1.ppt
P_E_R_I_L_A_K_U__K_O_N_S_E_L_O_R__v.1.pptP_E_R_I_L_A_K_U__K_O_N_S_E_L_O_R__v.1.ppt
P_E_R_I_L_A_K_U__K_O_N_S_E_L_O_R__v.1.ppt
 
MA Kelas XII Bab 1 materi musik mkontemnporerFase F.pdf
MA Kelas XII  Bab 1 materi musik mkontemnporerFase F.pdfMA Kelas XII  Bab 1 materi musik mkontemnporerFase F.pdf
MA Kelas XII Bab 1 materi musik mkontemnporerFase F.pdf
 
UNGGAH PEGANGAN LOKAKARYA DAN PENDAMPINGAN INDIVIDU DALAM KEGIATAN PEMBEKALAN...
UNGGAH PEGANGAN LOKAKARYA DAN PENDAMPINGAN INDIVIDU DALAM KEGIATAN PEMBEKALAN...UNGGAH PEGANGAN LOKAKARYA DAN PENDAMPINGAN INDIVIDU DALAM KEGIATAN PEMBEKALAN...
UNGGAH PEGANGAN LOKAKARYA DAN PENDAMPINGAN INDIVIDU DALAM KEGIATAN PEMBEKALAN...
 
MTK BAB 5 PENGOLAHAN DATA (Materi 2).pptx
MTK BAB 5 PENGOLAHAN DATA (Materi 2).pptxMTK BAB 5 PENGOLAHAN DATA (Materi 2).pptx
MTK BAB 5 PENGOLAHAN DATA (Materi 2).pptx
 
PPT-Sistem-Pencernaan-Manusia-Kelas-8-K13.pptx
PPT-Sistem-Pencernaan-Manusia-Kelas-8-K13.pptxPPT-Sistem-Pencernaan-Manusia-Kelas-8-K13.pptx
PPT-Sistem-Pencernaan-Manusia-Kelas-8-K13.pptx
 

TEORI BILANGAN

  • 1. DIKTAT KULIAH (2 sks) MX 127 Teori Bilangan (Revisi Terakhir: Juli 2009 ) Oleh: Didit Budi Nugroho, S.Si., M.Si. Program Studi Matematika Fakultas Sains dan Matematika Universitas Kristen Satya Wacana
  • 2. KATA PENGANTAR Diktat ini merupakan catatan kuliah Teori Bilangan (MX 127) tingkat sarjana tahun pertama yang diberikan di Universitas Kristen Satya Wacana dalam semester 1 tahun 2008-2009. Karena itu naskah ini disajikan dalam cara yang sangat dasar (elementer). Elementer berarti hampir tidak ada Analisis yang digunakan, dan hampir tidak ada Aljabar Abstrak. Naskah ini dirancang untuk mencakup beberapa ide dasar teori bilangan dalam satu semester. Selain itu, di sini juga disertakan masalah-masalah teori bilangan yang digunakan dalam berbagai pelatihan dan kompetisi matematika internasional untuk memotivasi dan memberikan tantangan kepada mahasiswa. Penulis berharap bahwa naskah ini akan memberikan manfaat yang lebih dalam pengajaran Teori Bilangan. Untuk itu masih diperlukan masukan dan saran dari pembaca demi perbaikan dan pengembangan naskah ini secara terus menerus. Salatiga, Juli 2009 Didit B. Nugroho i
  • 3. DAFTAR ISI KATA PENGANTAR i DAFTAR ISI ii DAFTAR SINGKATAN iii 1 Aksioma Dasar untuk Z 1 2 Bukti dengan Induksi 3 3 Keterbagian 3.1 Sifat-sifat Keterbagian Elementer . . . . . . . . . . . . . . . . . . . . . . 3.2 Algoritma Pembagian . . . . . . . . . . . . . . . . . . . . . . . . . . . . 3.3 Beberapa Identitas Aljabar . . . . . . . . . . . . . . . . . . . . . . . . . 6 6 12 14 4 Kongruensi Zn 4.1 Kongruensi . . . . . . 4.2 Persamaan Kongruensi 4.3 Uji Keterbagian . . . . 4.4 Sisa lengkap . . . . . . . . . . 18 18 22 25 25 5 Faktorisasi Tunggal 5.1 FPB dan KPK . . . . . . . . . . . . . . . . . . . . . . . . . . . . . . . . 5.2 Bilangan Prima dan Faktorisasi . . . . . . . . . . . . . . . . . . . . . . . 5.3 Teorema Fermat dan Teorema Euler . . . . . . . . . . . . . . . . . . . . 27 27 30 34 6 Algoritma Euclid 6.1 Sistem Kongruensi Linear . . . . . . . . . . . . . . . . . . . . . . . . . . 36 41 7 Fungsi-fungsi Bilangan-Teoritik 7.1 Fungsi Floor . . . . . . . . . . 7.2 Fungsi Legendre . . . . . . . . 7.3 Bilangan Fermat . . . . . . . . 7.4 Bilangan Mersenne . . . . . . . 7.5 Bilangan Sempurna . . . . . . . 45 45 50 51 52 53 . . . . . . . . . . . . . . . . . . . . . . . . . . . . . . . . . . . . . . DAFTAR PUSTAKA . . . . . . . . . . . . . . . . . . . . . . . . . . . . . . . . . . . . . . . . . . . . . . . . . . . . . . . . . . . . . . . . . . . . . . . . . . . . . . . . . . . . . . . . . . . . . . . . . . . . . . . . . . . . . . . . . . . . . . . . . . . . . . . . . . . . . . . . . . . . . . . . . . . . . . . . . . . . . . . . . . . . . . . . . . . . . . . . . . . . . . . . . 54 ii
  • 4. DAFTAR SINGKATAN USAMO IMO HMMT AHSME UMMC SMC AIME Putnam ARML APMC : : : : : : : : : : United States of America Mathematical Olympiad International Mathematical Olympiad Harvard– MIT Math Tournament American High School Mathematics Examination University of Michigan Mathematics Competition Stanford Mathematics Competition American Invitational Mathematics Examination The William Lowell Putnam Mathematical Competition American Regional Mathematics League Austrian– Polish Mathematics Competition iii
  • 5. Bab 1 Aksioma Dasar untuk Z Perhatian dalam teori bilangan yaitu pada sifat-sifat bilangan bulat : : : ; 4; 3; 2; 1; 0; 1; 2; 3; 4; : : : Karena itu pertama kali diperkenalkan beberapa notasi dan mengingat kembali beberapa sifat dasar dari bilangan bulat yang akan diperlukan pada bahasan-bahasan selanjutnya: N = f1; 2; 3; :::g (himpunan semua bilangan asli atau bulat positif) Z = f:::; 3; 2; 1; 0; 1; 2; 3; :::g (himpunan semua bilangan bulat) nn o Q = : n; m 2 Z dan m 6= 0 (himpunan semua bilangan rasional) m R = himpunan semua bilangan riil Dicatat bahwa N Z Q R. Beberapa aksioma dasar untuk Z: 1. Jika a; b 2 Z, maka a + b, a b, ab 2 Z. (Z dikatakan tertutup terhadap operasi penjumlahan, pengurangan, dan perkalian) 2. Jika a 2 Z, maka tidak ada x 2 Z sedemikian sehingga a < x < a + 1. 3. Jika a; b 2 Z dan ab = 1, maka a = b = 1 atau a = b = 1. 4. Hukum eksponen: Untuk n; m 2 N dan a; b 2 R berlaku: (a) (an )m = anm (b) (ab)n = an bn (c) an am = an+m : Aturan-aturan di atas berlaku untuk semua n; m 2 Z jika a; b 6= 0. 5. Sifat ketaksamaan: Untuk a; b; c 2 R berlaku: (a) Transitif : Jika a < b dan b < c, maka a < c. (b) Jika a < b maka a + c < b + c. (c) Jika a < b dan 0 < c maka ac < bc. 1
  • 6. 2 Bab 1. Aksioma Dasar untuk Z (d) Jika a < b dan c < 0 maka bc < ac. (e) Trikotomi : Diberikan a dan b, hanya berlaku salah satu dari: a = b, a < b, b < a. 6. Sifat terurut baik (well-ordering ) untuk N: setiap himpunan bagian tak kosong dari N memuat suatu elemen terkecil atau minimal. Suatu elemen terkecil dari suatu himpunan bagian S N adalah suatu elemen s0 2 S dimana s0 s untuk setiap s 2 S. 7. Prinsip Induksi Matematis: Diambil P (n) sebagai suatu pernyataan menyangkut variabel bilangan asli n. Diambil n0 adalah suatu bilangan asli. P (n) adalah benar untuk semua bilangan asli n n0 jika kedua pernyataan berikut ini berlaku: PIM(a) P (n) benar untuk n = n0 . PIM(b) Jika P (n) benar untuk n0 n k, maka P (n) benar untuk n = k + 1.
  • 7. Bab 2 Bukti dengan Induksi Pada bab ini diberikan beberapa pernyataan yang dapat dibuktikan dengan menggunakan Prinsip Induksi Matematis, atau secara sederhana disebut induksi. Berikut ini diberikan suatu pernyataan beserta bukti induksi. Proposisi 2.1 Jika n 5 maka 2n > 5n. Bukti. Di sini digunakan Prinsip Induksi Matematis. PIM(a) Diambil P (n) adalah pernyataan 2n > 5n. Untuk n0 diambil 5. Secara sederhana dapat dituliskan: P (n) : 2n > 5n dan n0 = 5: Sekarang jika n = 4 maka P (n) menjadi pernyataan 24 > 5 4 yang adalah salah. Tetapi jika n = 5 , P (n) adalah pernyataan 25 > 5 5 atau 32 > 25 yang adalah benar. Jadi P (n) benar untuk n = 5. PIM(b) Diasumsikan bahwa P (k) benar untuk suatu bilangan bulat positif k Artinya, diasumsikan bahwa 2k > 5k untuk suatu k 2 N dan k 5: 5. (2.1) Asumsi (2.1) dinamakan hipotesis induksi dan akan digunakan untuk membuktikan bahwa P (n) benar ketika n = k +1. Atau dengan kata lain akan dibuktikan bahwa 2k+1 > 5 (k + 1) (2.2) dan dilakukan seperti berikut ini. Berdasarkan (2.1), ruas kiri dari (2.2) dapat dituliskan sebagai 2k+1 = 2 2k > 2 5k = 10k; dan karena 5k > 5 untuk setiap k 5, maka 10k = 5k + 5k > 5k + 5 = 5 (k + 1), sehingga 2k+1 > 10k > 5 (k + 1) : yang berarti bahwa P (n) benar ketika n = k + 1. Disimpulkan bahwa P (n) berlaku untuk n 3 5:
  • 8. 4 Bab 2. Bukti dengan Induksi Contoh 2.2 (USAMO 1978) Suatu bilangan bulat dikatakan bagus (good) jika dapat dituliskan sebagai n = a1 + a2 + + ak ; dimana a1 ; a2 ; memenuhi ; ak adalah bilangan-bilangan bulat positif (tidak perlu berbeda) yang 1 1 + + a1 a2 + 1 = 1: ak Diberikan informasi bahwa bilangan-bilangan bulat 33 sampai 73 adalah bagus, buktikan bahwa setiap bilangan bulat 33 adalah bagus. Bukti. Diambil n = 33, maka bilangan-bilangan bulat 33 sampai 73 dapat dituliskan sebagai barisan n; n + 1; n + 2; :::; 2n + 7 yang adalah bagus berdasarkan yang diketahui. Akan dibuktikan bahwa 2n + 8 dan 2n + 9 adalah bagus. Karena n adalah bagus, maka dapat dituliskan 2n + 8 = 2 (a1 + a2 + = 2a1 + 2a2 + dan + ak ) + 4 + 4 + 2ak + 4 + 4 1 1 + + 2a1 2a2 + 1 1 1 1 1 1 + + = + + = 1: 2ak 4 4 2 4 4 Juga, 2n + 9 = 2a1 + 2a2 + dan 1 1 + + 2a1 2a2 + + 2ak + 3 + 6 1 1 1 1 1 1 + + = + + = 1: 2ak 3 6 2 3 6 Oleh karena itu, 2n + 8 dan 2n + 9 adalah bagus. Latihan 2.3 Buktikan bahwa 2n > 6n untuk n 5. n (n + 1) untuk n 1. 2 Latihan 2.5 Buktikan bahwa jika 0 < a < b maka 0 < an < bn untuk setiap n 2 N. Latihan 2.4 Buktikan bahwa 1 + 2 + +n= Latihan 2.6 Buktikan bahwa n! < nn untuk n 2. Latihan 2.7 Buktikan bahwa jika a; r 2 R dan r 6= 1, maka untuk n a + ar + ar2 + + arn = a rn+1 1 : r 1 Ini dapat dituliskan seperti a rn+1 1 = (r 1) a + ar + ar2 + + arn : Dan kasus khusus pentingnya adalah rn+1 1 = (r 1) 1 + r + r2 + + rn : 1 berlaku
  • 9. 5 Bab 2. Bukti dengan Induksi Latihan 2.8 Buktikan bahwa 1 + 2 + 22 + Latihan 2.9 Buktikan bahwa 111{z 1 = | } n kali + 2n = 2n+1 10n 1 untuk n 9 Latihan 2.10 Buktikan bahwa 12 +22 +32 + +n2 = 1 untuk n 1. 1. n (n + 1) (2n + 1) untuk n 6 1.
  • 10. Bab 3 Keterbagian 3.1 Sifat-sifat Keterbagian Elementer Pertama kali diperkenalkan pernyataan d j n yang dapat dibaca seperti berikut ini: 1. d membagi n. 2. d adalah pembagi dari n. 3. d adalah suatu faktor dari n. 4. n adalah kelipatan dari n. Jadi, lima pernyataan di bawah ini adalah ekivalen, artinya semua cara yang berbeda mengatakan hal yang sama. 1. 2 j 6: 2. 2 membagi 6. 3. 2 adalah pembagi dari 6. 4. 2 adalah suatu faktor dari 6. 5. 6 adalah kelipatan dari 2. De…nisi 3.1 d j n berarti terdapat suatu bilangan bulat k sedemikian sehingga n = dk, a sedangkan d - n berarti bahwa d j n adalah salah. Dicatat bahwa a j b 6= . b Suatu cara lain untuk menyatakan de…nisi dari d j n adalah seperti berikut ini. De…nisi 3.2 d j n () n = dk untuk suatu k. (Dicatat bahwa notasi () diinterpretasikan dengan arti jika dan hanya jika.) Teorema 3.3 (Sifat-sifat keterbagian) Jika n, m, dan d adalah bilangan-bilangan bulat maka pernyataan-pernyataan berikut ini adalah benar: (1) n j n (sifat re‡eksif ); (2) d j n dan n j m =) d j m (sifat transitif ); 6
  • 11. 7 Bab 3. Keterbagian (3) d j n dan d j m =) d j an + bm untuk setiap bilangan bulat a dan b (sifat linier); (4) d j n =) ad j an untuk a 6= 0 (sifat perkalian); (5) ad j an dan a 6= 0 =) d j n (sifat penghapusan); (6) 1 j n (1 membagi sembarang bilangan); (7) n j 1 =) n = 1 (hanya 1 dan 1 yang merupakan pembagi dari 1); (8) d j 0 (sembarang nilai membagi nol); (9) 0 j n =) n = 0 (nol hanya membagi nol); (10) d, n adalah positif dan d j n =) d n (sifat perbandingan). (11) d j n dan d j (n + m) =) d j m. Contoh 3.4 Buktikan sifat 1 sampai 10 dalam Teorema 3.3. Bukti. Untuk (1), dicatat bahwa n = 1 n. Untuk (2) sampai (5), (10) dan (11), syarat d j n diberikan, artinya n = kd untuk suatu bilangan bulat k. Untuk (2), dipunyai n j m, artinya m = k1 n, maka m = (k1 k) d atau d j m. Untuk (3), diasumsikan bahwa m = k2 d, maka an + bm = (ka + k2 b) d. Untuk (4) dan (5), karena a 6= 0, d 6= 0 jika dan hanya jika ad 6= 0. Dicatat bahwa n = kd jika dan hanya jika na = kda. Untuk (6), dicatat bahwa n = n 1. Untuk (7), dicatat bahwa 1 = 1 1 atau 1 = ( 1) ( 1). Untuk (8), dicatat bahwa 0 = d 0. Untuk (9), dipunyai 0 j n, artinya n = 0 k, maka n = 0. Untuk (10), dicatat bahwa d; n > 0, maka jkj 1 dan juga n = jkj d d. Untuk (11), dipunyai d j (n + m), artinya n + m = k1 d, maka kd + m = k1 d atau m = (k1 k) d atau d j m. De…nisi 3.5 Jika c = as + bt untuk suatu bilangan bulat s dan t, dikatakan bahwa c merupakan suatu kombinasi linier dari a dan b. Jadi, pernyataan (3) dalam Teorema 3.3 mengatakan bahwa jika d membagi a dan b, maka d membagi semua kombinasi linear dari a dan b. Khususnya, d membagi a + b dan a b. Contoh 3.6 Diambil x dan y adalah bilangan bulat. Buktikan bahwa 2x + 3y dapat dibagi oleh 17 jika dan hanya jika 9x + 5y dapat dibagi oleh 17. Bukti. 17 j (2x + 3y) =) 17 j 13 (2x + 3y) atau 17 j (26x + 39y) =) 17 j [(9x + 5y) + (17x + 34y)] =) 17 j (9x + 5y), dan sebaliknya, 17 j (9x + 5y) =) 17 j 4 (9x + 5y) atau 17 j (36x + 20y) =) 17 j [(2x + 3y) + (34x + 17y)] =) 17 j (2x + 3y). Contoh 3.7 Tentukan semua bilangan bulat positif d sedemikian sehingga d membagi n2 + 1 dan (n + 1)2 + 1 untuk suatu bilangan bulat n. h i Penyelesaian. Diambil d j n2 + 1 dan d j (n + 1)2 + 1 atau d j n2 + 2n + 2 . Jadi d j n2 + 2n + 2 n2 + 1 atau d j (2n + 1) =) d j 4n2 + 4n + 1 , sehingga 2 + 2n + 2 2 + 4n + 1 dj 4 n 4n atau d j (4n + 7) : Jadi d j [(4n + 7) 2 (2n + 1)] atau d j 5. Disimpulkan bahwa d adalah 1 atau 5. (Dapat ditunjukkan bahwa nilai dicapai dengan mengambil n = 2:)
  • 12. 8 Bab 3. Keterbagian Contoh 3.8 Buktikan bahwa 33n+3 untuk semua bilangan asli n. 26n 27 merupakan suatu kelipatan dari 169 Bukti. Di sini digunakan Prinsip Induksi Matematis. PIM(a) Diambil P (n) adalah pernyataan 33n+3 26n 27 = 169 n, n 2 N. Untuk n = 1, kita menyatakan bahwa 36 53 = 676 = 169 4 yang berarti dapat dibagi oleh 169. Jadi P (n) benar untuk n = 1. PIM(b) Diasumsikan bahwa pernyataan benar untuk n = k 33k 26k 1, k > 1, yaitu 1 = 169N untuk suatu bilangan bulat N . Karena itu 33k+3 26k 27 = 27 33k 26k 27 = 27 33k 26k 1 + 676k yang direduksi menjadi 27 169N + 169 4k, yang dapat dibagi oleh 169, yang berarti bahwa P (n) benar untuk n = k. Contoh 3.9 (IMO 1984) Diandaikan bahwa a1 , a2 , :::, a2n adalah bilangan-bilangan bulat berbeda sedemikian sehingga persamaan (x a1 ) (x a2 ) (x ( 1)n (n!)2 = 0 a2n ) mempunyai suatu penyelesaian bilangan bulat r. Tunjukkan bahwa r= a1 + a2 + ::: + a2n : 2n Bukti. Jelas r 6= ai untuk semua i, dan r sehingga j(r a1 ) (r a2 ) (r a2n )j ai adalah 2n bilangan bulat berbeda, j(1) (2) (n) ( 1) ( 2) ( n)j = (n!)2 ; dengan kesamaan terjadi jika dan hanya jika fr a1 ; r a2 ; ;r a2n g = f1; 2; ; n; 1; 2; ; ng : Oleh karena itu (r a1 ) + (r a2 ) + + (r a2n ) = 2nr (a1 + a2 + ::: + a2n ) = 1+2+ + n + ( 1) + ( 2) + = 0 yang mengakibatkan r= a1 + a2 + ::: + a2n : 2n + ( n)
  • 13. 9 Bab 3. Keterbagian Himpunan bilangan bulat dapat dipartisi menjadi dua himpunan bagian, yaitu himpunan bilangan bulat ganjil dan himpunan bilangan genap: f 1; 3; 5; :::g dan f0; 2; 4; :::g berturut-turut. Berikut ini diberikan beberapa ide dasar: (1) suatu bilangan ganjil mempunyai bentuk 2k + 1, untuk suatu bilangan bulat k; (2) suatu bilangan genap mempunyai bentuk 2k, untuk suatu bilangan bulat k; (3) jumlahan dari dua bilangan ganjil adalah suatu bilangan genap; (4) jumlahan dari dua bilangan genap adalah suatu bilangan genap; (5) jumlahan dari suatu bilangan ganjil dan genap adalah suatu bilangan ganjil; (6) hasil kali dari dua bilangan ganjil adalah suatu bilangan ganjil; (7) hasil kali dari bilangan-bilangan bulat adalah genap jika dan hanya jika paling sedikit dari faktor-faktornya adalah genap. Contoh 3.10 Diambil n adalah suatu bilangan bulat yang lebih besar dari 1. Buktikan bahwa (a) 2n adalah jumlahan dari dua bilangan bulat ganjil berturutan. (b) 3n adalah jumlahan dari tiga bilangan bulat berturutan. Bukti. Untuk (a), dari hubungan 2n = 22+n 2 = 4 2n 2 dimisalkan k = 2n 2 , sehingga karena 4k = (2k 1) + (2k + 1) maka diperoleh 2n = 4k = 2n 1 1 + 2n 1 + 1 . Untuk (b), dari hubungan 3n = 31+n 1 = 3 3n 1 dimisalkan s = 3n 1 , sehingga karena 3s = (s 1) + s + (s + 1) maka diperoleh 3n = 3s = 3n 1 1 + 3n 1 + 3n 1 + 1 . Contoh 3.11 Diambil k adalah suatu bilangan genap. Apakah mungkin untuk menuliskan 1 sebagai jumlahan dari kebalikan k bilangan ganjil? Penyelesaian. Diasumsikan bahwa 1= 1 + n1 + 1 nk untuk bilangan-bilangan ganjil n1 , ..., nk ; maka diperoleh n1 nk = s1 + + sk , dimana semua si adalah ganjil. Ini tidaklah mungkin terjadi karena ruas kiri adalah ganjil dan ruas kanan adalah genap. Contoh 3.12 (HMMT 2004) Andi memilih lima bilangan dari himpunan f1, 2, 3, 4, 5, 6, 7g. Selanjutnya Andi memberitahu Vian berapa hasil kali dari bilangan-bilangan terpilih tersebut, yang tidak akan menjadi informasi yang cukup bagi Vian untuk membayangkan apakah jumlahan dari bilangan-bilangan terpilih adalah genap atau ganjil. Berapa hasil kali dari bilangan-bilangan terpilih tersebut?
  • 14. 10 Bab 3. Keterbagian Penyelesaian. Mencari hasil kali dari bilangan-bilangan terpilih ekivalen dengan mengetahui hasil kali dari dua bilangan yang tidak terpilih. Hasil kali dari bilanganbilangan tidak terpilih dan mungkin diperoleh dari lebih satu pasangan bilangan yaitu hanya 12 ({3,4} dan {2,6}) dan 6 ({1,6} dan {2,3}). Tetapi dalam kemungkinan kedua, jumlahan dari dua bilangan (tidak terpilih) adalah ganjil (dan juga lima bilangan terpilih mempunyai jumlahan ganjil), sehingga belum tentu benar. Oleh karena itu kemungkinan pertama pasti benar, dan hasil kali dari lima bilangan terpilih sama dengan 1 2 3 12 7 = 420: Contoh 3.13 Buktikan bahwa 1+ p 2n 2 p + 1 2n 2 adalah suatu bilangan bulat genap dan bahwa 1+ p 2 2n p 1 2n 2 p =b 2 untuk suatu bilangan bulat positif b, untuk semua bilangan bulat n 1. Bukti. Diproses dengan induksi pada n 2 N. p 2n p 2n 2 PIM(a) Diambil P (n) adalah pernyataan: 1 + 2 + 1 adalah genap dan p 2n p p 2n 1 2 = b 2 untuk suatu b 2 N. Untuk n = 1, dipunyai 1+ 2 1+ p 2 2 + 1 p 2 2 =6 yang adalah bilangan genap, dan 1+ p 2 2 1 p 2 2 p = 4 2: Oleh karena itu P (1) adalah benar. PIM(b) Diasumsikan bahwa P (n) benar untuk n = k 1, k > 1, yaitu diasumsikan bahwa p 2(k 1) p 2(k 1) 1+ 2 + 1 2 = 2N untuk suatu bilangan bulat N dan bahwa 1+ p 2 2(k 1) untuk suatu bilangan bulat positif a. 1 p 2 2(k 1) p =a 2
  • 15. 11 Bab 3. Keterbagian Sekarang diperhatikan 1+ p 2k 2 + 1 p 2 2k = 1+ + 1 p 2 2 p 2 p 3+2 2 = + 3 = 3 p 2 2 p p p 1 2(k 1) 2k 2 2 1 1+ p 2 2 1+ 1+ 2 2 p 2k 2 2(k 1) 2 2(k 1) + 1 p 2 2(k 1) p p 2(k 1) 1 2 1+ 2 p p = 6N + 2 2 a 2 = 2 (3N + 2a) p 2 2 + 2(k 1) yang merupakan suatu bilangan bulat genap dan secara serupa 1+ p 2 2k 1 p 2 2k p p p = 3a 2 + 2 2 (2N ) = (3a + 4N ) 2: Jadi P (k) adalah benar. Contoh 3.14 (USAMO 2003) Buktikan bahwa untuk setiap bilangan bulat positif n terdapat suatu bilangan n-digit yang dapat dibagi oleh 5n dimana semua digit-nya ganjil. Bukti. Diproses dengan induksi pada n 2 N. PIM(a) Pernyataan adalah benar untuk n = 1 karena terdapat bilangan satu digit yang dapat dibagi oleh 5, yaitu 5. PIM(b) Diasumsikan bahwa N = a1 a2 :::an dapat dibagi oleh 5n dan hanya mempunyai digit-digit ganjil. Diperhatikan bilangan-bilangan N1 = 1a1 a2 :::an = 1 10n + 5n M = 5n (1 2n + M ) ; N2 = 3a1 a2 :::an = 3 10n + 5n M = 5n (3 2n + M ) ; N3 = 5a1 a2 :::an = 5 10n + 5n M = 5n (5 2n + M ) ; N4 = 7a1 a2 :::an = 7 10n + 5n M = 5n (7 2n + M ) ; N5 = 9a1 a2 :::an = 9 10n + 5n M = 5n (9 2n + M ) : Diperhatikan dua kemungkinan. Kemungkinan pertama, bilangan-bilangan 1 2n +M , 3 2n +M , 5 2n +M , 7 2n +M , 9 2n +M memberikan sisa-sisa yang berbeda ketika dibagi oleh 5. Kemungkinan kedua, beda dari dua diantaranya merupakan kelipatan dari 5, yang adalah tidak mungkin karena 2n bukan kelipatan dari 5 dan bukan beda dari sembarang bilangan-bilangan 1, 3, 5, 7, 9. Karena itu yang benar adalah kemungkinan pertama, dan ini berarti bahwa satu di antara bilangan-bilangan N1 , N2 , N3 , N4 , N5 dapat dibagi oleh 5n 5.
  • 16. 12 Bab 3. Keterbagian Latihan 3.15 Buktikan bahwa jika d j a dan d j b maka d j a Latihan 3.16 Buktikan bahwa jika d j a dan d j a b. b maka d j b. Latihan 3.17 Buktikan bahwa jika d j n dan n 6= 0 maka jdj jnj. Latihan 3.18 Buktikan bahwa jika d j n dan n j d maka jdj = jnj. Latihan 3.19 Buktikan bahwa jika d j n dan n 6= 0 maka n j n. d Latihan 3.20 Buktikan bahwa jika a 2 Z maka pembagi positif dari a dan a + 1 hanya 1: Latihan 3.21 Diambil a dan b adalah bilangan bulat positif sedemikian sehingga a j b2 , b2 j a3 , a3 j b4 , b4 j a5 , ... . Buktikan bahwa a = b. 3.2 Algoritma Pembagian Tujuan dari bagian ini adalah membuktikan hasil penting berikut ini. Teorema 3.22 (Algoritma Pembagian) Jika a dan b adalah bilangan bulat dan b > 0 maka terdapat secara tunggal bilangan bulat q dan r yang memenuhi dua kondisi: a = bq + r dan 0 r < b. (3.1) Dalam situasi ini q dinamakan hasil bagi (quotient) dan r dinamakan sisa (remainder ) ketika a dibagi oleh b. Dicatat bahwa terdapat dua bagian untuk hasil ini. Satu bagian adalah EKSISTENSI dari bilangan bulat q dan r yang memenuhi (3.1) dan bagian kedua adalah KETUNGGALAN dari bilangan bulat q dan r yang memenuhi (3.1). Bukti. Pertama kali diperkenalkan fungsi ‡oor : bxc = bilangan bulat terbesar yang lebih kecil atau sama dengan x dimana x adalah sembarang bilangan riil. Dipunyai sifat bahwa x Bahasan lebih lanjut mengenai fungsi ‡oor diberikan dalam Bab 7. Sekarang diambil b > 0 dan sembarang a mende…nisikan jak q = b r = a bq: 1 < bxc Secara jelas dipunyai a = bq+r. Tetapi kita perlu untuk membuktikan bahwa 0 Berdasarkan sifat fungsi ‡oor dipunyai jak a a 1< : b b b Sekarang dikalikan semua suku dari ketaksamaan dengan jak b a> b a: b x. r < b. b yang akan menghasilkan
  • 17. 13 Bab 3. Keterbagian Jika ditambahkan a ke semua ruas dari ketaksamaan dan diperoleh b > a bq 0: jak b diganti dengan q maka Karena r = a bq, maka persamaan terakhir memberikan hasil 0 r < b. Kita tetap harus membuktikan bahwa q dan r ditentukan secara tunggal. Untuk itu diasumsikan bahwa a = bq1 + r1 dan 0 r1 < b; dan a = bq2 + r2 dan 0 r2 < b: Kita harus menunjukkan bahwa r1 = r2 dan q1 = q2 . Jika r1 6= r2 , tanpa kehilangan keumuman, dapat diasumsikan bahwa r2 > r1 . Pengurangan kedua persamaan di atas akan menghasilkan 0=a a = (bq1 + r1 ) (bq2 + r2 ) = b (q1 q 2 ) + r1 r2 : Ini mengakibatkan r2 r1 = b (q1 q2 ) : (3.2) Ini berarti bahwa b j (r2 r1 ). Berdasarkan Teorema 3.3 nomor (10), ini mengakibatkan bahwa b r2 r1 . Tetapi karena 0 r1 < r2 < b maka dipunyai r2 r1 < b. Ini kontradiksi dengan b r2 r1 . Jadi kita harus menyimpulkan bahwa r1 = r2 . Sekarang dari (3.2) kita mempunyai 0 = b (q1 q2 ). Karena b > 0, akibatnya q1 q2 = 0, artinya q1 = q2 . Ini melengkapi bukti ketunggalan dari q dan r. Contoh 3.23 (AHSME 1976) Diambil r adalah sisa ketika 1059, 1417 dan 2312 dibagi oleh b > 1. Tentukan nilai dari b r. Penyelesaian. Berdasarkan Algoritma Pembagian, 1059 = q1 b+r, 1417 = q2 b+r, dan 2312 = q3 b + r untuk bilangan-bilangan bulat q1 , q2 , q3 . Dari sini, 358 = 1417 1059 = b (q2 q1 ), 1253 = 2312 1059 = b (q3 q1 ), dan 895 = 2312 1417 = b (q3 q2 ). Karena itu b j 358 atau b j 2 179, b j 1253 atau b j 7 179, dan b j 895 atau b j 5 179. Karena b > 1, disimpulkan bahwa b = 179. Jadi (sebagai contoh) 1059 = 5 179 + 164, yang berarti bahwa r = 164. Disimpulkan bahwa b r = 179 164 = 15. Contoh 3.24 Tunjukkan bahwa n2 + 23 dapat dibagi oleh 24 untuk n tak hingga banyaknya. Bukti. n2 + 23 = n2 1 + 24 = (n 1) (n + 1) + 24. Jika diambil n = 24k k = 0; 1; 2; :::, maka pernyataan dapat dibagi oleh 24. 1, De…nisi 3.25 Suatu bilangan prima (prime) p adalah bilangan bulat positif lebih besar 1 yang pembagi positifnya hanya 1 dan p. Jika bilangan bulat n > 1 bukan prima, maka bilangan tersebut dinamakan bilangan composite. Dicatat bahwa bilangan 1 bukan bilangan prima ataupun composite.
  • 18. 14 Bab 3. Keterbagian Contoh 3.26 Tunjukkan bahwa jika p > 3 adalah prima, maka 24 j p2 1 . Bukti. Berdasarkan Algoritma Pembagian, sembarang bilangan bulat dapat dinyatakan sebagai salah satu dari: 6k, 6k 1, 6k 2, atau 6k + 3, dengan k 2 Z: Jika p > 3 adalah prima, maka p mempunyai bentuk p = 6k 1 (karena pilihan lainnya dapat dibagi 2 atau 3). Dicatat bahwa (6k 1)2 1 = 36k 2 12k = 12k (3k 1). Karena k atau 3k 1 adalah genap, maka 12k (3k 1) dapat dibagi oleh 24. Contoh 3.27 Buktikan bahwa kuadrat dari sembarang bilangan mempunyai bentuk 4k atau 4k + 1. Bukti. Berdasarkan Algoritma Pembagian, sembarang bilangan bulat dapat dinyatakan sebagai salah satu dari: 2a atau 2a + 1. Dikuadratkan, (2a)2 = 4a2 ; (2a + 1)2 = 4 a2 + a + 1; sehingga pernyataan adalah benar. Contoh 3.28 Buktikan bahwa tidak ada bilangan bulat dalam barisan 11; 111; 1111; 11111; ::: (3.3) yang merupakan kuadrat dari suatu bilangan bulat. Bukti. Sudah diperoleh bahwa kuadrat dari sembarang bilangan bulat mempunyai bentuk 4k atau 4k + 1. Semua bilangan dalam barisan (3.3) mempunyai bentuk 4k 1, yang berarti tidak bisa menjadi kuadrat dari sembarang bilangan bulat. Contoh 3.29 Tunjukkan bahwa dari sembarang tiga bilangan bulat, selalu dapat dipilih dua diantaranya, misalnya a dan b, sehingga a3 b ab3 dapat dibagi 10. Bukti. Jelas bahwa a3 b ab3 = ab (a b) (a + b) selalu genap. Jika satu dari tiga bilangan bulat mempunyai bentuk 5k, maka selesai (misalnya diambil a = 5k). Jika tidak, dipilih tiga bilangan yang terletak dalam klas-klas sisa 5k 1 atau 5k 2. Dua dari tiga bilangan bulat pasti terletak di salah satu dari dua kelompok tersebut. Akibatnya jumlah atau selisih dari dua bilangan tersebut berbentuk 5k dan diperoleh hasil yang diinginkan. Contoh 3.30 Buktikan bahwa jika 3 j a2 + b2 , maka 3 j a dan 3 j b. Bukti. Dibuktikan dengan kontraposisi seperti berikut ini. Diandaikan bahwa 3 - a atau 3 - b, dan akan dibuktikan bahwa 3 - a2 + b2 . Dari hipotesis dapat dinyatakan a = 3k 1 atau b = 3m 1. Dari sini diperoleh a2 = 3 3k 2 2k + 1 atau a2 = 3x + 1, dan serupa dengan itu b2 = 3y + 1. Jadi a2 + b2 = 3x + 1 + 3y + 1 = 3s + 2, yang berarti 3 - a2 + b2 . 3.3 Beberapa Identitas Aljabar Pada bagian ini diberikan beberapa contoh dimana penyelesaiannya tergantung pada penggunaan beberapa identitas aljabar elementer.
  • 19. 15 Bab 3. Keterbagian Contoh 3.31 Tentukan semua bilangan prima berbentuk n3 n > 1. 1, untuk bilangan bulat Penyelesaian. n3 1 = (n 1) n2 + n + 1 . Jika pernyataan ini merupakan bilangan prima, karena n2 +n+1 > 1, pasti dipunyai n 1 = 1, yang berarti n = 2. Jadi bilangan prima yang dimaksud hanyalah 7. Contoh 3.32 Buktikan bahwa n4 + 4, n 2 N, adalah prima hanya untuk n = 1. Bukti. Diamati bahwa n4 + 4 = n4 + 4n2 + 4 2 4n2 (2n)2 = n2 + 2 = n2 + 2n + 2 = (n + 1)2 + 1 n2 2n + 2 1)2 + 1 : (n Setiap faktor lebih besar 1 untuk n > 1, akibatnya n4 + 4 bukanlah prima. 1 yang memenuhi n4 + 4n adalah Contoh 3.33 Tentukan semua bilangan bulat n prima. Penyelesaian. Untuk n = 1, jelas bahwa pernyataan merupakan bilangan prima. Lebih lanjut haruslah diambil n adalah ganjil. Untuk n 3, semua bilangan di bawah ini adalah bulat: n4 + 22n = n4 + 2n2 2n + 22n 2 2n2 2n n 2 + 2n = n2 + 2n + n2 2 (n+1) n2 2 (n+1) 1 Ini mudah dilihat bahwa jika n tersebut bukan prima. 2 1 = n 2 + 2n 1 n2 2 (n+1) : 3, maka setiap faktor lebih besar 1, sehingga bilangan Contoh 3.34 Buktikan bahwa untuk semua n 2 N, n2 membagi (n + 1)2 1. Bukti. Jika n = 1, maka pernyataan benar. Sekarang diandaikan n > 1, maka berdasarkan Teorema Binomial, (n + 1)2 1= n X n nk ; k k=1 dan setiap sukunya dapat dibagi oleh n2 : Contoh 3.35 Buktikan bahwa xn Jadi x y n = (x y) xn y selalu membagi xn 1 + xn yn. 2 y + xn 3 2 y + + xy n 2 + yn 1 :
  • 20. 16 Bab 3. Keterbagian Bukti. Diasumsikan bahwa x 6= y dan xy 6= 0. Dalam kasus ini, hasil di atas mengikuti identitas n 1 X an 1 ak = a 6= 1; a 1 k=0 x pada pengambilan a = dan dikalikan dengan y n . y Sebagai contoh, tanpa penghitungan dapat dilihat bahwa 87672345 dibagi 666: 81012345 dapat Contoh 3.36 (E½ tv½ s 1899) Tunjukkan bahwa o o 2903n 803n 464n + 261n dapat dibagi oleh 1897 unuk semua bilangan asli n. Bukti. Berdasarkan hasil sebelumnya, 2903n 803n dapat dibagi oleh 2903 803 = 2100 = 7 300, dan 261n 464n dapat dibagi oleh 261 464 = 203 = 7 ( 29). Jadi pernyataan 2903n 803n 464n + 261n dapat dibagi oleh 7. Dan juga, 2903n 464n dapat dibagi oleh 2903 464 = 9 271 dan 261n 803n dapat dibagi oleh 261 803 = 542 = ( 2) 271. Jadi pernyataan juga dapat dibagi oleh 271. Karena 7 dan 271 tidak mempunyai faktor prima yang sama (kecuali 1), maka disimpulkan bahwa pernyataan dapat dibagi oleh 7 271 = 1897. Contoh 3.37 (UMMC 1987) Diberikan bahwa 1002004008016032 mempunyai suatu faktor prima p > 250000, cari bilangan tersebut. Penyelesaian. Jika a = 103 dan b = 2, maka 1002004008016032 = a5 + a4 b + a3 b2 + a2 b3 + ab4 + b5 = a6 a b6 : b Pernyataan terakhir dinyatakan sebagai a6 a b6 b = (a + b) a2 + ab + b2 a2 ab + b2 = 1002 1002004 998004 = 4 4 1002 250501 k; dimana k < 250000. Oleh karena itu p = 250501. Contoh 3.38 (Grünert 1856) Jika x; y; z; n 2 N dan n hubungan xn + y n = z n : z, maka tidak berlaku Bukti. Jelas bahwa jika berlaku hubungan xn + y n = z n untuk x; y; z 2 N, maka x < z dan y < z. Berdasarkan kesimetrian, bisa diandaikan bahwa x < y. Selanjutnya pernyataan dibuktikan dengan kontraposisi. Jadi diandaikan bahwa xn + y n = z n dan n z, maka zn y n = (z y) z n 1 + yz n 2 + + yn yang bertentangan dengan pernyataan xn + y n = z n . 1 1 nxn 1 > xn
  • 21. 17 Bab 3. Keterbagian Contoh 3.39 Buktikan bahwa untuk n ganjil, xn + y n = (x + y) xn 1 xn 2 y + xn 3 2 y + + xy n 2 + yn 1 : Jadi jika n adalah ganjil, maka x + y membagi xn + y n . Bukti. Bukti diperoleh dengan pensubstitusian y untuk y dalam Contoh 3.35 dan diperhatikan bahwa ( y)n = y n untuk n ganjil. Contoh 3.40 Tunjukkan bahwa 1001 membagi 11993 + 21993 + 31993 + + 10001993 : Penyelesaian. Berdasarkan contoh sebelumnya karena setiap 11993 + 10001993 , 21993 + 9991993 , ..., 5001993 + 50011993 dapat dibagi oleh 1001. Contoh 3.41 (SMC 250) Tunjukkan bahwa untuk sembarang bilangan asli n, terdapat bilangan asli lain x sedemikian sehingga setiap suku dari barisan x x + 1; xx + 1; xx + 1; ::: dapat dibagi oleh n. Bukti. Cukup diambil x = 2n 1.
  • 22. Bab 4 Kongruensi Zn 4.1 Kongruensi De…nisi 4.1 Diambil n 2 N. Untuk x; y 2 Z, x dikatakan kongruen dengan y modulo n jika n j (y x) dan dituliskan x = y (mod n) atau . Selanjutnya y dinamakan sisa n dari x ketika dibagi oleh n. Lebih lanjut, modulo n adalah relasi ekivalensi pada Z, artinya untuk x; y; z 2 Z berlaku: 1. (re‡eksif ) x = x (mod n), 2. (simetris) x = y (mod n) =) y = x (mod n); 3. (transitif ) x = y (mod n) dan y = z (mod n) =) x = z (mod n). Klas kongruensi dari suatu bilangan bulat x modulo n, dinotasikan dengan xn (seringkali juga digunakan notasi x atau [x]n ), adalah himpunan dari semua bilangan bulat yang kongruen dengan x modulo n. Dengan kata lain, menggunakan de…nisi keterbagian, xn = fy 2 Z : n j (y = fy 2 Z : y x)g x = kn; k 2 Zg = fx + kn : k 2 Zg . Himpunan dari klas-klas kongruensi dinotasikan dengan Zn . Elemen-elemen berbeda dari Zn biasanya didaftar seperti 0n ; 1n ; 2n ; :::; (n 1)n : Contoh 4.2 Diambil n = 4, maka dipunyai klas-klas kongruensi dari Zn : 08 = f0 + 4k : k 2 Zg = f0; 4; 8; :::g ; 14 = f1 + 4k : k 2 Zg = f:::; 7; 3; 1; 5; 9; :::g ; 24 = f2 + 4k : k 2 Zg = f:::; 6; 2; 2; 6; 10; :::g ; 34 = f3 + 4k : k 2 Zg = f:::; 5; 1; 3; 7; 11; :::g : Klas-klas ekivalensi dapat dijumlahkan dan dikalikan menggunakan sifat berikut ini. 18
  • 23. 19 Bab 4. Kongruensi Zn Lemma 4.3 Diambil n 2 N. Pada Zn berlaku rumus xn yn = (x y)n ; xn yn = (xy)n : 0 Bukti. Jika x0 = xn dan yn = yn maka n x0 + y 0 = x + y + x0 0 0 xy = 0 x+ x x + y0 x 0 = xy + y x y 0 y + x0 y+ y x +x y y = x + y (mod n); 0 y0 x y = xy (mod n): Selanjutnya, dengan mengaplikasikan rumus perkalian di atas akan diperoleh sifat bahwa jika a = b (mod n) maka ak = bk . Contoh 4.4 Tentukan sisa ketika 61987 dibagi oleh 37. Penyelesaian. Dapat dituliskan 61987 = 6 61986 = 6 62 62 = 1 (mod 37). Jadi 61987 = 6 = 31 (mod 37). 993 = 6 ( 1)993 karena Contoh 4.5 Cari digit terakhir dalam ekspansi desimal dari 4100 . Penyelesaian. Digit terakhir adalah sisa ketika dibagi oleh 10. Jadi harus dihitung klas kongruensi dari 4100 (mod 10). Dipunyai 42 = 6(mod 10), dan selanjutnya 62 = 6(mod 10). Jadi 4100 = (42 )50 = 650 = 6(mod 10). 7 Contoh 4.6 Cari digit satuan dari 77 : 7 Penyelesaian. Harus dicari 77 (mod 10). Diamati bahwa 72 = 1 (mod 10), 73 = 2 72 7 = 7 = 3 (mod 10), dan 74 = 72 = 1 (mod 10). Selain itu, 72 = 1 (mod 4) 3 dan 77 = 72 7 = 3 (mod 4), yang berarti bahwa terdapat suatu bilangan bulat t sedemikian sehingga 77 = 3 + 4t. Sekarang diperoleh 7 77 = 74t+3 = 74 t 73 = 1t 3 = 3 (mod 10): Jadi digit terakhir adalah 3. Contoh 4.7 (Putnam 1986) Berapakah digit satuan dari 1020000 ? 10100 + 3 Penyelesaian. Dimisalkan a 1020000 10100 + 3 = 3 = 10100 , maka (a 200 3)200 1 X 200 200 = a a a k k ( 3)k k=0 = 199 X k=0 200 199 a k k ( 3)k : (4.1)
  • 24. 20 Bab 4. Kongruensi Zn Karena 200 X k=0 ( 1)k 200 k 3 199 = 0, maka untuk a = 3 (mod 10) persamaan (??) menjadi 199 X k=0 ( 1)k 200 k = 3199 = 3(mod 10): Contoh 4.8 Apakah 4100 dapat dibagi 3? Penyelesaian. Karena 4100 = 1100 = 1(mod 3), berarti 3 j 4100 . Contoh 4.9 Buktikan bahwa 7 membagi 32n+1 + 2n+2 untuk setiap n 2 N. Bukti. Diamati bahwa 32n+1 = 3 9n = 3 2n (mod 7) dan 2n+2 = 4 2n (mod 7). Karena itu 32n+1 + 2n+2 = 7 2n = 0 (mod 7), untuk setiap n 2 N. Contoh 4.10 Buktikan hasil Euler: 641 j 232 + 1 . Bukti. Diamati bahwa 641 = 27 5 + 1 = 24 + 54 . Karena itu 27 5 = 1 (mod 641) dan 54 = 24 (mod 641). Sekarang, 27 5 = 1 (mod 641) menghasilkan 54 228 = 4 5 27 = 1 (mod 641). Kongruensi terakhir dan 54 = 24 (mod 641) menghasilkan 32 = 24 228 = 54 228 = 1 (mod 641), yang berarti bahwa 641 j 232 + 1 . 2 Contoh 4.11 Tentukan bilangan-bilangan kuadrat sempurna di modulo 13: Penyelesaian. Karena r2 = (13 r)2 , maka hanya dipunyai r = 0; 1; :::; 6. Diamati bahwa 02 = 0, 12 = 1, 22 = 4, 32 = 9, 42 = 3, 52 = 12, dan 62 = 10 (mod 13). Jadi kuadrat sempurna di modulo 13 yaitu 0; 1; 4; 9; 3; 12; 10. Contoh 4.12 Buktikan bahwa tidak ada bilangan bulat yang memenuhi x2 5y 2 = 2: Bukti. Jika x2 = 2 + 5y 2 , maka x2 = 2 (mod 5). Tetapi karena 2 bukanlah kuadrat sempurna modulo 5, maka dapat disimpulkan tidak ada bilangan bulat yang memenuhi x2 5y 2 = 2: Contoh 4.13 Buktikan bahwa 7 j 22225555 + 55552222 . Bukti. Diamati bahwa 2222 = 3 (mod 7), 5555 = 4 (mod 7), 35 = 5 (mod 7), dan 42 = 2 = 5 (mod 7). Diperoleh 22225555 + 55552222 = 35555 + 42222 = 35(1111) + 42(1111) = 51111 51111 = 0 (mod 7): Contoh 4.14 Cari bilangan-bilangan bulat n sedemikian sehingga 2n + 27 dapat dibagi oleh 7.
  • 25. 21 Bab 4. Kongruensi Zn Penyelesaian. Diamati bahwa 21 = 2, 22 = 4, 23 = 1, 24 = 2, 25 = 4, 26 = 1 (mod 7) dan juga 23k = 1 (mod 3) untuk semua k 2 N. Karena itu 23k + 27 = 1 + 27 = 0 (mod 7) untuk semua k 2 N. Jadi n = 3k; k 2 N. Contoh 4.15 Tentukan semua bilangan bulat n sedemikian sehingga 13 j 4(n2 + 1). Penyelesaian. Ini adalah ekivalen dengan menyelesaikan kongruensi 4(n2 + 1) = 0(mod 13). Karena faktor persekutuan terkecil dari 4 dan 3 adalah 1, maka kita dapat menghapus 4 untuk mendapatkan n2 = 1(mod 13). Penghitungan kuadrat-kuadrat di modulo 13 memberikan ( 1)2 = 1, ( 2)2 = 4, ( 3)2 = 9, ( 4)2 = 3(mod 13), ( 5)2 = 1(mod 13), dan ( 6)2 = 3(mod 13). Jadi, telah dilakukan penghitungan untuk perwakilan dari setiap klas kongruensi, sehingga jawaban untuk pertanyaan asli adalah x 5(mod 13). Contoh 4.16 Di modulo 7, apakah ada x; y 2 N sedemikian sehingga x3 = 2y + 15? Penyelesaian. Diamati bahwa setiap pangkat dari 2 kongruen dengan 1; 2; atau 4 (mod 7). Jadi 2y + 15 = 2; 3; atau 5 (mod 7). Di sisi lain, karena pangkat tiga sempurna di modulo 7 yaitu 0; 1, dan 6, maka tidak mungkin terjadi 2y + 15 = x3 . Disimpulkan tidak ada x; y 2 N sedemikian sehingga x3 = 2y + 15. Contoh 4.17 Buktikan bahwa 2k ketika dibagi oleh 7. 5, k = 0; 1; 2; ::: tidak pernah mempunyai sisa 1 Bukti. Diamati bahwa 21 = 24 = ::: = 2, 22 = 25 = ::: = 4, 20 = 23 = 26 = ::: = 1 (mod 7). Jadi 2k 5 = 3; 4; atau 6 yang tidak lain adalah sisa atas pembagian oleh 7. Contoh 4.18 (AIME 1994) Barisan naik 3; 15; 24; 48; ::: (4.2) terdiri dari kelipatan positif dari 3 dan kurang satu dari suatu kuadrat sempurna. Berapakah sisa dari suku ke-1994 dari barisan tersebut ketika dibagi oleh 1000? Penyelesaian. Diinginkan 3 j n2 1 atau 3 j (n 1) (n + 1). Karena 3 adalah bilangan prima, ini mengharuskan n = 3k + 1 atau n = 3k 1, k = 1; 2; 3; :::. Barisan 3k + 1, k = 1; 2; ::: menghasilkan suku-suku n2 1 = (3k + 1)2 1 yang merupakan suku-suku di posisi genap dari barisan (4.2). Barisan 3k 1, k = 1; 2; ::: menghasilkan suku-suku n2 1 = (3k 1)2 1 yang merupakan suku-suku di posisi ganjil dari barisan (4.2). Selanjutnya harus dicari suku ke-997 dari barisan 3k + 1, k = 1; 2; :::, yaitu (3 997 + 1)2 1 = (3 ( 3) + 1)2 1 = 82 1 = 63 (mod 1000). Jadi, sisa yang dicari adalah 63. Contoh 4.19 (USAMO 1979) Tentukan semua penyelesaian tak negatif (n1 ; n2 ; :::; n14 ) di modulo 16, jika ada, dari persamaan n4 + n4 + 1 2 n4 = 1599. 14 (4.3)
  • 26. 22 Bab 4. Kongruensi Zn Penyelesaian. Semua pangkat 4 sempurna di modulo 16 adalah 0; 1 (mod 16). Ini berarti bahwa n4 + n4 + n2 1 2 14 paling besar adalah 14 (mod 16), padahal 1599 = 15 (mod 16). Jadi tidak ada penyelesaian untuk (4.3). Contoh 4.20 Diambil 1 2! n!! = n! 1 + 3! ( 1)n + n! : Buktikan bahwa untuk semua n 2 N, n > 3, n!! = n! (mod (n 1)) : Bukti. Dipunyai n! 1 2)! 1 2! ! n 1 n ( 1) ( 1) + + (n 1)! n! n!! = n (n = (n 1) m + ( 1)n = (n n ( 1)n n 1 1) (m + ( 1)n ) dengan m 2 Z, dan dicatat bahwa (n 4.2 1 + 3! 1) (n 1 n 1 + 2)! dapat dibagi oleh k; k n 2. Persamaan Kongruensi De…nisi 4.21 Bilangan bulat x0 yang memenuhi persamaan (kongruensi) dinamakan penyelesaian untuk persamaan tersebut. Pertama kali dipelajari persamaan linear terhadap penjumlahan. Persamaan linear (terhadap penjumlahan) dalam kongruensi: a + x = b (mod n) selalu mempunyai penyelesaian. Kunci dari penyelesaian persamaan tersebut adalah bilangan c sedemikian sehingga c + a = n. Contoh 4.22 Cari semua x yang memenuhi persamaan 7 + x = 4 (mod 5). Penyelesaian. Persamaan diubah menjadi 2 + x = 4 (mod 5) dan 3 + 2 + x = 3 + 4 (mod 5) 5 + x = 7 (mod 5) x = 2 (mod 5) :
  • 27. 23 Bab 4. Kongruensi Zn Berikutnya dipelajari persamaan linear terhadap perkalian dalam kongruensi: ax = b (mod n) : Kunci dari penyelesaian persamaan tersebut adalah bilangan c sedemikian sehingga ac = 1 (mod n). Tetapi hal ini tidak selalu terjadi. Sebagi contoh, diambil n = 4 dan a; c 2 f0; 1; 2; 3g. Jika a = 2 (mod 4), maka tidak ada c 2 f0; 1; 2; 3g sehingga 2c = 1 (mod 2). Contoh 4.23 Periksa apakah persamaan-persamaan berikut ini mempunyai penyelesaian: a) 2x = 1 (mod 4) b) 3x = 1 (mod 4) c) 12x = 8 (mod 15) Penyelesaian. Diperiksa satu persatu seperti berikut. a) Jika persamaan tersebut mempunyai penyelesaian, maka penyelesaiannya adalah 2x 1 = 4t, dengan t adalah suatu bilangan bulat. Karena ruas kiri adalah bilangan ganjil dan ruas kanan adalah bilangan genap, maka kesamaan tersebut tidak pernah terjadi. Jadi, persamaan kongruensi tidak mempunyai penyelesaian. b) Karena (3; 4) = 1, maka terdapat bilangan bulat p, q sehingga 3p + 4q = 1 atau 3p = 1 4q: Bilangan p = 3 dan q = 2. Jadi, persamaan di atas mempunyai penyelesaian x = p + 4r = 3 + 4r dengan r adalah suatu bilangan bulat. c) Jika persamaan tersebut mempunyai penyelesaian, maka terdapat x yang memenuhi 12x 8 = 15t: Ruas kanan dapat dibagi 3, maka ruas kiri harus dapat dibagi 3. Suku 12x habis dibagi 3, tetapi 8 tidak habis dibagi 3. Jadi persamaan tidak mempunyai penyelesaian. Contoh 4.24 Selesaikan setiap kongruensi di bawah ini. a) 5x = 7 (mod 12). b) 3x = 6 (mod 101) : c) 2x = 8 (mod 10) : Penyelesaian. Diselesaikan satu persatu seperti berikut.
  • 28. 24 Bab 4. Kongruensi Zn a) Dicatat bahwa 5 5 = 25 = 1 (mod 12). Karena itu 5 5x = 5 7 (mod 12) x = 35 (mod 12) = 11: b) Dicatat 34 3 = 102 = 1 (mod 101). Karena itu 34 3x = 34 6 (mod 101) x = 204 (mod 101) = 2: c) Di sini (2; 10) = 2, sehingga metode seperti di atas tidak bisa diaplikasikan. Berdasarkan de…nisi kongruensi dan keterbagian, diminta 2x 8 = k 10 = 0 (mod 10) untuk suatu k 2 Z. Persamaan dapat dituliskan menjadi 2 (x 4) = 0 (mod 10) = 10 dan memberikan persamaan x 4=0 atau x 4 = 5: Karena itu diperoleh penyelesaian untuk persamaan kongruensi yaitu x = 4 atau x = 9. Latihan 4.25 Cari penyelesaian untuk persamaan berikut ini. a) 5x = 4 (mod 11) : b) 3x = 7 (mod 17) : c) 9x = 4 (mod 49) : d) 100x = 7 mod 112 : Latihan 4.26 Jika ada, cari penyelesaian untuk persamaan berikut ini. a) 4x = 5 (mod 6) : b) 6x = 2 (mod 8) : c) 14x = 12 (mod 21) : d) 8x = 4 (mod 12) : Latihan 4.27 Untuk a = 1; 2; :::; 6, cari semua penyelesaian untuk persamaan ax = 1 (mod 7). Latihan 4.28 Cari semua bilangan bulat a dimana 1 a 5 sehingga ax = 1 (mod 6). Latihan 4.29 Diberikan bilangan-bilangan bulat a dan b. Jika 0 < a < 7 dan 0 < b < 7, tentukan a dan b sedemikian sehingga ab = 0 (mod 15).
  • 29. 25 Bab 4. Kongruensi Zn 4.3 Uji Keterbagian Berikut ini diberikan suatu contoh aturan keterbagian yang sangat terkenal. Teorema 4.30 (Casting-out 9’ Suatu bilangan asli n dapat dibagi oleh 9 jika dan s) hanya jika jumlahan dari digit-digitnya dapat dibagi oleh 9. Bukti. Diambil n = ak 10k + ak 1 10k 1 + + a1 10 + a0 sebagai ekspansi basisj = 1 (mod 9). Karena itu diperoleh 10 dari n. Untuk 10 = 1 (mod 9), dipunyai 10 n = ak + ak 1 + + a1 + a0 . Contoh 4.31 (AHSME 1992) Bilangan bulat dua digit dari 19 sampai 92 dituliskan secara berturutan untuk membentuk bilangan bulat 192021222324:::89909192: (4.4) Berapakah pangkat terbesar dari 3 yang membagi bilangan tersebut? Penyelesaian. Dengan penggunaan aturan casting-out 9, bilangan (4.4) dapat dibagi oleh 9 jika dan hanya jika 19 + 20 + + 91 + 92 = 372 3 Oleh karena itu, bilangan (4.4) dapat dibagi oleh 3 tetapi tidak oleh 9. Contoh 4.32 (IMO 1975) Ketika 44444444 dituliskan dalam notasi desimal, jumlahan dari digit-digitnya adalah A. Diambil B sebagai jumlahan dari digit-digit pada bilangan A. Tentukan jumlahan dari digit-digit pada bilangan B. (A dan B dituliskan dalam notasi desimal) Penyelesaian. Dipunyai 4444 = 7 (mod 9), karena itu 44443 = 73 = 1 (mod 9). Jadi 44444444 = 44443(1481) 4444 = 1 7 = 7 (mod 9). Diambil C sebagai jumlahan dari digit-digit pada bilangan B. Berdasarkan aturan casting-out 9, 7 = 44444444 = A = B = C (mod 9). Sekarang, 4444 log (4444) < 4444 log 104 = 17776. Ini berarti bahwa 44444444 mempunyai paling banyak 17776 digit, sehingga jumlahan dari digit-digit pada 44444444 adalah paling besar 9 17776 = 159984, yang berarti A 159984. Di antara semua bilangan asli 159984, bilangan yang mempunyai jumlahan digit terbesar adalah 99999, sehingga diperoleh B 45. Dari semua bilangan asli 45, bilangan yang mempunyai jumlahan digit terbesar adalah 39. Jadi jumlahan dari digit-digit B adalah paling besar 12. Tetapi karena C = 7 (mod 9), maka diperoleh C = 7. Jadi, jumlahan dari digit-digit pada bilangan B adalah 7. 4.4 Sisa lengkap De…nisi 4.33 Suatu himpunan x1 ; x2 ; :::; xn dinamakan sistem sisa lengkap (complete residue system) modulo n jika untuk setiap bilangan bulat y terdapat secara tepat satu indeks j sedemikian sehingga y = xj (mod n).
  • 30. 26 Bab 4. Kongruensi Zn Dalam hal ini jelas bahwa untuk sembarang himpunan berhingga A dari bilanganbilangan bulat, himpunan A akan membentuk himpunan sisa lengkap modulo n jika dan hanya jika himpunan A mempunyai n anggota dan setiap anggota dari himpunan tidak saling kongruen modulo n. Sebagai contoh, himpunan A = f0; 1; 2; 3; 4; 5g membentuk suatu himpunan sisa lengkap modulo 6, karena setiap bilangan bulat x kongruen dengan satu dan hanya satu anggota dari A. Himpunan B = f 3; 2; 1; 1; 2; 3g tidak membentuk himpunan sisa lengkap modulo 6 karena 3 = 3 (mod 6). Sekarang diperhatikan himpunan Zn = f0; 1; 2; :::; n 1g. Sebagai contoh, diambil n = 3 sehingga dipunyai Z3 = f0; 1; 2g. Elemen 0 menyatakan semua semua bilangan bulat yang dapat dibagi oleh 3, sedangkan 1 dan 2 berturut-turut menyatakan semua bilangan bulat yang mempunyai sisa 1 dan 2 ketika dibagi oleh 3. Dide…nisikan jumlahan pada Z3 seperti berikut ini. Diberikan a; b 2 Z3 , maka terdapat c 2 Z3 sedemikian sehingga a +3 b = c (mod 3). Tabel 4.1 memuat semua penjumlahan yang mungkin. Tabel 4.1: Tabel penjumlahan untuk Z3 . +3 0 1 2 0 0 1 2 1 1 2 0 2 2 0 1 Diamati bahwa Z3 bersama-sama dengan operasi +3 seperti yang diberikan dalam Tabel 4.1 memenuhi sifat-sifat: 1. Elemen 0 2 Zn merupakan suatu elemen identitas untuk Z3 , yaitu 0 memenuhi 0 +3 a = a +3 0 = a untuk semua a 2 Z3 . 2. Setiap elemen a 2 Z3 mempunyai suatu invers penjumlahan b, yaitu suatu elemen sedemikan sehingga a +3 b = b +3 a = 0. Invers penjumlahan dari a dinotasikan dengan a. Dicatat bahwa di Z3 dipunyai 0 = 0, 1 = 2, dan 2 = 1. 3. Operasi penjumlahan di Z3 adalah asosiatif, yaitu untuk setiap a; b; c 2 Z3 berlaku a +3 (b +3 c) = (a +3 b) +3 c. Selanjutnya dikatakan bahwa (Z3 ; +3 ) membentuk suatu grup (group) dan dinamakan grup dari sisa dibawah penjumlahan modulo 3. Secara serupa, dide…nisikan (Zn ; +n ) sebagai grup dari sisa dibawah penjumlahan modulo n. Latihan 4.34 Konstruksikan tabel penjumlahan untuk Z6 dan Z8 . Latihan 4.35 Berapa banyak pasangan berurutan (a; b) 6= 0 yang berbeda di Z12 sedemikian sehingga a +12 b = 0?
  • 31. Bab 5 Faktorisasi Tunggal 5.1 FPB dan KPK Diberikan a; b 2 Z dan keduanya tidak nol. Bilangan bulat positif terbesar yang membagi a dan b dinamakan faktor persekutuan terbesar (greatest common divisor ) dari a dan b, dan dinotasikan dengan (a; b). Dicatat bahwa jika d j a dan d j b maka d j (a; b). Sebagai contoh, (68; 8) = 2, (1998; 1999) = 1. Jika (a; b) = 1, maka a dan b dikatakan prima relatif (relatively prime) atau koprima (coprime). Jadi, jika a; b adalah prima relatif, maka keduanya tidak mempunyai faktor bersama yang lebih besar dari 1. Jika a; b 2 Z, keduanya tidak nol, bilangan bulat positif terkecil yang merupakan kelipatan dari a dan b dinamakan kelipatan persekutuan terkecil (least common multiple) dari a dan b, dan dinotasikan dengan [a; b]. Dicatat bahwa jika a j c dan b j c maka [a; b] j c. Berikut ini diberikan teorema-teorema yang berkaitan dengan faktor persekutuan terbesar. Teorema 5.1 (Teorema Bachet-Bezout) Faktor persekutuan terbesar, disingkat FPB, dari sembarang dua bilangan bulat a dan b dapat dituliskan sebagai kombinasi linier dari a dan b, yaitu terdapat bilangan bulat x; y dimana (a; b) = ax + by: Bukti. Dimisalkan F = fax + by > 0 : x; y 2 Zg. Jelas bahwa satu di antara a, b berada di F, untuk a dan b yang tak nol. Berdasarkan Prinsip Terurut Baik, F mempunyai elemen terkecil, misalnya d. Oleh karena terdapat x0 , y0 sedemikian sehingga d = ax0 + by0 . akan dibuktikan bahwa d = (a; b). Atau dengan kata lain akan dibuktikan bahwa d j a, d j b dan jika t j a, t j b maka t j d. Pertama kali akan dibuktikan d j a. Berdasarkan Algoritma Pembagian, dapat dicari bilangan bulat q, r, dengan 0 r < d sedemikian sehingga a = dq + r. Karena itu r=a dq = a q (ax0 + by0 ) = a (1 qx0 ) by0 : Jika r > 0, maka r 2 F lebih kecil daripada elemen terkecil d di F, yang kontradiksi dengan kenyataan bahwa d adalah elemen terkecil di F. Jadi r = 0. Akibatnya dq = a, yang berarti d j a. Dengan cara serupa dapat dibuktikan bahwa d j b. 27
  • 32. 28 Bab 5. Faktorisasi Tunggal Berikutnya diandaikan bahwa t j a dan t j b, maka a = tm dan b = tn untuk bilangan bulat m, n. Karena itu d = ax0 + by0 = t (mx0 + ny0 ), yang berarti t j d. Di sini jelas bahwa sembarang kombinasi linier dari a dan b dapat dibagi oleh (a; b). Akibat 5.2 Bilangan bulat positif a dan b adalah prima relatif jika dan hanya jika terdapat bilangan bulat x dan y sedemikian sehingga ax + by = 1. Lemma 5.3 (Lemma Euclid) Jika a j bc dan (a; b) = 1, maka a j c. Bukti. Untuk (a; b) = 1, berdasarkan Teorema Bachet-Bezout, terdapat bilangan bulat x; y dimana ax + by = 1. Karena a j bc, terdapat suatu bilangan bulat s dimana as = bc. Selanjutnya c = c 1 = cax + cby = cax + asy, yang berarti a j c. Teorema 5.4 Jika (a; b) = d, maka a b ; d d = 1: Bukti. Berdasarkan Teorema Bachet-Bezout, terdapat bilangan bulat x; y dimana a b a b ax + by = d. Karena itu diperoleh x+ y = 1 dimana , adalah bilangand d d d a b ; = 1: bilangan bulat. Disimpulkan bahwa d d Teorema 5.5 Jika c adalah suatu bilangan bulat positif, maka (ca; cb) = c (a; b) : Bukti. Diambil d1 = (ca; cb) dan d2 = (a; b). Akan dibuktikan bahwa d1 j cd2 dan cd2 j d1 . Untuk d2 j a dan d2 j b, maka cd2 j ca dan cd2 j cb. Jadi cd2 merupakan pembagi persekutuan dari ca dan cb, karena itu d1 j cd2 . Berdasarkan Teorema Bachet-Bezout, dapat ditemukan bilangan-bilangan bulat x; y dimana d1 = acx + bcy = c (ax + by). Tetapi karena ax + by merupakan kombinasi linier dari a dan b, maka ini dapat dibagi oleh d2 . Karena itu terdapat suatu bilangan bulat s sedemikian sehingga sd2 = ax+by. Ini berarti bahwa d1 = csd2 , artinya cd2 j d1 . Serupa dengan di atas, berlaku (ca; cb) = jcj (a; b) untuk sembarang bilangan bulat tak nol c. Lemma 5.6 Untuk bilangan-bilangan bulat tak nol a, b, c berlaku (a; bc) = (a; (a; b) c) : Bukti. Karena (a; (a; b) c) membagi (a; b) c dan (a; b) c membagi bc (menurut Teorema 5.5(a; b) c) maka (a; (a; b) c) membagi bc. Jadi (a; (a; b) c) membagi a dan bc, atau dituliskan (a; (a; b) c) j (a; bc). Di sisi lain, (a; bc) membagi a dan bc, karena itu (a; bc) membagi ac dan bc. Oleh karena itu, (a; bc) membagi (ac; bc) = (a; b) c. Jadi (a; bc) membagi a dan (a; b) c, atau dituliskan (a; bc) j (a; (a; b) c). Disimpulkan (a; bc) = (a; (a; b) c). Teorema 5.7 a2 ; b2 = (a; b)2 .
  • 33. 29 Bab 5. Faktorisasi Tunggal Bukti. Diandaikan bahwa (m; n) = 1. Diaplikasikan lemma sebelumnya dua kali untuk memperoleh m2 ; n2 = m2 ; m2 ; n n = m2 ; (n; (m; n) m) n : Untuk (m; n) = 1, ruas kanan dari pernyataan di atas sama dengan m2 ; n . Diaplikasikan kembali lemma di atas, diperoleh m2 ; n = (n; (m; n) m) = 1: Jadi (m; n) = 1 mengakibatkan m2 ; n2 = m2 ; n = 1. Berdasarkan Teorema 5.4, a b ; (a; b) (a; b) karena itu a2 b2 ; (a; b)2 (a; b)2 = 1, = 1. Berdasarkan Teorema 5.5, pernyataan terakhir dikalikan dengan (a; b)2 untuk memperoleh a2 ; b2 = (a; b)2 . Contoh 5.8 Diambil (a; b) = 1. Buktikan bahwa a + b; a2 ab + b2 = 1 atau 3. Bukti. Dimisalkan d = a + b; a2 ab + b2 . Berdasarkan Teorema Bachet-Bezout, sembarang kombinasi linier dari a + b dan a2 ab + b2 dapat dibagi oleh d. Karena itu d membagi (a + b) (a + b) + ( 1) a2 ab + b2 = 3ab: Karena itu d membagi a + b dan 3ab, akibatnya d membagi 3b (a + b) + ( 1) 3ab = 3b2 atau dituliskan d j 3b2 . Serupa dengan itu, diperoleh d j 3a2 . Jadi d j 3a2 ; 3b2 = 3 a2 ; b2 = 3 (a; b)2 = 3: Disimpulkan bahwa d = 1 atau 3. Contoh 5.9 (IMO 1959) Buktikan bahwa pecahan 21n + 4 adalah irreducible (tidak 14n + 3 dapat disederhanakan) untuk setiap bilangan asli n. Bukti. Untuk semua bilangan asli n dipunyai 3 (14n + 3) 2 (21n + 4) = 1. Jadi, berdasarkan Akibat 5.2, diperoleh bahwa pembilang dan penyebut adalah prima relatif, atau dengan kata lain tidak mempunyai faktor persekutuan yang lebih besar dari 1. Contoh 5.10 (AIME 1985) Bilangan-bilangan dalam barisan 101; 104; 109; 116; ::: mempunyai bentuk an = 100+n2 , n = 1; 2; :::. Untuk setiap n, diambil dn = (an ; an+1 ). Cari maksfdn gn 1 .
  • 34. 30 Bab 5. Faktorisasi Tunggal Penyelesaian. Diamati bahwa dn = 100 + n2 ; 100 + (n + 1)2 = = Jadi dn j 2 100 + n2 100 + n2 ; 100 + n2 + 2n + 1 100 + n2 ; 2n + 1 : n (2n + 1) atau dn j (200 dn j (2 (200 n). Oleh karena itu n) + (2n + 1)) atau dn j 401 untuk semua n. Jadi maksfdn gn 1 = 401. Contoh 5.11 Buktikan bahwa jika m dan n adalah bilangan-bilangan asli dan m adalah ganjil, maka (2m 1; 2n + 1) = 1. Bukti. Dimisalkan d = (2m 1; 2n + 1). Karena 2m 1 dan 2n +1 adalah ganjil, maka d haruslah suatu bilangan ganjil. Selain itu, dapat dituliskan 2m 1 = kd dan 2n +1 = ld untuk bilangan-bilangan asli k dan l. Oleh karena itu, 2mn = (kd + 1)n = td+1, dimana n 1 X n t= k n j dn j 1 . Melalui cara yang sama diperoleh 2mn = (ld 1)m = ud 1, j j=0 dengan menggunakan kenyataan bahwa m adalah ganjil. Untuk td + 1 = ud dapat dituliskan (u t) d = 2, haruslah d j 2. Akibatnya d = 1. Contoh 5.12 Berapa banyak bilangan bulat positif 1260? 1 atau 1260 yang prima relatif terhadap Penyelesaian. Karena 1260 = 22 32 5 7, sekarang masalahnya adalah mencari bilangan-bilangan yang lebih kecil dari 1260 dan tidak dapat dibagi oleh 2, 3, 5, atau 7. Diambil A menyatakan himpunan dari bilangan-bilangan bulat 1260 dan merupakan kelipatan dari 2, B adalah himpunan kelipatan dari 3, dan seterusnya. Berdasarkan Prinsip Inklusi-Eksklusi, jA [ B [ C [ Dj = jAj + jBj + jCj + jDj jA Bj jB Cj jA Cj jA Dj jB Dj jC Dj + jA B Cj + jA B Dj + jA C Dj + jB C Dj jA B C Dj = 630 + 420 + 252 + 180 +42 + 30 + 18 + 12 210 126 90 84 60 36 6 = 972: Jadi, banyaknya bilangan bulat positif 1260 972 = 288. 5.2 1260 yang prima relatif terhadap 1260 adalah Bilangan Prima dan Faktorisasi Diingat kembali de…nisi suatu bilangan prima, yaitu suatu bilangan bulat positif lebih besar dari 1 yang hanya mempunyai pembagi positif 1 dan dirinya sendiri. Jelas bahwa
  • 35. Bab 5. Faktorisasi Tunggal 31 hanya 2 yang merupakan bilangan prima genap, dan juga hanya 2 dan 3 yang merupakan bilangan-bilangan prima yang berturutan. Suatu bilangan, selain 1, yang tidak prima dinamakan bilangan composite. Jelas bahwa jika n > 1 adalah composite maka n dapat dituliskan sebagai n = ab, dimana 1 < a b < n dan a; b 2 N. Contoh 5.13 Tentukan semua bilangan bulat positif n untuk yang mana 3n 4, 4n 5, dan 5n 3 adalah bilangan-bilangan prima. Penyelesaian. Jumlah dari ketiga bilangan tersebut adalah 12n 12, yang jelas merupakan suatu bilangan genap, maka paling sedikit satu diantaranya adalah bilangan genap. Dipunyai bahwa bilangan prima genap hanyalah 2. Diamati bahwa 4n 5 tidak mungkin menjadi bilangan genap karena 4n selalu genap untuk setiap n, sehingga jika dikurangi suatu bilangan ganjil maka hasilnya ganjil. Tetapi 3n 4 dan 5n 3 adalah mungkin untuk menjadi bilangan genap. Karena itu diselesaikan persamaan 3n 4 = 2 dan 5n 3 = 2 yang secara berturutan menghasilkan n = 2 dan n = 1. Secara mudah bisa diperiksa bahwa n = 2 akan membuat ketiga bilangan tersebut adalah prima. Contoh 5.14 (AHSME 1976) Jika p dan q adalah prima, dan x2 mempunyai dua akar bulat positif berbeda, tentukan p dan q. px + q = 0 Penyelesaian. Diambil x1 dan x2 , dengan x1 < x2 , sebagai dua akar bulat positif yang berbeda. Karena itu bisa dituliskan x2 px+q = (x x1 ) (x x2 ), yang mengakibatkan p = x1 + x2 dan q = x1 x2 . Karena q adalah prima, maka x1 = 1. Jadi, q = x2 dan p = x2 + 1, yang berarti p dan q adalah dua bilangan prima yang berurutan, yaitu q = 2 dan p = 3. Teorema 5.15 Jika n > 1, maka n dapat dibagi oleh paling sedikit satu bilangan prima. Bukti. Karena n > 1, maka dipunyai paling sedikit satu pembagi > 1. Berdasarkan Prinsip Terurut Baik, n pasti mempunyai paling sedikit satu pembagi positif yang lebih besar dari 1, misalnya q. Diklaim bahwa q adalah prima. Jika q bukan prima maka dapat dituliskan q = ab, 1 < a b < q. Ini berarti bahwa a adalah suatu pembagi dari n yang lebih besar dari 1 dan lebih kecil dari q. Timbul kontradiksi dengan kenyataan bahwa q adalah minimal. Teorema 5.16 (Euclid) Terdapat tak hingga banyak bilangan prima. Bukti. Diandaikan terdapat berhingga banyak bilangan prima, misalnya p1 , p2 , ..., pn . Diambil N = p1 p2 pn + 1 Bilangan bulat N adalah lebih besar dari 1, sehingga berdasarkan teorema sebelumnya diperoleh bahwa N pasti mempunyai suatu pembagi prima p. Bilangan prima p haruslah salah satu dari bilangan-bilangan p1 , p2 , ..., pn . Tetapi, diamati bahwa p pasti berbeda dari sembarang p1 , p2 , ..., pn karena N mempunyai sisa 1 ketika dibagi oleh sembarang pi . Jadi timbul kontradiksi. Teorema 5.17 Jika bilangan bulat positif n adalah composite, maka n pasti mempunp yai suatu faktor prima p dengan p n.
  • 36. 32 Bab 5. Faktorisasi Tunggal p Bukti. Diandaikan bahwa n = ab, dimana 1 < a b < n. Jika a; b > n, maka p p n = ab > n n = n, yang adalah kontradiksi. Jadi n mempunyai suatu faktor 6= 1 p p dan n. Karena itu, faktor prima dari n adalah n. Contoh 5.18 Berapa banyak bilangan prima 100? p Penyelesaian. Diamati bahwa 100 = 10. Berdasarkan teorema sebelumnya, semua bilangan composite dalam range 10 n 100 mempunyai suatu faktor prima diantara 2, 3, 5, atau 7. Dimisalkan Am adalah himpunan bilangan-bilangan bulat positif yang merupakan kelipatan dari m dan 100. Diperoleh jA2 j = 50, jA3 j = 33, jA5 j = 20, jA7 j = 14, jA6 j = 16, jA10 j = 10, jA14 j = 7, jA15 j = 6, jA21 j = 4, jA35 j = 2, jA30 j = 3, jA42 j = 2, jA70 j = 1, jA105 j = 0, jA210 j = 0. Jadi, banyaknya bilangan prima 100 adalah = 100 (banyak bilangan composite 100) 1 = 100 + 4 (kelipatan dari 2; 3; 5; atau 7 = 100 + 4 (50 + 33 + 20 + 14) + (16 + 10 + 7 + 6 + 4 + 2) (3 + 2 + 1 + 0) 0 100) 1 1 = 25 dengan mengingat bahwa 1 bukanlah prima atau composite. Sekarang diperhatikan bilangan bulat 1332. Jelas bahwa bilangan tersebut dapat dibagi oleh 2, sehingga diperoleh 1332 = 2 666. Selanjutnya, 666 dapat dibagi oleh 6, sehingga 1332 = 2 2 3 111. Terakhir, 111 dapat dibagi oleh 3, sehingga diperoleh 1332 = 2 2 3 3 37. Karena 2, 3, 37 adalah bilangan-bilangan prima maka proses faktorisasi dari 1332 berhenti. Selanjutnya faktorisasi dari 1332 dapat dituliskan seperti 22 32 37. Faktorisasi demikian dinamakan faktorisasi kanonis (canonical factorisation). Teorema 5.19 (Teorema Fundamental Aritmatika) Setiap bilangan asli n mempunyai suatu faktorisasi tunggal dalam bentuk n = pa 1 pa 2 1 2 1 pa s s dimana pi adalah prima berbeda dan ai adalah bilangan bulat positif. Bukti. Diasumsikan bahwa n = pa 1 p a 2 1 2 b b pas = q11 q22 s b qt t merupakan dua faktorisasi kanonis dari n. Berdasarkan Lemma Euclid, disimpulkan bahwa setiap p pasti membagi suatu q dan setiap q membagi suatu p. Akibatnya s = t. Selanjutnya, dari p1 < p2 < < ps dan q1 < q2 < < qt disimpulkan bahwa pi = qi , 1 i s. Jika ai > bi untuk suatu i, atas pembagian oleh pai , diperoleh i p a 1 pa 2 1 2 pa i i bi pas = pb1 pb2 s 1 2 b b i+1 pi i 11 pi+1 pbs ; s yang adalah tidak mungkin, karena ruas kiri dapat dibagi oleh pi dan ruas kanan tidak dapat dibagi oleh pi . Serupa dengan itu, diperoleh hasil yang sama untuk ai < bi . Jadi haruslah ai = bi untuk semua i.
  • 37. 33 Bab 5. Faktorisasi Tunggal Contoh 5.20 Diambil p adalah suatu bilangan prima. Buktikan bahwa bilangan rasional. p p bukan suatu a p p Bukti. Diasumsikan bahwa p adalah rasional, artinya p = dimana a dan b b bilangan-bilangan asli yang prima relatif sebab faktor-faktor persekutuannya dapat dihapus. Karena itu bisa dituliskan pb2 = a2 . Jadi p j a2 dan juga p j a. Dituliskan a = a1 p untuk suatu bilangan bulat a1 , maka dipunyai pb2 = a2 p2 , sehingga b2 = a2 p yang 1 1 berarti p j b. Jadi, p adalah faktor persekutuan dari a dan b, sehingga ini kontradiksi p dengan asumsi. Jadi, p bukan suatu bilangan rasional. Contoh 5.21 Buktikan bahwa terdapat tepat satu bilangan asli n dimana 28 + 211 + 2n adalah pangkat dua sempurna. Bukti. Jika k 2 = 28 +211 +2n = 2304+2n = 482 +2n , maka k 2 482 = (k 48) (k + 48) = 2n . Berdasarkan faktorisasi tunggal diperoleh k 48 = 2s , k + 48 = 2t , s + t = n. Dari sini diperoleh 2t 2s = 96 = 3 25 atau 2s 2t s 1 = 3 25 . Berdasarkan faktorisasi tunggal, dari kesamaan terakhir diperoleh s = 5, t s = 2. Jadi, s = 5 dan t = 7, sehingga n = s + t = 12. Untuk suatu bilangan prima p, pk dikatakan membagi penuh (fully divide) n dan dituliskan pk k n jika k adalah bilangan bulat positif terbesar sedemikian sehingga pk j n. Contoh 5.22 (ARML 2003) Tentukan pembagi terbesar dari 1001001001 yang tidak melebihi 1000. Penyelesaian. Dipunyai 1001001001 = 1001 106 + 1001 = 1001 = 7 11 13 106 + 1 106 + 1 : 3 Berdasarkan rumus x6 + 1 = x2 + 1 = x2 + 1 x4 x2 + 1 , dituliskan 106 + 1 = 101 9901. Karena itu dapat dituliskan 1001001001 = 7 11 13 101 9901. Dari sini tidaklah sulit untuk memeriksa bahwa tidak ada kombinasi dari 7, 11, 13, dan 101 yang dapat menghasilkan suatu hasil kali yang lebih besar dari 9901 tetapi kurang dari 1000. Jadi jawabannya adalah 9901. n Contoh 5.23 Diambil n adalah suatu bilangan bulat positif. Buktikan bahwa 32 + 1 dapat dibagi oleh 2, tetapi tidak dapat dibagi oleh 4. n n Bukti. Jelas bahwa 32 adalah ganjil dan 32 + 1 adalah genap. Dicatat bahwa n 1 n n 1 2n 1 32 = 32 = 92 = (8 + 1)2 . Dipunyai rumus binomial (x + y)m = xm + m m x 1 1 y+ m m x 2 2 2 y + + m xy m m 1 1 + ym: Diambil x = 8, y = 1, dan m = 2n 1 dalam persamaan di atas, maka pada ruas kanan dapat dilihat bahwa setiap bagian jumlahan merupakan kelipatan dari 8 kecuali yang n terakhir (yaitu y m = 1). Karena itu sisa dari 32 ketika dibagi oleh 4 adalah sama n dengan 1, dan sisa dari 32 + 1 ketika dibagi oleh 4 adalah sama dengan 2. Contoh 5.24 Tentukan n sedemikian sehingga 2n k 31024 1 .
  • 38. 34 Bab 5. Faktorisasi Tunggal Penyelesaian. Dicatat bahwa 210 = 1024 dan x2 32 10 1 = = = 32 9 2 y 2 = (x + y) (x y). Karena itu 12 9 32 + 1 32 9 9 = 32 + 1 9 1 = 32 + 1 8 32 + 1 8 32 + 1 7 32 + 1 32 1 8 32 + 1 1 0 32 + 1 (3 1) : k Berdasarkan contoh sebelumnya, 2 j 32 + 1 untuk bilangan bulat positif k. Karena itu, dari persamaan terakhir dapat dilihat bahwa setiap faktor dapat dibagi oleh 2. 10 Karena banyaknya faktor adalah 11 dan juga 32 1 dapat dibagi oleh 20 , maka n = 11 + 1 = 12. 5.3 Teorema Fermat dan Teorema Euler Untuk sembarang bilangan bulat positif m dinotasikan ' (m) sebagai banyaknya bilangan bulat positif yang kurang dari m dan prima relatif terhadap m. Fungsi ' dinamakan fungsi totient Euler. Ini jelas bahwa ' (1) = 1 dan ' (p) = p 1 untuk sembarang bilangan prima p. Selain itu, jika n adalah suatu bilangan bulat positif sedemikian sehingga ' (n) = n 1, maka n adalah prima. Selain itu ' pk = pk pk 1 untuk semua bilangan bulat positif k, karena terdapat pk 1 bilangan bulat x yang memenuhi 0 x < pk yang dapat dibagi oleh p, dan bilangan-bilangan bulat yang prima relatif terhadap pk tidak dapat dibagi oleh p. Proposisi 5.25 Diambil m adalah suatu bilangan bulat positif dan a adalah bilangan bulat yang prima relatif terhadap m. Diandaikan bahwa S adalah sistem sisa lengkap modulo m. Himpunan T = aS = fas : s 2 Sg juga merupakan sistem lengkap modulo m. Proposisi di atas menyediakan dua teorema yang sangat terkenal dalam teori bilangan. Teorema 5.26 (Teorema Euler) Diambil a dan m adalah bilangan-bilangan bulat positif prima relatif, maka a'(m) = 1 (mod m) : Bukti. Diperhatikan himpunan S = a1 ; a2 ; :::; a'(m) yang terdiri dari semua bilangan bulat positif yang kurang dari m dan prima relatif terhadap m. Karena (a; m) = 1, dari proposisi sebelumnya diperoleh bahwa aS = aa1 ; aa2 ; :::; aa'(m) merupakan sistem sisa lengkap modulo m yang lain, maka (aa1 ) (aa2 ) ::: aa'(m) = a1 a2 :::a'(m) (mod m): Dengan penggunaan (ak ; m) = 1, k = 1; 2; :::; ' (m) diperoleh hasil yang diinginkan. Teorema 5.27 (Teorema Fermat) Diambil p adalah suatu bilangan prima, maka ap = a (mod p) untuk semua bilangan bulat a. Selain itu jika a adalah prima relatif terhadap p, maka ap 1 = 1(mod p)
  • 39. 35 Bab 5. Faktorisasi Tunggal Contoh 5.28 Diambil p adalah bilangan prima. Buktikan bahwa p membagi abp untuk semua bilangan bulat a dan b. bap Bukti. Dicatat bahwa abp bap = ab bp 1 ap 1 . Jika p j ab, maka p j (abp bap ); jika p - ab, maka (p; a) = (p; b) = 1, sehingga bp 1 = ap 1 = 1(mod p), berdasarkan Teorema kecil Fermat. Karena p j bp 1 ap 1 , akibatnya p j (abp bap ). Oleh karena itu p j (abp bap ) untuk semua p. Contoh 5.29 Diambil suatu bilangan prima p 7. Buktikan bahwa bilangan prima 11:::1 | {z } p 1 dapat dibagi p. Bukti. Dipunyai 11:::1 = | {z } 10p 1 1 9 p 1 ; dan kesimpulan diperoleh dari Teorema Fermat. (Dicatat juga bahwa (10; p) = 1). Contoh 5.30 Diambil suatu bilangan prima p 5. Buktikan bahwa p8 = 1(mod 240). Bukti. Dicatat bahwa 240 = 24 3 5. Berdasarkan Teorema Fermat, p2 = 1(mod 3) dan p4 = 1(mod 5). Karena suatu bilangan bulat positif adalah prima relatif terhadap 24 jika hanya jika bilangan bulat positif adalah ganjil, maka ' 24 = 23 . Berdasarkan Teorema Euler, dipunyai 28 = 1(mod 16). Oleh karena itu p8 = 1(mod m) untuk m = 3, 5, dan 6, yang berakibat p8 = 1(mod 240). Contoh 5.31 (IMO 2005) Diperhatikan barisan a1 ; a2 ; ::: yang dide…nisikan oleh an = 2n + 3n + 6n 1 untuk semua bilangan bulat positif n. Tentukan semua bilangan bulat positif yang prima relatif terhadap setiap suku dari barisan. Penyelesaian. Cukup ditunjukkan bahwa setiap bilangan prima p membagi an untuk suatu bilangan bulat positif n. Dicatat bahwa p = 2 dan p = 3 membagi a2 = 48. Diandaikan p 5. Berdasarkan Teorema Fermat, dipunyai 2p 1 = 3p 1 = 6p 1 = 1 (mod p), maka 3 2p 1 + 2 3p 1 + 6p 1 = 3 + 2 + 1 = 6(mod 6); atau 6 2p 2 + 3p 2 + 6p 2 1 = 0(mod p), artinya 6ap 2 dapat dibagi oleh p. Karena p adalah prima relatif terhadap 6, ap 2 dapat dibagi oleh p. Karena itu jawabannya adalah 1.
  • 40. Bab 6 Algoritma Euclid Sekarang akan diperiksa suatu prosedur yang menghindari pemfaktoran dua bilangan bulat positif untuk memperoleh faktor persekutuan terbesar. Ini dinamakan Algoritma Euclid dan digambarkan seperti berikut ini. Diambil a, b adalah bilangan-bilangan bulat tak nol. Setelah Algoritma Pembagian diaplikasikan secara berulang-ulang, diperoleh barisan kesamaan, dengan r0 = a dan r1 = b, r0 = q 1 r1 + r2 ; r1 = q 2 r2 + r3 ; r2 = q 3 r3 + r4 ; . . . rn rn rn 3 2 1 = q n 2 rn = q n 1 rn = q n rn : 0 < r2 < jr1 j ; 0 < r3 < r2 ; 0 < r4 < r3 ; . . . + rn 1 ; + rn ; 1 0 < rn 1 < rn 2 ; 0 < rn < rn 1 ; 2 Dicatat bahwa 0 rn < rn 1 < < r3 < r2 < b; karena itu pada akhirnya dicapai rn+1 yang sama dengan nol. Selain itu, diperhatikan bahwa barisan kesamaan memperbolehkan setiap rk , k = 2; :::; n dinyatakan dalam suku-suku dari rk 2 dan rk 1 . Sebagai contoh, dipunyai rn 1 = rn 2 qn 1 rn 1 : Digunakan pernyataan tersebut secara berulang, maka dapat dituliskan rn = ur0 + vr1 = ua + vb: Jadi rn dapat dinyatakan sebagai suatu kombinasi linier dari a dan b. Berdasarkan Teorema Bachet-Bezout, disimpulkan bahwa rn adalah FPB dari a dan b. Jadi, suku sisa tak nol terakhir rn yang dihasilkan oleh algoritma Euclid adalah (a; b). Selanjutnya, FPB dari dua bilangan bulat boleh dinyatakan sebagai suatu kombinasi linier dari dua bilangan bulat tersebut dengan menggunakan metode substitusi balik. Contoh 6.1 Tentukan (84; 60), dan selanjutnya nyatakan sebagai suatu kombinasi linier dari kedua bilangan bulat tersebut. 36
  • 41. 37 Bab 6. Algoritma Euclid Penyelesaian. Diambil a = 84 dan b = 60, maka 84 = 1 60 + 24; 60 = 2 24 + 12; 24 = 1 12; 24 = 84 + ( 1) 60; 12 = 60 + ( 2) 24; 12 = (84; 60) : Selanjutnya dikerjakan secara mundur untuk mendapatkan 12 = 60 + ( 2) 24 = 60 + ( 2) (84 + 60 ( 1)) = ( 2) 84 + 3 60. Jadi, (84; 60) = 12 = ( 2) 84 + 3 60. Contoh 6.2 Tentukan (190; 72), dan selanjutnya nyatakan sebagai suatu kombinasi linier dari kedua bilangan bulat tersebut. Penyelesaian. Diambil a = 190 dan b = 72, maka 190 = ( 2) ( 72) + 46; 72 = ( 2) 46 + 20; 46 = 2 20 + 6; 20 = 3 6 + 2; 6 = 3 2; 46 = 190 + 2 ( 72) ; 20 = 72 + 2 46; 6 = 46 + ( 2) 20; 2 = 20 + ( 3) 6; 2 = (190; 72) : Selanjutnya dikerjakan secara mundur untuk mendapatkan 2 = 20 + ( 3) 6 = 20 + ( 3) (46 + ( 2) 20) = ( 3) 46 + 7 20 = ( 3) 4 + 7 ( 72 + 2 46) = 7 ( 72) + 11 46 = 7 ( 72) + 11 (190 + 2 ( 72)) = 11 190 + 29 ( 72) : Jadi, (190; 72) = 2 = 11 190 + 29 ( 72). Ini juga dapat dilakukan dengan menggunakan kenyataan bahwa (190; 72) = (190; 72) dan dikerjakan seperti berikut ini. Contoh 6.3 Tentukan (190; 72), dan selanjutnya nyatakan sebagai suatu kombinasi linier dari kedua bilangan bulat tersebut.
  • 42. 38 Bab 6. Algoritma Euclid Penyelesaian. Diambil a = 190 dan b = 72, maka 190 = 2 72 + 46; 72 = 1 46 + 26; 46 = 1 26 + 20; 26 = 1 20 + 6 20 = 3 6 + 2; 6 = 3 2; 46 = 190 + ( 2) 72; 26 = 72 + ( 1) 46; 20 = 46 + ( 1) 26; 6 = 26 + ( 1) 20 2 = 20 + ( 3) 6; 2 = (190; 72) : Selanjutnya dikerjakan secara mundur untuk mendapatkan 2 = 20 + ( 3) 6 = 20 + ( 3) (26 + ( 1) 20) = ( 3) 26 + 4 20 = ( 3) 26 + 4 (46 + ( 1) 26) = 4 46 + ( 7) 26 = 4 46 + ( 7) (72 + ( 1) 46) = ( 7) 72 + 11 46 = ( 7) 72 + 11 (190 + ( 2) 72) = 11 190 + 29 ( 72) : Jadi, (190; 72) = 2 = 11 190 + 29 ( 72). Dicatat bahwa jika (a; b) = ua + vb, maka nilai-nilai u; v tidak tunggal. Sebagai contoh, 83 190 + 219 ( 72) = 2: Secara umum, bilangan-bilangan u; v dapat dimodi…kasi menjadi u + tb dan v karena (u + tb) a + (v ta) b = (ua + vb) + (tba tab) = ua + vb: ta Jadi, pendekatan-pendekatan berbeda untuk menentukan kombinasi linier dari (a; b) dapat menghasilkan jawaban-jawaban berbeda. Suatu persamaan yang meminta penyelesaian-penyelesaian bilangan bulat dinamakan persamaan diophantine. Berdasarkan Teorema Bachet-Bezout, diperhatikan bahwa persamaan diophantine linier ax + by = c (6.1) mempunyai suatu penyelesaian bilangan bulat jika dan hanya jika (a; b) j c. Algoritma Euclid merupakan suatu cara yang e…sien untuk mencari suatu penyelesaian bagi persamaan (6.1). Sebagai contoh, dari masalah sebelumnya, penyelesaian bilangan bulat untuk persamaan diophantine 190x + 72y = 2 adalah x = 11, y = 29. Contoh 6.4 Tunjukkan bahwa persamaan diophantine 206x + 446y = 40 mempunyai penyelesaian-penyelesaian bulat. Cari penyelesaian (x; y) sedemikian sehingga x + y mengambil nilai positif terkecil.
  • 43. 39 Bab 6. Algoritma Euclid Penyelesaian. Diaplikasikan Algoritma Euclid: 446 = 2:206 + 34 =) 206 = 6:34 + 2 =) 34 = 2:17: Karena (206; 446) = 2 dan 2 j 40, maka terdapat penyelesaian-penyelesaian bilangan bulat. Selanjutnya disubstitusi balik untuk memperoleh 2 = 206 6 34 = 206 6 (446 2 206) = 13:206 6:446: Sekarang, karena 40 = 20 2, maka dapat dituliskan 40 = 20 (13 206 6 446) = 260 206 Jadi, penyelesaiannya adalah x = 260 dan y = 120 446: 120. Penyelesaian umumnya adalah 446 t = 260 223t; 2 206 t = 120 + 103t; 120 + 2 x = 260 y = untuk suatu t 2 Z. Karena itu, x + y = 140 120t, dan t 2 Z, sehingga x + y bernilai positif terkecil untuk t = 1, bernilai negatif untuk t 2, dan x + y > 140 untuk t 0. Jadi, penyelesaian yang diminta yaitu x = 37 dan y = 17. Contoh 6.5 Cari semua penyelesaian bulat x dimana 0 x < 9 dari kongruensi linear 6x = 15 (mod 9), atau, jika tidak ada penyelesaian, berikan alasan kenapa tidak ada penyelesaian. Penyelesaian. Kita harus menyelesaikan persamaan diophantine 6x + 9y = 15 atau ekivalen dengan 2x + 3y = 5. Karena 2 ( 1) + 3 (1) = 1, maka 2 ( 5) + 3 (5) = 5. Oleh karena itu, suatu penyelesaiannya adalah x = 5. Karena (6; 9) = 3, semua penyelesaiannya mempunyai bentuk x= 5 9 t= 3 5 3t untuk suatu t 2 Z. Terdapat tiga penyelesaian mod 9, yaitu 3 ( 3) = 4, dan 5 3 ( 4) = 7. 5 3 ( 2) = 1, 5 Contoh 6.6 Vian ingin membeli beberapa prangko klas kedua dengan harga $20 per prangko, dan beberapa prangko klas pertama dengan harga $26 per prangko. Uang yang saya miliki 264. Berapa banyak prangko yang dapat dibeli oleh Vian? Penyelesaian. Dimisalkan x adalah banyaknya prangko klas kedua, dan y adalah banyaknya prangko klas pertama. Selanjutnya x; y 2 Z, dengan 20x + 26y = 264, dan x; y 0. Sekarang dicari suatu penyelesaian bilangan bulat dari persamaan (menggunakan Algoritma Euclid) 26 = 1 20 + 6 =) 6 = 3 2 + 0: Selanjutnya disubstitusi balik untuk memperoleh 2 = 20 3 6 = 20 3 (26 20) = 4 20 3 26:
  • 44. 40 Bab 6. Algoritma Euclid Jadi, 2 = 4 20 3 6. Sekarang 264 = 132 2, sehingga 264 = 132 (4 20 3 26) = 528 20 396 26: Dari sini, penyelesaian umumnya adalah 26 t dan 2 x = 528 + y= 396 20 t 2 untuk suatu t 2 Z. Dengan kata lain, x = 528 + 13t dan y = 396 10t untuk suatu t 2 Z. Diminta juga bahwa x dan y adalah tak negatif. x 0 ekivalen dengan 528 + 13t 0, 528 8 yaitu t (40 + 3 ). Untuk t 2 Z, harus dipunyai t 40. y 0 ekivalen 13 = 6 396 (39 + 10 ). Untuk t 2 Z, harus dipunyai dengan 396 10t 0, yaitu t 10 = t 40. Penyelesaian persekutuannya yaitu t = 40. Ini memberikan hasil: x = 528 + ( 40:13) = 8; dan y= 396 ( 40) 10 = 4: Dengan kata lain, Vian membeli 8 prangko klas kedua dan 4 prangko klas pertama. Contoh 6.7 Tentukan suatu penyelesaian bulat untuk persamaan 91x + 126y + 294z = 21: Penyelesaian. Dimulai dengan mencari (126; 294). Diaplikasikan Algoritma Euclid: 294 = 2 126 + 42 =) 126 = 3:42 + 0: Jadi, (126; 294) = 42. Selanjutnya disubstitusi balik untuk memperoleh 42 = 1:294 2:126: (6.2) Ini berarti bahwa (91; 126; 294) = (91; (126; 294)) = (91; 42). Diaplikasikan Algoritma Euclid: 91 = 2:42 + 7 =) 42 = 6:7 + 0; dan disubstitusi balik untuk memperoleh 7 = 1:91 2:42: (6.3) Sekarang dicari suatu penyelesaian bulat dari persamaan 91X + 42W = 21: Karena (91; 42) = 7 j 21, maka terdapat penyelesaian-penyelesaian bulat. Karena 21 = 3:7, maka dengan penggunaan (6.3) dan (6.2) dipunyai 21 = 3 7 = 3 (1 91 2 42) = 3 91 = 3 91 6 (1 294 2 126) = 3 91 6 294 + 12 126: 6 42
  • 45. 41 Bab 6. Algoritma Euclid Jadi, dipunyai penyelesaian bulat x = 3; y = 12; z = 6: Contoh 6.8 (HMMT 2002) Hitung x = 2002 + 2; 20022 + 2; 20023 + 2; ::: : Penyelesaian. Dicatat bahwa 20022 + 2 = 2002 (2000 + 2) + 2 = 2000 (2002 + 2) + 6: Jadi, berdasarkan Algoritma Euclid dipunyai 2002 + 2; 20022 + 2 = (2004; 6) = 6: Karena itu x j 2002 + 2; 20022 + 2 = (2004; 6) atau x j 6. Di sisi lain, setiap bilangan dalam barisan 2002+2; 20022 +2; 20023 +2; ::: dapat dibagi oleh 2. Lebih lanjut, karena 2002 = 2001 + 1 = 667 3 + 1, untuk semua bilangan bulat positif k, 2002k = 3ak + 1 untuk suatu bilangan bulat ak . Jadi, 2002k + 2 dapat dibagi oleh 3. Karena 2 dan 3 adalah prima relatif, setiap bilangan dalam barisan tersebut dapat dibagi 6. Oleh karena itu x = 6. Latihan 6.9 Tentukan 1. (34567; 987) 2. (560; 600) 3. (4554; 36) 4. (8098643070; 8173826342) Latihan 6.10 Selesaikan persamaan diophantine berikut ini, jika diketahui bahwa penyelesaiannya ada. 1. 24x + 25y = 18 2. 3456x + 246y = 44 3. 1998x + 2000y = 33 6.1 Sistem Kongruensi Linear Suatu sistem kongruensi linear dalam variabel x mempunyai bentuk 8 > a1 x = b1 (mod n1 ) > > < a2 x = b2 (mod n2 ) : . . > . > > : ar x = br (mod nr )
  • 46. 42 Bab 6. Algoritma Euclid Berikut ini diberikan suatu contoh yang mengilustrasikan bahwa hubungan antara modulo kongruensi-kongruensi merupakan syarat terpenting dalam menentukan apakah suatu sistem kongruensi linear mempunyai penyelesaian atau tidak. Contoh 6.11 Apakah sistem kongruensi x = 8 (mod 12) ; x = 6 (mod 9) mempunyai penyelesaian? Berikan penjelasan. Penyelesaian. Karena (12; 9) = 3 dan kongruensi pertama mengakibatkan x = 8 = 2 (mod 3), sedangkan kongruensi kedua mengakibatkan x = 6 = 0 (mod 3), maka sistem tidak mempunyai penyelesaian. Sifat penyelesaian dari suatu sistem kongruensi linear ditemukan pertama kali oleh matematikawan Cina kuno dan ditulis pertama kali dalam Shushu Jiuzhang (Nine Chapters on the Mathematical Arts) oleh matematikawan abad 13 Qin Jiushao. Teorema 6.12 (Teorema Sisa Cina) Jika n1 ; n2 2 Z+ adalah koprima dan b1 ; b2 2 Z, maka sistem kongruensi x = b1 (mod n1 ) ; x = b2 (mod n2 ) mempunyai suatu penyelesaian tunggal di modulo n1 n2 . Contoh 6.13 Selesaikan sistem x = 2 (mod 5) ; x = 1 (mod 3) : Penyelesaian. Dituliskan x = 2 (mod 5) menjadi x = 2 + 5m untuk suatu m 2 Z dan dituliskan x = 1 (mod 3) menjadi x = 1 + 3n untuk suatu n 2 Z. Disamakan kedua persamaan untuk memperoleh 2 + 5m = 1 + 3n atau 3n 5m = 1. Persamaan terakhir dapat diselesaikan menggunakan algoritma Euclid yang menyatakan kombinasi linear dari (3; 5) = 1: 3 2 + ( 5) 1 = 1: Jadi penyelesaiannya adalah m = 1 dan n = 2, sedangkan penyelesaian umumnya yaitu m = 1 + 3t dan karena 3 (2 + 5t) n = 2 + 5t; untuk suatu t 2 Z; 5 (1 + 3t) = 1. Jadi, x yang memenuhi kedua kongruensi yaitu x = 2 + 5 (1 + 3t) = 7 + 15t; t 2 Z; atau dengan kata lain x = 7 (mod 15). Contoh 6.14 Selesaikan sistem kongruensi 3x = 1 (mod 4) ; 5x = 2 (mod 7) : Penyelesaian. Dimulai dengan pengamatan bahwa 32 = 9 = 1 (mod 4) dan 3 5 = 15 = 1 (mod 7), karena itu sistem kongruensi asli ekivalen dengan sistem kongruensi x = 3 (mod 4) ; x = 6 (mod 7) :
  • 47. 43 Bab 6. Algoritma Euclid Selanjutnya sistem diselesaikan seperti pada contoh sebelumnya (diserahkan kepada pembaca sebagai latihan) untuk memperoleh penyelesaian bilangan bulat umum yaitu x = 27 (mod 28). Contoh 6.15 Tentukan semua penyelesaian bulat dari sistem kongruensi 7x = 1 (mod 8) ; x = 2 (mod 3) ; x = 1 (mod 5) : Penyelesaian. Diproses dalam dua tahap. Tahap pertama diselesaikan sistem kongruensi 7x = 1 (mod 8) ; x = 2 (mod 3) di modulo 24. Diamati bahwa 72 = 1 (mod 8), sehingga sistem kongruensi menjadi x = 7 (mod 8) ; x = 2 (mod 3) : Diaplikasikan Algoritma Euclid untuk memperoleh ( 1) 8+3 3 = 1; dan karena itu diperoleh penyelesaian tunggal x = ( 1) 8 2+3 3 7 = 23 (mod 24) : Tahap kedua diselesaikan sistem kongruensi x = 23 (mod 24) ; x = 1 (mod 5) : Dipunyai bahwa x = 23 (mod 24) = x= 1 (mod 24), sehingga sistem kongruensi menjadi 1 (mod 24) ; x = 1 (mod 5) : Diaplikasikan Algoritma Euclid untuk memperoleh ( 1) 24 + 5 5 = 1; dan karena itu diperoleh penyelesaian tunggal x = ( 1) 24 1+5 5 ( 1) = 71 (mod 120) : Jadi, penyelesaian bulat umum dari sistem yaitu x = 71 + 120n, n 2 N. Latihan 6.16 Selesaikan sistem persamaan berikut ini. a) x = 1 (mod 2) ; x = 2 (mod 3) : b) 3x = 1 (mod 5) ; 2x = 3 (mod 7) : c) x = 5 (mod 15) ; 4x = 7 (mod 11) : d) 2x = 3 (mod 5) ; 7x = 9 (mod 13) : Latihan 6.17 Selesaikan sistem persamaan berikut ini. a) x = 1 (mod 2) ; x = 1 (mod 3) ; x = 1 (mod 5) :
  • 48. Bab 6. Algoritma Euclid b) x = 1 (mod 2) ; x = 2 (mod 3) ; x = 4 (mod 5) : c) 2x = 1 (mod 3) ; 3x = 4 (mod 5) ; 3x = 7 (mod 8) : d) 5x = 2 (mod 3) ; 4x = 3 (mod 10) ; 5x = 9 (mod 25) : Latihan 6.18 Cari penyelesaian untuk sistem persamaan x = 1 (mod 3) ; x = 3 (mod 5) ; x = 5 (mod 7) ; x = 7 (mod 9) : 44
  • 49. Bab 7 Fungsi-fungsi Bilangan-Teoritik 7.1 Fungsi Floor Untuk suatu bilangan riil x, terdapat secara tunggal bilangan bulat n sedemikian sehingga n x < n + 1. Dengan kata lain, n adalah bilangan bulat terbesar yang tidak melebihi x, atau n dinamakan ‡oor dari x, dan dinotasikan dengan bxc. Selisih x bxc dinamakan bagian pecahan dari x dan dinotasikan dengan fxg. Bilangan bulat terkecil yang lebih besar atau sama dengan x dinamakan ceiling dari x dan dinotasikan dengan dxe. Jika x adalah suatu bilangan bulat, maka bxc = dxe dan fxg = 0; jika x bukan suatu bilangan bulat, maka dxe = bxc + 1. Berikut ini diberikan contoh-contoh sederhana: 1. b3; 1c = 3 dan d3; 1e = 4 2. b3c = 3 dan d3e = 3 3. b 3; 1c = 4 dan d 3; 1e = 3 Lemma 7.1 Untuk setiap x 2 R berlaku x 1 < bxc x: Bukti. Diambil n = bxc, maka dipunyai bahwa n x < n + 1. Hal ini memberikan bxc x, seperti dalam de…nisi. Ini juga memberikan x < n + 1 yang mengakibatkan bahwa x 1 < n, artinya x 1 < bxc. Selanjutnya, fungsi ‡oor memiliki sifat-sifat seperti dalam teorema berikut ini. Teorema 7.2 Jika ; 2 R, a 2 Z, n 2 N, maka (1) b + ac = b c + a j k b c = (2) n n (3) b c + b c b + c b c+b c+1 Bukti. (1) Diambil m = b + ac, maka m +a < m+1. Karena itu m a Ini berarti bahwa m a = b c atau m = b c + a. 45 < m a+1.
  • 50. 46 Bab 7. Fungsi-fungsi Bilangan-Teoritik (2) Dituliskan sebagai j k = + , 0 < 1. Karena n n n n bilangan bulat, berdasarkan (1) disimpulkan bahwa j j k k j k b c= n +n =n + bn c : n n Dipunyai 0 bn c maka diperoleh Ini berarti bahwa j k n adalah suatu bn c < 1. Jika diambil n n < n, sehingga 0 b c j k = + ; 0 n n j k b c = . n n = bn c , n < 1: (3) Dari ketaksamaan 1<b c dan 1<b c diperoleh + 2< b c+b c + . Karena b c + b c adalah suatu bilangan bulat yang kurang dari atau sama dengan + , maka b c+b c pasti kurang dari atau sama dengan bagian bulat dari + , yaitu b + c. Selain itu, + kurang dari bilangan bulat b c + b c + 2, sehingga b + c pasti kurang dari b c + b c + 2, dan akibatnya b + c < b c + b c + 2 menghasilkan b + c b c + b c + 1. Contoh 7.3 (APMC 1999) Diambil suatu barisan bilangan riil a1 , a2 , ... yang memenuhi ai+j ai + aj untuk semua i; j = 1; 2; :::. Buktikan bahwa a1 + a2 a3 + + 2 3 + an n an untuk semua bilangan bulat positif n. Bukti. Digunakan induksi kuat seperti berikut ini. Basis induksi untuk n = 1 dan 2 adalah trivial. Sekarang diasumsikan bahwa pernyataan benar untuk n k untuk suatu bilangan bulat positif k 2. Artinya, a1 a2 a1 + 2 a1 + a2 + 2 + a1 ; a2 ; . . . ak k ak : Dijumlahkan semua ketaksamaan untuk memperoleh ka1 + (k 1) a2 + 2 + ak k a1 + a2 + + ak : Selanjutnya kedua ruas dari ketaksamaan terakhir ditambahkan dengan a1 +a2 + yang menghasilkan (k + 1) a1 + a2 a3 + + 2 3 + an n (a1 + ak ) + (a2 + ak kak+1 : 1) + +ak , + (ak + a1 )
  • 51. 47 Bab 7. Fungsi-fungsi Bilangan-Teoritik Jika kedua ruas dari ketaksamaan terakhir dibagi dengan (k + 1), maka a2 a3 + + 2 3 a1 + + an n kak+1 k+1 atau a2 a3 an ak+1 + + + + 2 3 n k+1 Ini berarti pernyataan benar untuk n = k + 1. a1 + ak+1 : Contoh 7.4 (USAMO 1981) Untuk suatu bilangan positif x, buktikan bahwa bxc + b2xc b3xc + + 2 3 bnxc n + bnxc : Bukti. Berdasarkan Teorema 7.2 (3), ini merupakan kasus khusus dari Contoh 7.3 dengan mengambil ai = bixc. Contoh 7.5 (Putnam 1948) Jika n adalah suatu bilangan bulat positif, tunjukkan bahwa p p p n+ n+1 = 4n + 2 : Bukti. Diperhatikan bahwa 4n + 1 < p n+ p n+1 2 = 2n + 2 p n2 + n + 1 < 4n + 3 p p p p karena n2 + n > n2 = n dan juga n2 + n < n2 + 2n + 1 = n + 1. Karena itu diperoleh p p p p 4n + 1 < n + n + 1 < 4n + 3: Bilangan 4n+2 dan 4n+3 bukan merupakan bilangan kuadrat karena bilangan-bilangan kuadrat dalam mod(4) kongruen dengan 0 atau 1, sehingga p p 4n + 2 = 4n + 3 : Oleh karena itu p n+ p n+1 = p 4n + 2 : Contoh 7.6 (Australia 1999) Selesaikan sistem persamaan: x + byc + fzg = 200; fxg + y + bzc = 190; 1; bxc + fyg + z = 178; 8: Penyelesaian. Karena x = bxc + fxg untuk semua bilangan riil x, maka jumlahan dari tiga persamaan dalam sistem adalah 2x + 2y + 2z = 568; 9 atau x + y + z = 284; 45:
  • 52. 48 Bab 7. Fungsi-fungsi Bilangan-Teoritik Selain itu, jika persamaan terakhir dikurangi dengan setiap persamaan dalam sistem, maka diperoleh fyg + bzc = 84; 45; bxc + fzg = 94; 35; fxg + byc = 105; 65: Oleh karena itu bzc = bfyg + bzcc = b84; 45c = 84. Jadi, bzc = 84 dan fyg = 0; 45. Dengan cara serupa diperoleh bxc = 94 dan fzg = 0; 35, dan juga byc = 105 dan fxg = 0; 65. Dari sini dihasilkan x = 94; 65, y = 105; 45, dan z = 84; 35. 1 Contoh 7.7 (ARML 2003) Cari bilangan bulat positif n sedemikian sehingga pan p ling dekat dengan 123456789 . Penyelesaian. Dicatat bahwa 11111; 112 = 123456765; 4321 < 123456789 < 123456789; 87654321 = 11111; 11112 : Karena itu dan jp k 123456789 = 11111 np o 1 1 < 0; 11 < 123456789 < 0; 1111 < : 10 9 Contoh 7.8 (AIME 1997) Diandaikan bahwa a adalah positif, a 2 < a2 < 3. Cari nilai dari a12 144a 1 . 1 = a2 , dan Penyelesaian. Pertama kali dicatat bahwa hipotesis yang diberikan mengakibatkan a 1 = a 1 fkarena 1 < a dan 0 < a 1 < 1) dan a2 = a2 2. Karena itu a harus memenuhi persamaan a 1 = a2 2 atau a3 2a 1 = 0. Persamaan terakhir dapat dituliskan seperti (a + 1) a2 a 1 = 0 p 1+ 5 yang akar positifnya hanya a = . Digunakan hubungan a2 = a+1 dan a3 = 2a+1 2 untuk menghitung a6 = 8a + 5, a12 = 144a + 89, dan a13 = 233a + 144 yang mengakibatkan bahwa a12 144a 1 = a13 144 a = 233. Contoh 7.9 Cari semua penyelesaian riil untuk persamaan 4x2 40 bxc + 51 = 0:
  • 53. 49 Bab 7. Fungsi-fungsi Bilangan-Teoritik Penyelesaian. Dicatat bahwa (2x 17) = 4x2 3) (2x 40x + 51 2 4x 40 bxc + 51 = 0; yang memberikan 3 x 17 , dan berakibat 1 2 2 diberikan dapat dituliskan seperti p 40 bxc x= 2 bxc 51 8. Di sisi lain, persamaan yang ; sehingga dari sini dipunyai bxc = $p 40 bxc 2 51 % : Dari pilihan bxc 2 f1; 2; :::; 8g, hanya 2, 6, 7, atau 8 yang memenuhi persamaan terakhir. p p p p 29 Jadi, penyelesaian untuk x adalah 2 , 189 , 229 , dan 269 . 2 2 2 Proposisi 7.10 (Identitas Hermite) Jika x adalah suatu bilangan riil dan n adalah suatu bilangan bulat positif, maka bxc + x + 2 1 + x+ + n n + x+ n 1 n = bnxc : Bukti. Jika x adalah suatu bilangan bulat, maka hasil jelas benar. Diandaikan bahwa x bukan suatu bilangan bulat, artinya 0 < fxg < 1. Karena itu terdapat 1 i n 1 sedemikian sehingga fxg + i 1 n < 1 dan fxg + i n 1; (7.1) artinya n i n fxg < n i+1 : n (7.2) Berdasarkan (7.1) dipunyai bxc = x + dan x+ i = n 1 = n = x+ = x+ n 1 n i 1 n = bxc + 1; dan juga bxc + x + 1 2 + x+ + n n + x+ n 1 n = i bxc + (n i) (bxc + 1) = n bxc + n Di sisi lain, berdasarkan (7.2) diperoleh n bxc + n i n bxc + n fxg = nx < n bxc + n i + 1; i: (7.3)
  • 54. 50 Bab 7. Fungsi-fungsi Bilangan-Teoritik yang mengakibatkan bnxc = n bxc + n i: (7.4) Dari (7.3) dan (7.4) disimpulkan bahwa bxc + x + 2 1 + x+ + n n + x+ n 1 = n bxc + n n i = bnxc : Contoh 7.11 (AIME 1991) Diandaikan bahwa r adalah suatu bilangan riil dimana r+ 20 19 + r+ + 100 100 + r+ 91 = 546: 100 Cari b100rc. Penyelesaian. Jumlahan yang diberikan mempunyai 91 19+1 = 73 suku, dan setiap suku sama dengan brc atau brc + 1. Diamati bahwa 73 7 < 546 < 73 8, sehingga diperoleh brc = 7. Karena 546 = 73 7 + 35, maka 38 suku pertama bernilai 7 dan 35 suku terakhir bernilai 8; artinya r+ Akibatnya 7; 43 56 = 7 dan 100 r+ 57 = 8: 100 r < 7; 44, dan karena itu b100rc = 743. Contoh 7.12 (IMO 1968) Diambil x adalah suatu bilangan riil. Buktikan bahwa 1 X x + 2k 2k+1 k=0 = bxc : Bukti. Dalam identitas Hermite diambil n = 2 yang memberikan bxc + x + atau x+ 1 = b2xc ; 2 1 = b2xc 2 bxc : Diaplikasikan identitas terakhir secara berulang untuk memperoleh ! 1 1 1 X x + 2k X Xj x k j x k x 1 = + = = bxc : 2k+1 2k+1 2 2k 2k+1 k=0 7.2 k=0 k=0 Fungsi Legendre Diambil p adalah suatu bilangan prima. Untuk sembarang bilangan bulat positif n, dide…nisikan ep (n) = k sedemikian sehingga pk k n!. Fungsi aritmatika ep (n) dinamakan fungsi Legendre yang berasosiasi dengan bilangan prima p.
  • 55. 51 Bab 7. Fungsi-fungsi Bilangan-Teoritik jak Dicatat bahwa jika a > 0 dan n adalah suatu bilangan bulat positif, maka adalah n banyaknya bilangan bulat positif yang tidak melebihi a dan merupakan kelipatan dari n. Teorema 7.13 (Rumus Legendre) Jika n adalah suatu bilangan bulat positif dan p adalah suatu bilangan prima, maka ep (n) = k = 1 X j=1 n pj = n n n + 2 + 3 + p p p : Bukti. Diandaikan m 2 N dan 1 m n. Jika pr j m dan pr+1 - n, ingin dihitung kontribusi dari r. Dengan kata lain, ingin dihitung kontribusi 1 untuk setiap j 2 N sedemikian sehingga pj j m. Karena itu k= n 1 XX 1= m=1 j=1 pj jm 1 n XX 1= j=1 m=1 pj jm 1 X j=1 n pj dalam pandangan dari catatan di atas. Contoh 7.14 Diambil s dan t adalah bilangan bulat positif sedemikian sehingga 7s k 400! dan 3t k ((3!)!)!: Hitung s + t. Penyelesaian. Dicatat bahwa ((3!)!)! = (6!)! = 720!. Diaplikasikan rumus Legendre untuk memperoleh s = e7 (400) = 400 400 400 + + = 57 + 8 + 1 = 66 2 7 7 73 dan 720 720 720 720 720 + + + + 3 32 33 34 35 = 240 + 80 + 26 + 8 + 2 = 356: t = e3 (720) = Jadi s + t = 66 + 356 = 422. 7.3 Bilangan Fermat Untuk mencari semua bilangan prima berbentuk 2m + 1, Fermat melihat bahwa m harus merupakan pangkat dari 2. Tentu saja jika m = k h dengan k adalah suatu bilangan bulat ganjil yang lebih besar dari 1, maka 2m + 1 = 2h k + 1 = 2h + 1 2h(k 1) 2h(k 2) dan juga 2m + 1 tidak akan menjadi suatu bilangan prima. Bilangan-bilangan bulat n fn = 22 + 1; n 0 + 2h + 1 ;
  • 56. 52 Bab 7. Fungsi-fungsi Bilangan-Teoritik dinamakan bilangan-bilangan Fermat. Dipunyai f0 = 3; f1 = 5; f2 = 17; f3 = 257; f4 = 65537; dan f5 = 4294967297: Setelah diperiksa bahwa lima bilangan di atas adalah prima, Fermat menduga bahwa fn adalah prima untuk semua n. Tetapi Euler membuktikan bahwa 641 j f5 . Argumennya adalah seperti berikut: f5 = 232 + 1 = 228 54 + 24 = 641 2 28 5 27 2 639 640 + 1 4 + 1 = 228 641 6404 1 : Ini tetap tidak diketahui apakah terdapat tak berhingga banyak bilangan-bilangan Fermat prima. Contoh 7.15 Buktikan bahwa untuk bilangan bulat positif m dan n dengan m > n, fn membagi fm 2. Bukti. Diaplikasikan rumus a2 jukkan bahwa fm b2 = (a 2 = fm b) (a + b) secara berulang untuk menun1 fm 2 f1 f0 : Contoh 7.16 Untuk bilangan bulat positif berbeda m dan n, buktikan bahwa fm dan fn adalah prima relatif. Bukti. Berdasarkan contoh sebelumnya, dipunyai bahwa (fm ; fn ) = (f2 ; 2) = 1. Contoh 7.17 Buktikan bahwa untuk semua bilangan bulat positif n, fn membagi 2fn 2. Bukti. Dipunyai 2fn 2 = 2 22 2n 22 1 =2 n 22 n n 1 : (7.5) n Jelas bahwa 22 n adalah genap. Dicatat bahwa untuk suatu bilangan bulat positif 2n n 2m, x2m 1 dapat dibagi oleh x + 1. Karena itu x + 1 membagi x2 1. Dengan n n pengambilan x = 22 disimpulkan bahwa fn = 22 + 1 = x + 1 membagi 22 sehingga dari (7.5) diperoleh bahwa fn membagi 2fn 2. 7.4 n 22 n n 1, Bilangan Mersenne Bilangan-bilangan bulat Mn = 2n 1, n 1, dinamakan bilangan Mersenne. Ini jelas bahwa jika n adalah composite, maka Mn juga composite. Karena itu Mk adalah prima hanya jika k adalah prima. Selain itu, jika n = ab, dimana a dan b adalah bilanganbilangan bulat lebih besar dari 1, maka Ma dan Mb membagi Mn . Meskipun begitu terdapat bilangan prima n untuk yang mana Mn adalah composite. Sebagai contoh, 47 j M23 , 167 j M83 , 263 j M13 , dan seterusnya. Berikut ini diberikan suatu hasil tanpa bukti pada bilangan Mersenne. Teorema 7.18 Jika p adalah suatu bilangan prima ganjil dan q adalah suatu pembagi prima dari Mp , maka q = 2kp + 1 untuk suatu bilangan bulat positif k.
  • 57. 53 Bab 7. Fungsi-fungsi Bilangan-Teoritik 7.5 Bilangan Sempurna Untuk setiap bilangan bulat positif n dide…nisikan fungsi X m; (n) = mjn dimana m adalah bilangan bulat positif. Secara jelas, nilai (n) menyatakan jumlahan dari semua pembagi positif dari bilangan bulat positif n. Suatu bilangan bulat n 2 dimana (n) = 2n dikenal sebagai bilangan sempurna (perfect number ). Sebagai contoh, bilangan 6, 28, 496 adalah sempurna. Mudah dilihat bahwa 6 = 1+ 2+ 3 dan 28 = 1+ 2+ 4+ 7+ 14. Berikut ini diberikan dua hasil (tanpa bukti) pada bilangan sempurna. Teorema 7.19 Suatu bilangan bulat positif genap n adalah sempurna jika dan hanya jika n = 2k 1 Mk = 2k 1 2k 1 untuk suatu bilangan bulat positif k dimana Mk adalah prima. Lebih lanjut, tidak ada bilangan sempurna genap lainnya. Teorema 7.20 Jika n adalah suatu bilangan sempurna ganjil, maka faktorisasi prima dari n mempunyai bentuk 2b 2b 2b n = pa q 1 1 q 2 2 qt t ; dimana a dan p kongruen terhadap 1 modulo 4 dan t 2.
  • 58. DAFTAR PUSTAKA [1] Andreescu, T., D. Andrica, Z. Feng (2006). 104 Number Theory Problems: From the Training of the USA IMO Team. Birkhäuser Boston. [2] Baker, A. (2003). Algebra & Number Theory. Naskah. University of Glasgow. [3] Budhi, W.S. (2003). Langkah Awal Menuju ke Olimpiade Matematika. Ricardo. [4] Chen, W.W.L. (2003). Elementary Number Theory. Naskah. University of London. [5] Clark, W.E. (2002). Elementary Number Theory. Naskah. University of South Florida. [6] Santos, D.A. (2007). Number Theory for Mathematical Contests. GNU Free Documentation License. [7] Sato, N. (2009). Number Theory. CiteSeerX. [8] Wilkins, D.R. (2005). Part I: Topics in Number Theory. Naskah. 54